人人范文网 教学试卷

崇明县第一学期教学质量调研测试卷九年级语文(精选多篇)

发布时间:2021-08-07 07:59:57 来源:教学试卷 收藏本文 下载本文 手机版

推荐第1篇:第一学期九年级语文测试卷

2018年第一学期九年级语文测试卷

一:字音字形(每个1分)

1、下列加点字的注音,完全正确的一项是( ) A.妖娆(ráo) 重荷(hé) 苦心孤诣(yì) B.绯(fēi)红 阔绰(chuò) 惟妙惟肖 (xiāo) C.丰硕(shuò) 馈(kuì)赠 根深蒂(dì)固 D.芳馨(xīn) 鞭挞(dá) 叱咤(zhà)风云

2.选出下列词语中加点字的读音完全正确的一项( ) A、峰峦(lán) 妖娆(ráo) 即物起兴(xìng) B、蝉蜕(tuì) 执拗(ào) 迥乎不同(jiǒng) C、哂笑(shěn) 滑稽(jī) 越俎代庖(páo) D.骈进(pián) 匀称(chèn) 苦心孤诣(zhǐ) 3.下列词语中加点的字读音全都正确的一项是( )

A.缄(xián)默 迸(bèng)发 苗圃(pǔ) 津(jīn)津有味 B.哀号(háo) 溃(kuì)退 忐(tǎn)忑 长吁(yū)短叹 C.哺(bǔ)育 诘(jié)问 蓬蒿(gāo) 歇(xiē)斯底里 D.亢(kàng)奋 啜(chuò)泣 沉湎(miǎn) 随声附和(hè) 4.下列词语中加点的字读音全都正确的一项是( )

A.恐吓(hè) 愤懑(mèn) 埋怨(mái) 校对(jiào) 吹毛求疵(cī) B.徙倚(xǐ) 慰藉(jí) 羸弱(léi) 眼睑(jiǎn) 恪尽职守(kè) C.沉浸(jìn) 哂笑(shěn) 鞭挞(tà) 炽痛(chì) 舐犊情深(shì) D.荫庇(yìn) 干瘪(biě) 纤巧(qiàn) 惬意(qiè) 龙吟凤哕(huì) 5.下列词语词语中加点字注音完全正确的一项是( ) A.馈赠(kuì) 荣膺(yún) 隐匿(nì) 怡然自得(tái) B.晨曦(sén) 阔绰(chò)涸 干(gù) 山崩地裂(bōng) C.窒息(zhì) 簇新(zú) 蹒跚(cè) 鳞次栉比(jié) D.胚芽(pēi) 糍粑(cí) 胆怯(qiè) 重蹈覆辙(zhé) 6.下列各组词语中,加点字读音没有错误的一项是( ) A.丫杈(chà) 长吁(xū)短叹 应和(hè) 猝(cuì)然长逝 B. 羁绊(pàn ) 中流砥(dǐ)柱 蛮横(hénɡ) 戛(jiá)然而止 C.媲(pì)美 引颈受戮(lù) 拗(ào)口 根深蒂(dì)固

D.绮(qǐ)丽 莫衷(chōnɡ)一是 炮制(páo) 怏怏(yānɡ)不乐 7.下列加点字的读音完全正确的一项是( )

A.编辑(jī)

簇(cù)新

圈(quān)养

讪(shàn)笑 B.慰藉(jiè)

绮(qǐ)丽

纤(xiān)细

迸(bìng)溅 C.静谧(mì)

掺(chān)和

殷(yān)红

孪(luán)生 D.蹒(mǎn)跚

社稷(jì)

澎湃(pài)

行(háng)辈 8.下列词语中加点字的字音、字形完全正确的一项是( )

A.发酵 (jiào) 潜行(qián) 决别(jué) 转弯抹角(mǒ) B.琐屑 (xuè) 两栖 (qi) 胆怯 (qiè) 应接不瑕(xiá)

C.倔强(jué) 拮据(jié) 颓唐(tuí)

一代天骄(jiāo) D.取缔(tì)

哺育(bu) 烦躁(zào) 暗然失色(àn) 9.下列加粗字读音全都相同的一组是:( ) A.惋伤

绾髻

婉言谢绝

音容宛在 B.拾掇

舵手

惰性气体

腐化堕落 C.扁柏

扁豆

神医扁鹊

一叶扁舟 D.肖像

俏皮

硝烟弥漫

销声匿迹

10.下列词语中加点字的注音 有错误的一项是( )

A.堕落(duò) 弥漫(mí) 迥然不同(jiǒng)

B.干瘪(biě)

叮嘱(shǔ)

苦心孤诣(zhǐ)

C.取缔(dì)

蜷伏(quán) 叱咤风云(zhà)

D.嗔视(chēn) 阻抑(yì)

面面相觑(qù)

11、下列字形和加点字注音全部正确的一项是( ) A、厄运(è)

牟取 参差不齐(cī)

顾名思意 B、哂笑(xī) 隔膜

迥乎不同(jiǒng) 吹毛求疵 C、喑哑(yīn) 沉湎

草长莺飞(yīng)

海誓山盟 D、濯洗(zhuó) 征兆

怡然自得yí) 因地治宜 12.下面加点字的注音全都正确的一项是(

A.阔绰(chuò) 羼水(chàn) 恣睢(zìsuī) 惴惴不安(zhuì) B.亲家(qìn) 系(jì)鞋带 溺爱 (nì) 露出马脚(lòu)

C.聒噪(ɡuō) 悖谬(bó)

倔强(juějiànɡ) 分道扬镳(biāo) D.浣妆(huǎn) 赔偿(chánɡ)惘然(wǎnɡ) 肆无忌惮(dàn) 13.下列词语中加点的字,读音有错误的一项是( )

A.迸(bèng)溅 伫(zhù)立 咆(páo)哮体

颔(hàn)首低眉

B.砭(biān)骨 炽(chì)热 堰塞(sè)湖 硕果累(léi)累

C.哺(pǔ)育 窥伺(sì) 扎(zhā)小辫 惟妙惟肖(xiào)

D.檄(xí)文 蜜饯(jiàn) 一抔(póu)土 吹毛求疵(cī) 14.下列加点字的注音完全正确的一项是( )

A.痴(chī)想 扒(bá)窃 气冲斗(dǒu)牛 义愤填膺(yīng) B.磐(pán)石 亢(kàng)奋 颔(hàn)首低眉 期期艾艾(ài ) C.寒噤(jìn) 藩篱([fān) 戛(gá)然而止 龙吟凤哕(huì) D.襁(qiǎng)褓 拮据(jié) 鳞次栉(jié)比 怏怏(yàng)不乐 15.下列词语中加点字的读音,完全正确的一项是(

) A.洗濯(dí) 畸形(qí)

B.慰藉(jí) 嗔怒(diān) C.沉湎(miǎn)

酝酿(niànɡ) D.纤维(qiān) 澎湃(bài) 16.下列词语中,加点字的读音完全正确的一项是( ) A.花圃(pǔ) 棘手(là) 迥乎不同(jiǒng) B.稽首(qǐ) 喑哑(yīn) 吹毛求疵(cī) C.慰藉(jì) 枯涸(hé) 骇人听闻(hài) D.狡黠(xiá) 剽悍(piāo) 锲而不舍(qì) 17.下列词语中, 没有错别字的一项是( ) A.尴尬 威摄 广袤无垠 B.荒谬 孪生 迫不急待 C.亵渎 真谛 恪尽职守 D.愧作 褶皱 咫尺天涯

18.下列加点字的注音全对的一项是(

A.倔强(juè) 聒噪(guō) 度德量力(duó) B.粗犷 (guǎng) 梦魇(yǎn) 锲而不舍(qiè) C.讣告(fù) 针砭(biǎn) 令人咋舌(zhá) D.亘古(gěn) 骊歌(lí) 期期艾艾(ài) 19.下列词语中,没有错别字的一项是(

) A.陨落 借鉴 班门弄斧 B.泛烂 点缀 醍醐灌顶 C.积攒 暇想 脍炙人口 D.孺子牛 愧怍 胜卷在握

20.下列词语中,每对加点字读音全都相同的一项是( )

A.重现/重荷 差使/参差不齐 B.崎岖/畸形 门楣/媚上欺下 C.撩拨/嘹亮 踌躇/更胜一筹 D.较量/气量 拮据/引经据典 21.下列词语中加点的字,读音有错误的一项是( )

A.迸(bèng)溅 伫(zhù)立 咆(páo)哮体

颔(hàn)首低眉

B.砭(biān)骨 炽(chì)热 堰塞(sè)湖 硕果累(léi)累

C.哺(pǔ)育 窥伺(sì)

扎(zhā)小辫 惟妙惟肖(xiào)

D.檄(xí)文 蜜饯(jiàn) 一抔(póu)土 吹毛求疵(cī) 22.下列词语中,没有错别字的一项是( ) A.宽恕 相得益章 剽悍 洗耳恭听 B.愕然 无可质疑 寒噤 暗然失色 C.烦燥 肃然起敬 愧怍 慷概淋漓 D.祷告 迫不及待 臃肿 一拍即合

23.下列加点字的注音正确的一项是(

A.褴褛(lǔ) 栈(zhàn)桥 头盔(kuī) 钟灵毓(sù)修

B.祈(qí)祷 粗犷(kuǎng) 气氛(fēn) 味同嚼(jiáo)蜡

C.酝酿(niàng) 尴(gān)尬 谙(ān)熟 乳臭(xiù)未干

D.唱和(hè) 恣睢(zuī) 肖(xiào)像 风雪载(zǎi)途 24.下列各组词语中,加点字的注音有误的一项是(

A芳馨(xīn) 亵渎(dú) 寒噤(jìn) 豆蔻年华(kòu) B.荣膺( yīng) 恣睢(suī) 嗔怒(chēn) 锲而不舍(qiè) C愧怍(zuò) 剽悍(biāo) 轩榭(xiè) 戛然而止(jiá)

D.遒劲(qiú) 哂笑(shěn) 娴熟(xián) 广袤无垠(mào) 25.请选出下列词语中加点字读音完全正确的一项是( ) A.骊歌(lí) 应和(hé) 门楣(méi) 强聒不舍(guā) B.惬意(qiè) 差使(chāi) 梦魇(yǎn) 骇人听闻(hài) C.凛冽(lǐng)感慨(kài) 蜿蜒(wān) 重峦叠嶂(luán) D.点缀(zhuì)迸溅(bìng)脊椎(jí) 戛然而止(jiá)

26.下列词语的字形与加点字的读音完全正确的一项是(

) A.黄晕(yūn) 惬(qiè)意 舌苔(tāi) 义愤填膺 B.狩(shòu)猎 蝉蜕(tuì) 差(chā)劲 精巧绝伦 C.沉湎(miǎn) 朔(shuò)方 落(lào)枕 气势磅礴 D.蟠(pán)龙 凛(lǐng)然 着(zhuó)落 如法炮制 27.加点字注音完全正确的一项是( ) A.吮吸(shǔn) 脊梁(jí)阔绰(chuò) 荣膺(yīng) B.鞭挞(tà) 冗长(rǒng) 缄默(jiān) 呜咽(yè) C.炽热(chì) 诘责(jié) 悲恸(tòng) 星宿(sù) D.恐吓(xià) 狙击(jū) 回溯(sù) 荫庛(yìn) 28.语中加点字的读音全都正确的一组是( )。

A. 暂时(zhán) 栖息(qī) 纤细(qiān) 诲人不倦(huì) B.瑰丽(guī) 泥泞(nìng) 拮据(jí) 海市蜃楼(shèn) C. 解剖(pōu) 粗犷(guǎng) 挑衅(xìn) 苦心孤诣(yì) D. 蓦然(mù)陨落(yǚn) 贮藏(chù) 揠苗助长(yà) 29.下列加点字的注音全都正确的一项是(

) A.滑稽(jī) 佝偻(ɡōu) 越俎代庖(bāo) B.酝酿(niàng) 藩篱(fān) 鸿鹄之志(hú) C.行辈(háng) 蟾蜍(zhān) 觥筹交错(gōng) D.曝晒(bào) 告罄(qìng) 锲而不舍(qiè) 30.下列各组词语中加点字注音完全正确的一项是(

) A.恻隐(cè) 峰峦(luán) 譬如(pì) 情不自禁(jīn) B.攒动(zuǎn) 莅临(wèi) 凛然(lǐn) 随声附和(hè) C.憎恶(è) 骊歌(lí) 炽热(zhì) 强聒不舍(guō)

D.蹊跷(qiào) 证券(quàn) 赡养(shàn) 踉踉跄跄(qiàng) 31.下列各组词语中字形完全正确的一项是。(

) A.忌讳 弥漫 鄙夷 相题并论 B.锁屑 颓唐 拮据 孜孜不倦 C.颤抖 蜷曲 剽悍 矫揉造作 D.徇职 黯然 造诣 人情事故

32.下列各组词语中加点字注音完全正确的一项是(

) A.恻隐(cè) 峰峦(luán) 譬如(pì) 情不自禁(jīn) B.攒动(zuǎn) 莅临(wèi) 凛然(lǐn) 随声附和(hè) C.憎恶(è) 骊歌(lí) 炽热(zhì) 强聒不舍(guō)

D.蹊跷(qiào) 证券(quàn) 赡养(shàn) 踉踉跄跄(qiàng) 33.下列各组词语中字形完全正确的一项是。(

) A.忌讳 弥漫 鄙夷 相题并论 B.锁屑 颓唐 拮据 孜孜不倦 C.颤抖 蜷曲 剽悍 矫揉造作 D.徇职 黯然 造诣 人情事故

34选出下列词语在加点字的字音、字形完全正确的一项是() A.贮藏(zhù)屏障(zhàng)提防(dī)空穴来风(xuè) B.琴弦(xián)褴褛(lán)蓓蕾(lěi)人才济济(jǐ) C.蝙蝠(biān)冒然(mào)狭窄(zǎi)相形见绌(chù)

D.创伤(chuāng)笨拙(zhuō)症结(zhèng)一泄千里(xiè) 35.下面词语加点字注音完全正确的一项是( )

A.埋怨(mái) 荫蔽(yīn) 账簿(bù) 豁然开朗(huò) B.框架(kuāng) 吊唁(yàn) 绮丽(qǐ) 差强人意(chā) C.污秽(huì) 扶掖(yè) 哂笑(shěn) 连篇累牍(dú) D.颔联(hán) 粗犷(guǎng) 倏忽(shū) 长吁短叹(xū) 36.下列词语书写完全正确的一项是( ) A.天轮之乐 迫不及待 人声鼎沸 张皇失措 B.无动于衷 惟妙惟肖 袖手旁观 通宵达旦 C.相得益章 因地制益 妇孺皆知 历尽心血 D.正襟危坐 顾名思义 暗然失色 穷困潦倒 37下列加点字音完全正确的一项是( )

A.栈桥(jiàn) 襁褓(qáng) 蹑手蹑脚(liè) B.荇藻(xìn) 陨落(yǚn) 成吉思汗(hàn) C.馈赠(guì) 恣睢(zhì) 辙乱旗靡(fēi) D.缟素(gǎo) 扶掖(yè) 周道如砥(dǐ)

38.下列词语中加点的字,注音完全正确的一项是( ) A.锃亮(zèng) 熏陶(tāo) 谥号(shì) 白雪皑皑(kǎi) B.省亲(xǐng) 上颌(hé) 跻身(jī) 居心叵测(pǒ) C.啜泣(cuò) 束缚(fù) 恪守(gè) 鳞次栉比(zhì) D.门槛(kǎn) 租赁(lìn) 联袂(jué) 义愤填膺(yīng) 39.下列词语中书写完全正确的一项是( ) A.苦脑 矫健 针砭时弊 妄费心机 B.引伸 谗言 耀武扬威 轻歌慢舞 C.羁绊 简陋 鬼鬼祟祟 再接再厉 D.蹒跚 坦诚 屈指可数 千均一发

40.下列词语中加点字注音完全正确的一组是( ) A. 嫉妒(jì) 蔷薇(wéi) 教学相长(zhǎng) B. 胴体(dòng) 裙袂(mèi) 龙吟凤哕(huì) C. 怪诞(dàn) 糍粑(chí) 呷浪之鳞(jiā) D. 鲑鱼(guī) 感慨(kǎi) 蹑手蹑脚(liè) 41.下列加点字注音完全正确的一项是( ) A.踟蹰(chú) 吞噬(shì) 浩浩汤汤(shāng) 气吞斗牛(dǒu) B.雎鸠(jū) 忏悔(chàn) 乐此不疲(pǐ) 选贤与能 (jǔ) C.俨然(yǎn) 荒谬(miù) 花团锦簇(zú) 千山万壑(hè) D.滞留(zhì) 扶掖(yè) 踉踉跄跄(qiàng) 引颈受戮(lǜ) 42.下列书写完全正确的一项是( )

A.亵读 煞白 更胜一筹 芸芸众生 B.阔绰 诘问 物竟天择 持之以恒 C.羁绊 恻隐 张皇失措 歇斯底里 D.睥睨 荫庇 相形见拙 沿溯阻绝

43.下列词语中加点的字,每对读音都不同的一项是( ) A.踏实/踏青 奢靡 /风靡一时 低声悄语/悄然泪下 B.孵化/浮雕 鞠躬/笑容可掬 风流倜傥/丝绸之路 C.角色/角逐 推磨/磨杵成针 烟熏火燎/星火燎原 D.旷野/粗犷 剽悍/膘肥体壮 踌躇满志/铸就辉煌 44.下列词语中没有错别字的一项是( ) A.壁垒 易拉罐 娇生惯养 独竖一帜 B.咋舌 顶梁柱 责无旁贷 黯然失色 C.静谧 蒸溜水 花枝招展 立竿见影 D.提练 满堂彩 无动于衷 别出心裁

45.下列词语中加点字注音全部正确的一项是(

A.伫立(zhù) 镌刻(juàn) 胼胝(dǐ) 豁然开朗(huò) B.伛偻(lǚ) 竣工(jùn) 邂逅(hòu) 半身不遂(suí) C.裨益(bì) 女红(hóng) 庇护(pì) 揠苗助长(yà) D.篛蓬(ruò) 连翘(qiáo) 剔透 (tì) 五行缺土(háng) 46.下列词语中加点字注音完全正确的一项是( ) A.庇护(bì) 祷告(dǎo) 招徕(lái) B.挑衅(pàn) 匀称(chèn) 嘹亮(liáo) C.蓦然(mò) 嗤笑(cī) 田圃(pǔ) D.倒坍(tān) 栖息(qī) 酷肖(xiào) 47.下列词语中,没有错别字的一项是( ) A.因地制宜 隐性埋名 花枝招展 B.风尘苦玄 义愤填膺 张皇失措 C.肃然起敬 油光可鉴 涕泗横流 D.吹毛求疵 怡然自得 相安无是

48.下列加点字的注音正确的一项是( ) A.嫉妒(jí) 休憩(qì) 戏谑(nuè) 苦心孤诣(zhǐ) B.狡黠(xiá) 荤菜(hūn)

逾越(yú) 坦荡如砥(dǐ) C.引吭(kàng)栖息(qī)

隘口(ài) 拈轻怕重(zhān) D.娴熟(xián) 绰号(chuò) 星宿(sù) 呱呱坠地(guā) 49.下列词语中加点宇的宇音、字形完全正确的( ) A.畅想(chàng)

棱角(líng) 险俊(jùn) 销声匿迹(xāao) B.倾倒(qīng)

烘托(hōng) 稀罕(hān) 容光涣发(huàn) C.豁达(huò) 赐教(cì) 勾勒(lè) 老谋深算(móu) D.处理(chù) 宽敞(chǎng) 键谈(jiàn)举世闻名(wén) 50、下列词语中加点的字,注音完全正确的一项是( ) A、瞥见(piē) 什物(shí) 蓦然回首(mù ) 味同嚼蜡(jiáo)

B、慰藉(jiè) 气馁( něi) 苦心孤诣(yì) 惟妙惟肖(xiào) C、暂时(zàn) 粗糙(cāo) 寻章摘句(zh āi) 长吁短叹(yū) D、呜咽(yàn) 脊梁(jǐ) 鲜为人知(xiǎn) 恃才放旷(shì)

二:成语(每个3分)

1.下列句中加点词语使用不恰当的一项是( )

A.盛开的紫藤萝花吸引不少人驻足观赏,那一片辉煌的淡紫色,像一条瀑布,从空中垂下。 B.夕阳像一个巨人被捆缚了手脚,使出浑身解数,憋红了脸,在天边挣扎。

C.阅读课上,同学们像一只只勤奋的小蜜蜂,扑在书上,呕心沥血地吮吸着知识的营养。 D.湖边的那棵柳树,依旧舒展着美丽的枝桠,随风轻轻地摇曳着它妖娆的身子。 2.下列句子中加点成语运用不当的一项是(

A.聪明的重庆人因地制宜,依山建楼,跨水筑桥,让重庆呈现一种独特的美。

B.晚上,我悠闲地在花园里散步,明亮的路灯戛然而止,四周一片漆黑,原来是停电了。 C.课间十分钟,班上的“开心果”张晓松使尽浑身解数,搞怪逗乐,整个教室充满了快活的气氛。

D.留学生易文婷对故乡的思念与日俱增,收看央视国际频道成了她每天的必修课。 3.依次填入下列句子横线处的词语,最恰当的一项是(

) 家风,是一个家庭或家族长期以来形成并____ 的道德操守和处世方法。家风中蕴藏着先人所___的价值理念和道德规范,也包含着简单朴素的为人______ 。千百年来,家风在一个个家庭里承载、延续,奠定了整个民族文明的基础。 A.传播

恪守

规则

B.传承

保守

规则 B.传承

恪守

准则

D.传播

保守

准则 4.下列句子中加点的词语使用恰当的一项是(

A.父母亲或者其他监护人应当自觉学习家庭教育知识,正确履行对未成年人的监护职责和赡养义务。

B.在实体书店深受互联网冲击的背景下,育人书店的管理团队精诚合作,苦心钻营,使书店利润稳步增长。

C.八年后记者重访地震重灾区,只见居民住宅鳞次栉比,农贸市场人声鼎沸,处处呈现一派安定祥和的景象。

D.这次青少年官组织的国学知识竞赛,试卷题量多、难度大,令我叹为观止,深感自己传统文化知识储备不足

5.选出以下各句加点成语使用正确的一项。(

) A.除夕之夜,腾空而起的礼花和不绝于耳的爆响声交相辉映。(《人民日报》2011-2 -4) B.“白马撵蹄近相映,欢声四合壮士呼。”无论是打球的,还是观看比赛的,全都精神亢奋、呼天抢地。

(《沈阳日报》2014- 2- 21) C.白纸上,我还没有种上一粒字。多半天,我一文不名。终于,在一阵鸽哨声里,我写下了第八次开头。

(《散文》2014年第4期)

D.这个十年,季羡林在《中国蔗糖史》的基础上,又完成了80万字的《糖史》。这部呕心沥血之作,被季羡林视为平生最重要的著作。

(《中国剪报》2015-10-27) 6.下列句子中加点成语的使用恰当的一项是(

A.文艺晚会上,学生们自编自演的节目绘声绘色,街舞、相声、小品等都赢得了阵阵掌声。 B.上海迪斯尼公司认为经济放缓不会阻止游客蜂拥而至,因为它注入了鲜明的中国元素。 C.两位阔别多年的老同学意外地在杭州西湖湖畔萍水相逢,别提有多高兴了。

D.这是一个脏、乱、差的居住小区,楼道里贴的像牛皮癣一样的各类小广告琳琅满目。 7.下列句子中加点成语使用恰当的一项是(

A. “趵突腾空”为“济南八景”之首,泉水日夜喷涌,络绎不绝。

B.作家曹文轩荣获儿童文学最高奖—国际安徒生奖,消息传来,骇人听闻。 C.杨绛先生的离世,如巨星陨落,使中国文坛黯然失色。

D.漫步曲水亭街,我们用相机去捕风捉影,记录家乡的古风古韵。 8下列加点的成语使用不恰当的一项是(

A.对于跑鞋来说,舒适的重要性不言而喻。跑鞋制造商也是绞尽脑汁,重点考虑消费者双脚的感受。

B.来到兵马俑博物馆,你会发现,每个兵马俑的表情都惟妙惟肖,古代匠人们的工艺实在令人震撼。

C.我钦佩《荒野求生》的男主角贝尔,因为面对艰苦的生存环境,他不以为然,总能乐观面对困难。

D.目前,全球大约有20亿人拥有智能手机,智能手机销量逐年上涨,普及程度让平板电脑相形见绌。

9下列加点的成语使用不正确的一项是(

A.中国人民对战争带来的苦难有着刻骨铭心的记忆,对和平有着孜孜不倦的追求。 B.卓凡批评同桌写作文经常文不加点,错字百出。

C.九旬高龄的老教授在二中礼堂讲授国学,他旁征博引,侃侃而谈。

D.长河公园景色宜人,周围高楼鳞次栉比,你不会想到几年前这里还是一片荒地。 10下列加点的成语使用有误的一项是(

) A.国文和英文,则必须学有素养,临阵磨枪、临时抱佛脚,是不行的。 B.观赏日出的人们,无不赞叹太阳升起时的巧夺天工。 C.这舞松之风更把云雾吹得千姿万态,令人眼花缭乱。

D.在西部,沙子一旦摆脱了水,它们就会纵横捭阖,所向披靡。 11下列语句中加点的成语使用有误的一项是( )

A.在世界杯羽毛球女子双打决赛中,中国队的两位姑娘配合得天衣无缝,获得冠军。 B.孩子向家长倾吐心声时,家长应洗耳恭听,这是家庭沟通中特别需要注意的地方。 C.几经周折,老人终于回到了阔别三十年的故乡,长年漂泊在外的愁绪荡然无存了。 D.杨绛先生为人谦逊低调,做学问孜孜不倦,钱钟书先生评价她:最贤的妻,最才的女。 12下列加点成语使用错误的一项是(

A.近年来,泸州加快了城市化建设的步伐。鳞次栉比的高楼大厦,风景如画的滨江大道,雄伟壮观的跨江大桥,无不彰显出酒城特有的魅力。

B. 随着光伏电站市场的开启,越来越多的非光伏企业也开始涌进这一领域。这些具有创新意识的企业,或许将成为光伏行业的中流砥柱。

C. 理解小说中的人物形象,必须从整体入手,要紧扣文体特点,联系人物生活的环境及作者的创作意图,任何断章取义的分析都是不可取的。

D. 5月下旬,发生在广州台山市凤凰峡的漂流事故导致8名游客死亡。事件发生后,有些部门推卸自己的管理责任,这真让人叹为观止。

13下列各句中加点词语的使用,不正确的一句是(

A.爱迪生试验电灯的消息,通过报纸传遍了全美国,也轰动了大西洋两岸。 B. 在集训途中,因为海拔过高缺水,李国瑞甚至只能用尿液使自热米饭升温。 C. 在这个封闭的山村,对一些简单的科学常识,这里的人们都会觉得不可理喻。 D. 世界上的艺术珍品已是洋洋大观,但它们都曾经历了时间的考验与筛选。 14下列句子中,加点的成语或俗语使用不恰当的一项是(

A.在美丽乡村建设中,不能“眉毛胡子一把抓”,要找准着力点,软硬兼施,内外兼修,使乡村既有颜值担当,更有内涵气质。

B.2016年4月4日,曹文轩问鼎世界儿童文学最高荣誉——国际安徒生奖。这标志着中国儿童文学已经走向世界,也是中国文学多年发展指日可待的结果。

C.家风是一种“软约束”,通过潜移默化的影响,实现对家庭成员行为、作风、操守的有效约束。

D.中国历来不乏出色的工匠,如游刃有余的庖丁,刻木成鸟的鲁班……他们不仅是工艺纯熟的技者,还是明法通道的智者。

15下列各句中加点的成语,使用正确的一项是(

A. 京剧大师梅葆玖先生演技高超,他在传统京剧《霸王别姬》中扮演虞姬,演得惟妙惟肖,活灵活现。

B.陈忠实老师看到出版社的工作条件很差,就自掏腰包两万元,为改善我们的工作条件尽了一点绵薄之力。

C.中国首个“4.0”产业园——潍坊国际创意港5月8日开园,国际机构争相入园,众多资深创客也心无旁骛。

D.与广袤的宇宙相比,人类只是沧海桑田,极其渺小,只有遵循规律,按规律办事,才能实现“天人”和谐。

三:文学常识(每个5分)

1.下列各项中表述有误的一项是( ) A.虎妞是小说《骆驼祥子》中的主要人物之一,她大胆泼辣甚至有点变态,后因难产而死。 B.《水浒》中的鲁智深疾恶如仇、侠肝义胆,围绕他展开的情节有:拳打镇关西、大闹五台山、倒拔垂杨柳、醉打蒋门神等。

C.童话色彩只是《格列佛游记》这部小说的表面特征,尖锐而深刻的讽刺才是其灵魂所在。比如,作者描写小人国利立浦特的党派之争以鞋跟高低划分阵营,这是挖苦英国两个争斗不休的政党;讲述利立浦特与邻国兵戎相见,是影射当时英法两国连年征战。

D.《诗经》是我国最早的一部诗歌总集,收录了从西周到春秋时期的诗歌305篇,这些诗歌分为“风”“雅”“颂”三个部分。

2、下列说法有误的一项是:()

A、在人际交往中,我们要尽量做到用语得体。比如:初次见面用“久违”,好久不见用“久仰”,请人帮忙用“拜托”,麻烦别人用“劳驾”。

B、古代称谓词很多,其中“令爱”指别人的女儿,“令妹”指别人的妹妹,“高足”指别人的有才华的学生,“泰山”指岳父。

C、“令”是一种敬辞。所以,我们在称呼别人的父母时,可用“令尊”和“令堂”这样的称呼。

D、汉语中常用“烽烟”代战争,“手足”代兄弟,“桃李”代学生,“鸿雁”代书信。

3、下列关于作家、作品的表述,错误的一项是:(

A.记叙的顺序通常有顺叙、倒叙和插叙等,《羚羊木雕》和《爸爸的花儿落了》都采用了插叙手法。

B.丹麦童话作家安徒生的《皇帝的新装》写了一个皇帝受骗上当的故事,主要是为了告诉人们要识破骗子的阴谋诡计。

C.“城春草木深”中的“城”指长安城,“留取丹心照汗青”中的“汗青”特指史册。

D.《隆中对》中诸葛亮为刘备勾画了一幅战略蓝图,《出师表》中又为后主刘禅提出了以“亲贤远小”为核心的三条建议。

4、下列说法有误的一项是:()

A、中国古代神话名篇有:女娲补天、后羿射日、精卫填海、(盘古)开天辟地、黄帝战蚩尤。

B、我国第一位田园诗人是东晋的陶渊明(陶潜),他“不为五斗米折腰”。

C、《诗经》“六义”指:风、雅、颂(分类)、赋、比、兴(表现手法)。

D、古代把山的北面或水的南面叫做阳,山的南面或水的北面叫阴。如“河阴”便是黄河北岸。

5、下列说法有误的一项是:()

A、《论语》是春秋战国时期儒家学派的创始人孔子所著的一本书,记录的是孔子的言行。

B、《史记》是我国第一部纪传体通史,全书一百三十篇。作者是西汉史学家、文学家司马迁。被誉为“史家之绝唱,无韵之离骚”。

C、王羲之是东晋杰出的书法家,被称为“书圣”,他的《兰亭集序》帖是我国古代书法艺术最灿烂的瑰宝,被称为“天下第一行书”。

D、莫泊桑是法国作家,被称为短篇小说的巨匠,代表作品有《我的叔叔于勒》《项链》等

推荐第2篇:九年级语文第一学期调研检测

九年级语文第一学期调研检测

卷首寄语:亲爱的同学们,当你走进考场,你就是这里的主人。只要心境平静,只要细心、认真地阅读、思考,你就会感到试题并不难,一切都在你的掌握之中。别忘了,对自己说一声:我能行!

一、积累运用(20分)

1、请你给下面这段话中的两个加点字注音,并改正这段话中的两个错别字。(2分)

人生如同登山。初出发时,仰首望处,群峰揽秀,令人心情激荡,浮想连翩。于是拾级而上,沿途花团锦簇,浓阴滴翠,美不胜收。但这些都挽留不了我们前进的脚步,更不会让我们与成功分道扬镖,因为我们知道,无限风光在险峰。到达峰顶,我们将会欣赏更多绝美的风景,才能体悟生命的精髓„„生命之美,美在进取!

拾级(

精髓(

2、名句(8分)

①、晓战随金鼓,

②、

,正是河豚欲上时。

(苏轼《春江晚景》) ③、竹外桃花三两枝,

④、

,望峰息心;经纶世务者,窥谷忘反。(《与朱元思书》)

⑤、且壮士不死即已,

。(《陈涉世家》)

⑥、衣带渐宽终不悔,

。 (柳永《凤栖梧》)

⑦、

,固国不以山溪之险,

。 ⑧、众里寻他千百度。

,那人却在,

。(辛弃疾《青玉案》)

3、名著阅读(4分)

祥子是老舍的长篇代表作《__________》中的一个人物形象。这部作品描写来自农村的淳朴健壮的祥子,到北平谋生创业,_____次买车又______次失去,并终于堕落到生活的谷底的故事。__________,是这部小说的基本线索,与祥子有着密切关系的女主人公是___

__。

4、材料题。(3分)

五千年灿烂悠久的中华文化曾经在人类文明史上创造过无数的奇迹,孔子和孟子便是这些奇迹中两颗耀眼的明星,他们超人的智慧、深邃的思想、积极的人生态度,影响了一代又一代的人。 请阅读下面的几段材料,并回答问题。

材料一:东汉时,《论语》进入经书行列,成为读书人的必读之书。南宋理学家朱熹将《大学》《中庸》与《论语》《孟子》合编为“四书”,其中《论语》被列为首位。元明清六百年间,“四书”一直是科举考试的主要内容。

材料二:据统计,日本现存德川幕府时期的《论 语》研究论著就将近九十种。日本近代资本主义“创业者”的涩泽荣一(1840—1931),在从事实业的四十余年里,一直坚持亲自向企业员工讲授《论语》。 材料三:十六世纪下半叶,欧洲传教士将《论语》首次传到西方,很多欧洲著名人物,对孔子的思想、人格都十分崇拜。直到现在,还有不少西方思想家,力图在孔子学说中寻求解决伦理道德方面问题的启示。 材料四:

2004年11月21日,全球第一所“孔子学院”在韩国首都首尔挂牌。截止2007年9月,全球已经启动孔子学院(包括孔子学校、孔子课堂)175所,分布在156个国家和地区。国内61所高校和机构参与孔子学院的合作办学。到2010年,全球将建成500所孔子学院和孔子课堂。 ① 你从上述材料中得出什么结论?(1分)

②当今社会还需要孔孟之道吗?或者说在这个讲究竞争,讲究效益的时代,孔孟之道还有用吗?谈谈你的理解。(2分)

答:

5、阅读下面这段话回答问题。(3分)

①读报是每天都要做的事,积累报纸资料也要有恒心,否则往往事半功倍。②一疏忽,一间断,就有可能把有价值的资料白白遗漏。③陈毅同志在给儿女的诗中告戒道:“应知学问难,在于点滴勤。”④积累报纸资料,就要发挥这种“点滴勤”的精神。

⑪文中有一个错别字,是

,应改为

。(1分)

⑫文中有一个成语使用不当,是

,应改为

。(1分)

⑬文中第

句是病句,应改为

。(1分)

二、理解、感悟(50分)

(一)诗歌赏析(4分) 春雪

(唐)韩愈

新年都未有芳华,二月初惊见草芽。 白雪却嫌春色晚,故穿庭树作飞花。

6、第二句中的“惊”字表达了诗人怎样的心情?(2分)

7、

三、四两句用了什么修辞手法?请简要赏析。(2分)

(二)阅读下面文言文,完成8—11题(16分)

(甲)晋太元中,武陵人捕鱼为业。缘溪行,忘路之远近。忽逢桃花林,夹岸数百步, 中无杂树,芳草鲜美,落英缤纷。渔人甚异之。复前行,欲穷其林。

林尽水源,便得一山,山有小口,仿佛若有光。便舍船,从口入。初极狭,才通人。复行数十步,豁然开朗。土地平旷,屋舍俨然,有良田美池桑竹之属。阡陌交通,鸡犬相闻。其中往来种作,男女衣着,悉如外人。黄发垂髫,并怡然自乐。

(乙)老人引杨氏入山之大穴。鸡犬陶冶,居民之大聚落也。至一家,老人谓曰:“此公欲来,能相容否?”对曰:“老人肯相引至此,则必贤者矣。吾此间衣服、饮食、牛畜、丝纱、麻之属,皆不私藏,与众共之,故可同处。子果来,勿携金珠锦绣珍珠异等物,所享者惟薪米鱼肉,此殊不缺也。惟计口授地,以耕以蚕,不可取食于人耳。”杨氏谢而从之。又戒曰:“子来或迟,则封穴矣。”迫暮,与老人同出。

(南宋•康与之《昨梦录》)

8、解释下列句中加点字。(4分)

(1)渔人甚异之 (

(2)有良田美池桑竹之属(

(3)老人引杨氏入山(

(4)迫暮,与老人同出(

9、下列各组句子中加点的文言虚词意义和用法相同的一项是(

)(2分) A、忘路之远近

老人引杨氏入山之大穴

B、以耕以蚕

以草火烧 C、杨氏谢而从之

水落而石出者

D、不可取食于人也

苛政猛于虎也

10、翻译下面文言句子。(4分)

①见渔人,乃大惊,问所从来。

②所享者惟薪米鱼肉,此殊不缺也。

11、阅读理解。(6分)

(1)甲乙文段都以人物的

为线索,甲文段按

顺序进行记叙描写,乙文段是通过

来描写洞内生活情况的。(3分)

(2)甲乙文段所写的农民生活是有共同之处的,你能用简洁的语言概括出共同点吗?你相信两文段所写生活的真实性吗?为什么?(3分)

(三)阅读下面文言文,完成12—16题(13分)

【甲】蒋氏大戚,汪然出涕曰:“君将哀而生之乎?则吾斯役之不幸,未若复吾赋不幸之甚也。向吾不为斯役,则久已病矣。自吾氏三世居是乡,积于今六十岁矣,而乡邻之生日蹙。殚其地之出,竭其庐之入,号呼而转徙,饥渴而顿踣,触风雨,犯寒暑,呼嘘毒疠,往往而死者相藉也。曩与吾祖居者,今其室十无一焉,与吾父居者,今其室十无二三焉,与吾居十二年者,今其室十无四五焉,非死则徙尔,而吾以捕蛇独存。悍吏之来吾乡,叫嚣乎东西,隳突乎南北,哗然而骇者,虽鸡狗不得宁焉。吾恂恂而起,视其缶,而吾蛇尚存,则弛然而卧。谨食之,时而献焉。退而甘食其土之有,以尽吾齿。盖一岁之犯死者二焉,其余则熙熙而乐,岂若吾乡邻之旦旦有是哉。今虽死乎此,比吾乡邻之死则已后矣,又安敢毒耶?” 余闻而愈悲。孔子曰:“苛政猛于虎也。”吾尝疑乎是,今以蒋氏观之,犹信。呜呼!孰知赋敛之毒有甚是蛇者乎?故为之说,以俟夫观人风者得焉。 (节选自柳宗元《捕蛇者说》)

【乙】田家少闲月,五月人倍忙。

右手秉遗穗,左臂悬敝筐。

夜来南风起,小麦覆陇黄。

听其相顾言,闻者为悲伤。

妇姑荷箪食,童稚携壶浆。

家田输税尽,拾此充饥肠。

相随饷田去,丁壮在南冈。

今我何功德,曾不事农桑。

足蒸暑土气,背灼炎天光。

吏禄三百石,岁晏有余粮。

力尽不知热,但惜夏日长。

念此私自愧,尽日不能忘。

复有贫妇人,抱子在其旁。

(白居易《观刈麦》)

12、解释下列加点的词。(4分)

(1)弛然而卧

(2)苛政猛于虎 (

(3)故为之说

(4)但惜夏日长

13、用现代汉语写出下列句子的意思。(4分)

(1)君将哀而生之乎?

(2)孰知赋敛之毒有甚是蛇者乎?

14、用“/”线划出下面句子的朗读节奏。(只划一处)(1分) 乡 邻 之 生 日 蹙

15、阅读上面诗文的材料,选出赏析有误的一项(2分)(

) A“力尽不知热,但惜夏日长”这种近乎变态的矛盾心理写出了刈麦者的

痛苦生活与不幸遭遇。

B.“今我何功德,曾不事农桑”中“事农桑”指的是从事农业劳动。 C.“吏禄三百石,岁晏有余粮”写出了一个古代贪官的形象。

D.蒋氏自陈心曲的核心内容是“吾斯役之不幸,未若复吾赋不幸之甚也”。

16、比较《捕蛇者说》与《观刈麦》在写作手法或内容上的相同之处。(2分)

(四)阅读下面的文字,完成17—20题(10分) 海洋垃圾

①日本“3•11”9级大地震引发强烈海啸,导致大量的房屋、汽车和各种残骸卷入太平洋,形成了一个长约111公里的“垃圾岛”。研究人员估计,这个漂浮在海上的“垃圾岛”两年内会漂至夏威夷,3年后漂到美国西岸。

②其实,在日本地震和海啸导致大量垃圾卷入海洋之前,人们生活的这个星球中的海洋上就已经漂浮着大量的海上垃圾。2007年,美国科学家发现,太平洋上漂浮着一个巨大的“太平洋垃圾岛”,其面积有两个得克萨斯州那么大。虽然后来有人认为其面积被过分夸大了,但其存在却是不争的事实。

③海洋垃圾不仅影响海洋景观,还可能威胁航行安全。但更可怕的是,会对海洋生态系统的健康产生致命的影响,进而对海洋经济产生负面效应。海洋垃圾已引起全球的高度重视,新加坡等发达国家正采取措施,尝试处理这一问题。

④由于海上垃圾大部分是塑料,所以处理海上垃圾的首选办法是焚烧发电。不过海洋垃圾的焚烧发电有几个问题需要解决。除了打捞、运输、搁置脱水外,还会遇到二恶英的排放问题。由于塑料焚烧可产生大量的二恶英,会危及环境和生态,需要有特别能控制二恶英排放的高级焚化炉。

⑤海洋垃圾的另一个处理办法是建造人工岛屿。1998年,新加坡政府在两个离岸的小岛实马高和西康之间建造了1公里长的岩石长堤,并分出了11个相互连接的海湾单元;将单元里的海水抽干,排放好一层厚厚的塑料膜;然后将垃圾灰烬倾倒在这些单元里进行密封,以防止泄漏。至于垃圾中那些不能燃烧和回收的材料,比如石棉,也被塑料密封并掩埋在泥土中。此后,每个月都要对单元周围的海水取样检测,到现在为止,还没有发现任何单元有泄漏和污染海水的情况。每当一个单元的垃圾填到二三米高时,就进行铺沙种草,接着继续埋置垃圾。如此反复,垃圾最高可埋置到30米。最后在上面栽种植物,不再堆放垃圾。 (选自2011年4月20日《北京日报》,有删改。作者张田勘) 【相关链接】

材料一

目前,我国垃圾堆存量已达60亿吨,占用耕地5亿平方米。全国660个主要城市中,有200个城市陷入垃圾包围之中。以城市人口6亿为例,如每人每年产生440公斤垃圾,年产生垃圾量为2.64亿吨。

材料二

英国的垃圾填埋率为90%,意大利为74%,美国为67%,法国为45%,德国为46%。瑞士的垃圾焚烧率为74%,日本为72%,丹麦为70%。美国的废纸利用率为60%,铁罐头盒回收率为25%,玻璃回收率为20%。

17、文章第①节,从日本大地震产生的垃圾岛写起有何用意?(2分)

19、结合上下文,分别指出下列句子中加点部分的作用。(4分) (1)到现在为止,还没有发现任何单元有泄漏和污染海水的情况。

(2)太平洋上漂浮着一个巨大的“太平洋垃圾岛”,其面积有两个得克萨斯州那么大。

19、请用简洁的语言说明第⑤节中新加坡建造人工岛屿的具体步骤。(限30字以内)(2分)

20、根据文章内容和链接材料,就垃圾问题进行探究,写出你的两点发现。(2分)

(五)阅读下列文字,完成21—24小题(7分)

做一个终身读者

周国平

①读者是一个美好的身份。

②在很大程度上,人类精神文明的成果是以书籍的形式保存的,而读书就是享用这些成果并把它们据为已有的过程。做一个读者,就是加入到人类精神文明的传统中去,做一个文明人。历史上有许多伟大的人物.在他们众所周知的声誉背后,往往有一个人所不知的身份,便是终身读者,即一辈子爱读书的人。在某种意义上,一个民族的精神素质也取决于人口中高趣味读者的比例。

③然而,一个人并不是随便读点什么就可以称作读者的。在我看来,一个真正的读者应该具备以下特征——

④第一,养成了读书的癖好。

⑤也就是说,读书成了生活的必需,真正感到不可缺少,几天不读书就寝食不安,自惭形秽。如果你必须强迫自己才能读几页书,你就还不能算是一个真正的读者。当然,这种情形决非刻意为之,而是自然而然的,是品尝到了阅读的快乐之后的必然结果。事实上,每个人天性中都蕴涵着好奇心和求知欲,因而都有可能依靠自己去发现和领略阅读的快乐。

⑥第二,

⑦世上书籍如汪洋大海,再热衷的书迷也不可能穷尽,只能尝其一瓢,区别在于尝哪一瓢。读书是一件非常私人的事情,喜欢读什么书,不论范围是宽是窄,都应该有自己的选择,体现自己的个性和兴趣。其实,形成自己的阅读趣味与养成读书癖好是不可分的,正因为找到了和预感到了书中知己,才会锲而不舍,欲罢不能。没有自己的趣味,仅凭道听途说东瞧瞧,西翻翻,连兴趣也谈不上,遑论癖好。针对当今图书市场的现状,我要特别强调,千万不要追随媒体的宣传只读一些畅销书和时尚书,倘若那样,你绝对成不了真正的读者,永远只是文化市场上的消费大众而已。须知时尚和文明完全是两回事,一个受时尚支配的人仅仅生活在事物的表面,貌似前卫,本质上却是一个野蛮人,唯有扎根于人类精神文明土壤中的人才是真正的文明人。

⑧第三,有较高的读书品位。

⑨一个真正的读者具备基本的判断力和鉴赏力,仿佛拥有一种内在的嗅觉,能够嗅出一本书的优劣,本能地拒斥劣书,倾心好书。这种能力部分地来自阅读的经验,但更多地源自一个人灵魂的品质。当然,灵魂的品质是可以不断提高的,读好书也是提高的途径,二者之间有一种良性循环的关系。重要的是一开始就给自己确立一个标准,每读一本书,一定要在精神上有收获,能够进一步开启你的心智。只要坚持这个标准,灵魂的品质和对书的判断力就自然会同步得到提高。一旦你的灵魂足够丰富和深刻,你就会发现,你已经上升到了一种高度,不再能容忍那些贫乏和浅薄的书了。

⑩能否成为一个真正的读者,青少年时期是关键。经验证明,一个人在这个时期倘若没有养成读好书的习惯,以后再要培养就比较难了,倘若养成了,则必定终身受用。青少年对未来有种种美好的理想,我对你们的祝愿是,在你们的人生蓝图中千万不要遗漏了这一种理想,就是立志做一个真正的读者,一个终身读者。

(选文略有改动)

21、本文的中心论点是_______________________________________________;本文的主要论证方法是____________。(2分)

22、作者认为,一个真正的读者应该具备以下特征:第一,养成了读书的嗜好;第二,_____________________________________________;第三,有较高的读书品位。(1分)

23、阅读第②段画线句,请你用一个事例论据来支持作者这一论断。(2分)

24、第⑦段中加点的“尝其一瓢”是比喻说法,在文中具体指_______________

;作者把图书市场上某些人称为“野蛮人”,这些人的具体表现是_______

。(用原文中的话回答)(2分)

三、实践

写作(50分)

成长路上,有彩虹,也有风雨。成功时,留一点清醒给 自己,你会发现,更多的理想需要追求;失败时,留一点梦想给自己,你会发现,前方的道路何其宽广;幸福时,留一点责任给自己,你会发现,帮助别人多么快乐;忧伤时,留一点微笑给自己,你会发现,平凡的生活充满阳光„„ 请以“留一点

给自己”为题,写一篇文章。 要求:

(1)将标题补充完整;

(2)文体不限,字数不少于600;

(3)文中不得出现真实的人名、地名、校名。 题目:

祝贺你顺利完成答卷,请认真检查!

九年级语文调研试题

参考答案

1、shè

suǐ

连改联

镖改镳

2、略

3、名著阅读

《骆驼祥子》、

三、

三、祥子与车、虎妞 4、①《论语》作为中华民族重要的精神文化遗产,从古至今都产生过巨大的影响,甚至遍及世界各地。②当然需要孔孟之道。在这个拜金潮日益涌流的社会中,人们在向钱看的同时,也应注重汲取精神的食量。我们需要从孔孟之道中找寻我们的道德感,责任感,增强我们的是非观念。

5、⑪、、戒 诫 ⑫、事半功倍

事倍功半⑬、②积累报纸资料,就要发扬这种“点滴勤”的精神。(每题各1分,指出错误而改不正确均不得分)

6.答案:表达了诗人终于见到春色的惊讶、欣喜之情。

评分:共2分。意思对即可。

7.答案:运用拟人手法,赋予白雪以人的灵性,把初春的景象写得富有情趣。

评分:答对拟人手法得1分,分析正确得1分。共2分。意思对即可。 8、(1)觉得奇怪 (2)类 (3)拉,带领 (4)接近

9、C

10、(1)源中人看见了渔人,竟然大吃一惊,就问他从哪里来。(2)(在这儿)能享用的只有柴米鱼肉,这些一点也不缺少。11、(1)行踪

空间

人物对话

(2)第一问:没有剥削、没有压迫,和平安定、自给自足的原始农耕生活。第二问:不真实。这样的生活,只是表达了人民渴望过上平等、安宁、幸福的美好生活的愿望,在当时的黑暗社会中只是一种幻想,是不可能实现的。

12、(1)轻松的样子。(2)比。(3)因此。(4)只。13、(1)你将要同情我,使我活下去吗?(2)谁知道苛捐杂税搜括百姓的毒害有比这蛇毒更厉害的呢?

14、乡 邻 之 生 /日 蹙 15、C 25 16、将自己和蒋氏的生活进行对比,突出封建统治阶级对劳动人民的剥削和压迫,表现作者对劳动人民的同情。《观刈麦》把“贫妇人”的悲惨生活和“我”“曾不事农桑”“岁晏有余粮”进行对比。表现了作者对劳动人民的同情。思想内容上的相同之处是:都表现了作者对劳动人民悲惨生活的同情。

17.引出海洋垃圾的话题,激发阅读兴趣。

18.(1)“到目前为止”,限制了时间,准确地说明了没有发现任何单元有泄漏和污染海水的现象只是现在的运行状况,并不表示以后一定不会发生。

(2)(运用了作比较的说明方法),具体地介绍了“太平洋垃圾岛”面积之大,海上漂浮的垃圾之多,使读者对“太平洋垃圾岛”的印象更直观、更清晰。

19。建长堤,、分单元;抽干海水,排放塑料膜;倾倒并密封垃圾;(定期检测);反复地铺沙种草掩埋垃圾;栽种植物。

20参考示例:(1)我国城市面临严重的生活垃圾问题,加强环保教育刻不容缓;(2)海洋面临着人类生活垃圾危险;(3)发达国家在处理垃圾问题方面有各自成熟的技术。 ?21.我们要立志做一个真正的读者,一个终身读者;道理论证。 22.形成自己的阅读趣味

23.所写必须是历史上伟大的人物,必须是做为终身读者的身份。 24.有选择地阅读;追随媒体的宣传只读一些畅销书和时尚书

推荐第3篇:上海市静安区学年九年级语文第一学期期末教学质量调研试卷

上海市静安区2012-2013学年九年级语文第一学期期末教学质量调研试卷

一、基础积累

1.下列选项中加点的字注音全对的一项是(

) A.陆蠡(lǐ)

涸(hé)辙

揠( yàn )苗助长

淅(xī )沥

B.猗( yī )郁

抑(yì)郁

蕈(xùn )菌

婆娑(suō) C.瞥(piē)见

茎(jǐng)叶

纤(xiān)细

脉络(luò) D.移徙(xǐ)

葱茏(lóng)

囚系(jì)

嵌(qiàn)着 2.下列句子中有两个错别字的一项是(

) A.我快活地坐在我的窗前。度过了一个月,两个月,我留恋于这片绿色。我开始了解度越沙漠者望见绿洲的欢喜。

B.我占据着高广不过一丈的小房间,砖铺的潮湿的地面,纸糊的墙壁和天花板,两扇木格子嵌玻璃的窗。

C.在七月中旬,不能再留恋于烽烟四逼中的旧都,火车已经断了数天,我每日需得留心开车的消息。

D.圆窗外面长着常春藤。当太阳照过它繁密的枝叶,透到我房里来的时候,便有一片绿影。 3.依次填入下文标号处的标点符号正确的一组是

) 如果你走近细看,就会看出战士们的苦心①他们是用手电筒灯泡涂了红漆,做成小白兔的眼睛②把瓶口切下来,镶上花瓷碗片,做成了蝴蝶翅上的花点③就是在那漱口池里,也砌了红日、雄鸡和“早晨好”的祝词④正象战士诗里说的“园地道路作锦绸,摆花好似坐绣楼⑤这里的一花一叶,都渗透着战士们的汗水和深情。”

① ② ③ ④ ⑤ A 。 ; ; : ”, B : ; ; 。 。” C : 。 。 : 。” D : ; ; 。 ”。

4.依次填入横线处的词语,恰当的一项是(

①终身教育改变了学校教育的功能,拿了毕业证并不意味着学习____。 ②由于环境污染和一些人为的原因,阿尔巴斯白山羊绒的品质在____。 ③没有____过人生的苦辣,又怎能懂得长辈们创业的艰难呢? A.终止

蜕化

体验

B.中止

退化

体验 C.中止

蜕化

体味

D.终止

退化

体味 5.下列句子中加点成语使用正确的一项是(

)

A.他与这个有杀父之仇的敌人展开了一场不共戴天的斗争。 B.现在,我国打出自由撰稿人招牌的作家依然寥若星辰。 C.年逾不惑的二月河,创作了《康熙大帝》,终于在文坛崭露头角。 D.在学习中遇到关键问题不求甚解是要吃苦头的。 6.下列各句中意思明确,没有语病的一项是(

) A.周谷城先生早年投身“五四”运动,所以最终成为蜚声海内外的著名学者、历史学家。 B.我们一定要上好语文课,因为语文课是人们用以交流思想的不可缺少的工具。

C.金秋八月,丹桂飘香,美丽的北戴河海滨张开她热情的臂膀,迎来了宇航夏令营的少年朋友。 D.“春兰杯”第十届亚洲象棋锦标赛昨日在江苏泰州落幕,中国男队笑捧金杯,夺得十连冠。

7.下面的句子有两处前后脱节,请在相应的序号处添加必要的词语,使之完整连贯。①文人与民间创作结合,②中国小说呈现了重视情节的特点。③重视写情节并不意味着忽视写人物,④要通过情节表现人物,⑤以外在的情节动作来表现人物的内心活动和精神状态。 答:在

处补上

,在

处补上

。 8.在下面句中的空格上填入适当的词或短语,使全句更加生动形象。 我怀念□□□□□□□□□□小溪,□□□□□□□□□□鹅卵石,□□□□□□□□□□少女,□□□□□□□□□□鸭子。(每空不超过10字)

二、课内阅读

阅读《囚绿记》的节选部分,回答问题。

绿色是多宝贵的啊!它是生命,它是希望,它是慰安,它是快乐。我怀念着绿色把我的心等焦了。我欢喜看水白,我欢喜看草绿。我疲累于灰暗的都市的天空,和黄漠的平原,我怀念着绿色,如同涸辙的鱼盼等着雨水!我急不暇择的心情即使一枝之绿也视同至宝。当我在这小房中安顿下来,我移徙小台子到圆窗下,让我的面朝墙壁和小窗。门虽是常开着,可没人来打扰我,因为在这古城中我是孤独而陌生。但我并不感到孤独。我忘记了困倦的旅程和已往的许多不快的记忆。我望着这小圆洞,绿叶和我对语。我了解自然无声的语言,正如它了解我的语言一样。 我快活地坐在我的窗前。度过了一个月,两个月,我留恋于这片绿色.我开始了解渡越沙漠者望见绿洲的欢喜,我开始了解航海的冒险家望见海面飘来花草的茎叶的欢喜。人是在自然中生长的,绿是自然的颜色。

我天天望着窗口常春藤的生长。看它怎样伸开柔软的卷须,攀住一根缘引它的绳索,或一茎枯枝;看它怎样舒开折叠着的嫩叶,渐渐变青,渐渐变老。我细细观赏它纤细的脉络、嫩芽,我以揠苗助长的心情,巴不得它长得快,长得茂绿。下雨的时候,我爱它淅沥的声音,婆娑的摆舞。 1.用一句话概括这几段文字的主要内容。 2.“我了解自然无声的语言,正如它了解我的语言一样。”这句话表达了作者什么感情?从下文“囚绿”的内容来看,我真的了解绿吗? 3.“人是在自然中生长的,绿是自然的颜色。”这句话的言下之意是什么? 4.这几段文字中有许多表现作者心理感受的句子,选择一处进行赏析。

5.结合全文内容来看,作者赞美常春藤的绿与《绿》一文中朱自清赞美梅雨潭的绿所表达的情感有什么不同之处?

三、比较阅读

阅读《囚绿记》和《爬上窗台的绿色》,回答下列问题。 [甲文]

囚绿记(文略)

[乙文]

爬上窗台的绿色 ①窗台在二楼,那晌午里散着泥土芳香、夜晚嚼着寒露沁凉的绿色就顺着我家的门铃线义无反顾地爬了上来。不知不觉间,一线的绿色和点缀其间的串串白花便搭在了小院上空,并在窗台前探头探脑,映照出一户庭院的温馨与恬静。

②望着这线积极蓬勃的生命,真让人心情爽快并急于俯向贴面的绿色。也许,这正是绿色爬上窗台的惟一心愿吧。 ③确切地说,这束绿色来自一株黄瓜和一株莓豆。早在夏天它们就兄弟样并肩挽手攀援于小院南侧,在这个无风无雨的夏季艰辛地生长着,暑气和干旱丝毫没有抵消它们坚强向上的意志,它们的信念,永远是忠诚于脚下这片临时堆砌的菜园和我那朝夕呵护的妻子。除了寒冬,已经没有什么能让这群绿色在小院中消失,尤其在秋天,远离了燥夏的绿色反而越加显现出遮掩不住的郁葱,在院落、窗台无声地抖动着朗朗秋波。 ④这时爬上窗台的绿色犹如举向天空的思想和灵感,带着大地和秋天独有的体温,干脆利落地扑向你,让人觉得这绿色从一开始就是为了在这里等着你,让你怦然心动,让你热身奔涌。 ⑤仰望天空,仰望这线执著的绿色,总令我心旷神怡,思绪万端,无以言说的沟通感拓展着我也分解着我。它与我们一样,也向往自由,追逐光明,真心热爱脚下的一方水土和苦乐生活呀。有谁能说自然是冷漠的、无情的?江水奔流不息,倾诉的是自己澎湃的波涛;树木傲雪参天,挺拔的是自己无边的苍翠。而我一直相信在宇宙规律的作用下,世间万物都有生命有情感有思维,你无法评说谁更伟大谁更渺小谁更补益于对方。就像今晚,秋空如洗,夜凉如水,银河横波,一盘银月垂至天幕中央,说不出是欲升欲沉还是欲远欲近,最主要的是我无法忽略窗台前的这线绿色,它笼在迷人的夜色里,梯次排开的豆荚和瓜纽如同披上一身轻纱,轮廓分明地在秋风下回忆曾经汗流浃背的夏天的火热,算计着丰硕的收成,说不定一声秋虫的低吟浅唱就成全了豆荚们瓜纽们酝酿了整夏的爱情,刚刚坠入甜蜜的小生命于是急不可待地探出绿色的小手敲打窗子,要把一肚子的幸福与快乐说给你听。

⑥若是晌午呢,前提是阳光并不火辣,只是响脆脆地跌在这线绿色上,那些叶呀瓜呀豆呀的心情和爱情虽被一夜寒露湿透,却能在顷刻间还你会飞的欢笑与娇媚。就在不远处,建筑工地上不知歇息的轰鸣,也因了这绿色的宁静抚慰而富有节奏感。绿色,就这样以生的迫切、力量和宁静,同时给了我们视觉和听觉上的冲动愉悦。

⑦终于爬上窗台了,为实现这个简洁纯真的心愿,黄瓜和莓豆共同托举着绿色,克服了来自韭菜、芹菜、油菜等园内其它蔬菜的嫉妒嘲讽,选择惟一向上的道路来到我的窗前,让绿阴、生机、果实、清新悄然挤进房间的每一个角落,它相信有爱的地方就应该有绿色,有家的地方就该布满生机。这时候,绿色的内心便会感到不虚此行,并以芬芳的花蕾和沉实的种子重新构思着盎然的激情。

1.联系上下文,说说甲文第8段中“忽然有一种自私的念头触动了我。”和乙文中第7段中“为实现这个简洁纯真的心愿”中的“自私念头”和“简洁纯真的心愿”分别指什么? (1)自私念头:

(2)简洁纯真的心愿:

2.甲文第10段说“植物是多固执啊!”,其固执体现在

; 乙文第5段说“仰望这线执著的绿色”,其执著体现在

。 3.通读全文,说说两篇文章中的“绿色”分别具有什么样的精神品质。

甲文:

; 乙文:

4.甲文赞美的是绿色,文章的开头为什么要交待公寓的简陋?而乙文赞美的绿色来自黄瓜和莓豆,文章的结尾为什么又写“芹菜、韭菜、油菜”?

5.联系上下文,说说下列句中加点词语的表达效果。

(1)我好像发现了一种“生的欢喜”,超过了任何种的喜悦。

(2)那晌午里散着泥土芳香、夜晚嚼着寒露沁凉的绿色就顺着我家的门铃线义无反顾地爬了上来。

6.甲文第9段中运用了

的记叙顺序,其作用是

。 7.“绿色”一词在生活中还被赋予了很多新的内涵,如“绿色通道”中的“绿色”主要指安全、畅通不受任何阻碍。请你再举一例并说明它的其他新内涵。

例子:

内涵:

四、类文阅读 书房花木深 冯骥才

一天忽发奇想,用一堆木头在阳台上搭一座木屋,还将剩余的板子钉了几只方形的木桶,盛满泥土,栽上植物,分别放在房间四角。鲜花罕有,绿叶为多。 渐渐的这间搭在阳台上的木屋成了花房。但得不到我的照顾。我只是在想起给那些植物浇水才提着水壶进去,没时间修葺与收拾。房内四处的花草便自由自在、毫无约束地疯长起来。从云南带回来的田七,张着耳朵大的碧绿的圆叶子,沿着墙面向上爬,像是“攀岩”;几棵年轻又旺足的绿萝已经蹿到房顶,一直钻进灯罩里;最具生气的是窗台那些泥槽里生出的野草,已经把窗子下边一半遮住,上边一半又被蒲扇状的葵叶黑糊糊地捂住。由窗外射入的日光便给这些浓密的枝叶撕成一束束,静静地斜在屋子当中。一天,两只小麻雀误以为这里是一片天然的树丛,从敞着的窗子唧唧喳喳地飞了进来。我怕惊吓它们,不走进去,它们居然在里边快乐地鸣唱起来了。

一下子,我感受到大自然野性的气质,并感受到大自然的本性乃是绝对的自由自在。我便顺从这个逻辑,只给它们浇水,甚至还浇点营养液,却从不人为地改变它们。于是它们开始创造奇迹——

首先是那些长长的枝蔓在屋子上端织成一道绿盈盈的幔帐。常春藤像长长的瀑布直垂地面,然后在地上愈堆愈高。绿萝是最淘皮的,它在上上下下胡乱“行走”——从桌子后边钻下去,从藤椅靠背的缝隙中伸出鲜亮的芽儿来。几乎每次我走进这房间,都会惊奇地发现一个画面:一些凋落的粉红色的花瓣落满一座木佛身上;几片黄叶盖住桌上打开的书;一次,我把水杯忘在竹几上,一枝新生的绿蔓从杯柄中穿过,好似一弯娇嫩的手臂挽起我的水杯。于是,在我写作过于劳顿之时,或在画案上挥霍一通水墨之后,便会推开这房间的门儿,撩开密叶纠结的垂幔,独坐其间,让这种自在又松弛的美,平息一下写作时心灵中涌动的风暴。 以前我一直认为写作是一种忘我的想像,只有离开写作才能回到现实来。这间小屋却告诉我,我的写作往往十分尖刻地切入现实,放下笔坐在这里享受到的反倒是一种理想。 我被它折服了。并把这种感受告诉一位朋友。朋友说:“何必把现实与理想分得太清楚。你们不满现实是因为你们太理想主义。你们的问题是总用理想要求现实,因此你们常常被现实击倒在地。这因为你们天生是一群理想主义者。”

于是当我坐在这间花木簇拥的木屋中,心里常常会蹦出这么一句话: 我们是天生用理想来生活的人! (节选自2007年第12期《读者》) 1.“我”本来是想把这间木屋作为新辟的书房,但坐在那里却写不出东西,你帮作者解释一下原因。

2.文章语言生动形象,富有表现力,试举例进行具体说明。 3.结合文章内容,说说“我们是天生用理想来生活的人!”的含义。 4.作者对待花草的态度是顺其自然,由此得到的收获是什么?陆蠡 “囚绿”的结果是什么?

五、中考链接(2009年河北省) 昙花终于怒放 ①爱昙花。

②三年前的春天,朋友亲自给我送来一株昙花亩,种在白瓷壶里,枝叶铁青如剑,精心系上的红丝带临风飘拂。我高兴极了! ③不久,我便依照朋友的嘱咐,把这株昙花移植到花盆里,天天浇水,日日松土,不时除草,但由于阳光不足,长得很慢。我有点心急了,但急有何用?一天一天过去,一年一年到来,一直未见她结蕾开花。这株昙花长势还算正常,从原来的一枝发成三枝,叶子变得厚实起来,三年时间差不多长高三倍。时间久了,心情也就淡然了,花开不开任由她去吧。心想,她终是一株昙花在生长着,只要她能延续生命,也就可以了。 ④今年中秋佳节将临的前几天,我突然发现最长的那枝已冒出小蕾来,每隔一天就大了许多,如小型的佛手.农历八月十三日,我来到花架边观察,只见嫩绿的花苞低首含羞,微启小口。我立即预感到,今夜昙花终于要怒放了。大约到晚上九时,她便开始悄然开放。原先柔软下垂的筒状萼部,徐徐向上翘升,作白天鹅抬头状。随之花瓣缓缓展开,如玉女提裙起舞,清香阵阵,优美绝伦。啊,她就是“月下美人”啊! ⑤过去未见昙花开放,感到非常神秘,真想一睹其仙姿芳颜。以前看到的都只是画家笔下的清影和摄影家镜头下的清艳,见不到她的动态美姿,也闻不出她的清香。今夜如愿以偿,真正见到鲜活美丽的昙花怒放,真是欢喜欲狂! ⑥这朵昙花,花冠硕大,高雅洁丽,雍容华贵,香气清新诱人。皓月当空,一家人在花架边,尽情欣赏“月下美人”,乃是今秋最美好的享受。

⑦种昙花三年,不开属自然,开了亦属自然,何必计较时间的迟早?里尔克说:“有何胜利可言,挺立就意味着一切。”此乃真理名言。她一年不开,两年不开,第三年终于怒放了。世上一切美好的事物,只要能坚持,锲而不舍,就能开花结果。

⑧罢花的品性令人感佩:她不急功近利,不争宠于群芳,只求一瞬的辉煌就已满足.昙花的生命力可谓强矣!她随处能够生存,叶可青万年,花可放万年! ⑨我爱昙花。

(选文有改动) 链接材料:

昙花,常绿灌木,主枝圆筒状,分枝扁平呈叶状,绿色,没有叶片,花大,白色,在分枝边缘上,多在夜间开放,开花的时间极短。供观赏。原产于墨西哥。

1.在下面方框内,填写表明作者心情随昙花的生长而发生变化的词语。(3分) 高兴极了

—> —>

—>

2.简要分析选文第④段画线句子的表达效果。(3分) 答:

3.选文第⑧段和链接材料运用的表达方式有什么不同? 答:

4.选文第⑨段在全文内容与结构上的作用分别是什么? 答:

5.作者从昙花终于怒放的过程得出的人生哲理是什么? 答:

参考答案

一、基础积累

1.B(A揠yà苗助长 C茎jīng叶 D囚系xì)

2.C(留连、须得 )

3.D

4.D(中止:中途停止。终止:停止、结束。第一句结合前面的“终身教育”,应填“终止”。体验:用实践来认识周围的事物,其中有“经验”的意思。体味:仔细体会,其中含有“品味”的意思。第三句横线后面有“人生的苦辣”,应当用“体味”才能搭配。蜕化:蛇虫类脱皮,引申为腐化堕落。退化:引申为事物由优就劣,由好就坏) 5.D(A“不共戴天”应作“你死我活”。 B“寥若星辰”应作“寥若晨星”C“崭露头角”多指青少年)

6.D(A项无因果关系,B项搭配不当,C项“金秋八月”与“夏令营”矛盾) 7.②处补上“使得”;④处补上“而是”。

8.例:故乡后山流下的清澈的/溪水中彩色的/到溪畔洗衣和汲水的/在水面追逐欢笑的。

二、课内阅读

1.写“我”享受房间的绿色。

2.这句话表达了作者对绿的热爱,对这样一格安宁、平和环境的追求。从下文“囚绿”的内容来看,我并不能真的了解绿。 3.人是不能脱离自然环境的,而绿就是自然环境最好的代表,人不能生活在缺少绿的环境之中。 4.示例:“我疲累于灰暗的都市的天空,和黄漠的平原,我怀念着绿色,如同涸辙的鱼盼等着雨水!”采用比喻的修辞手法,抒发了对绿的强烈的热爱。 5.《囚绿记》中作者借常春藤的精神,抒发自己忠于祖国的爱国情怀;《绿》赞美梅雨潭的绿是为了表现作者对新生活的希望和积极向上的思想感情。

三、比较阅读 1.(1)指我囚住这绿色如同幽囚一只小鸟,要它为我作无声的歌唱。(2)爬上窗台。 2.它的尖端总朝着窗外的方向。

甚至于一枚细叶,一茎卷须,都朝原来的方向。

3.甲文:不屈服,对光明与自由的追求与向往;

乙文:积极蓬勃,执著追求,无私奉献。 4.甲文交待公寓的简陋是为了衬托作者对绿喜爱到极致,为后文囚绿作铺垫。 乙文写芹菜、韭菜、油菜是反衬其积极向上的精神。 5.(1)“生的欢喜”是指对于生命成长的喜悦,表达了作者对充满生机的绿的喜爱,对充满活力的生活的向往。 (2)“义无反顾”是指绿色向上爬的坚决与坚定,表现了绿色的执著、积极向上的品格。

6.插叙

补充交待了“我”对“绿”的喜爱是由来以久的,表达了对充满活力的生活的向往。

7.示例:绿色食品

无污染的安全、优质、营养类食品

四、类文阅读

1.因为作者到了那里就觉得是一种享受,被大自然的美所陶醉,因此无心写作。

2.动词的运用,如“蹿”字写出了绿萝旺盛的生命力。拟人化比喻的修辞手法,如 “一枝新生的绿蔓从杯柄中穿过,好似一弯娇嫩的手臂挽起我的水杯。”表现了“我”的喜爱之情。 3.提示:根据朋友的话来回答。意思接近即可。

4.收获是“我感受到大自然野性的气质,并感受到大自然的本性乃是绝对的自由自在。” 陆蠡出于对绿的喜爱,想把它们永远留在自己身边,所以才“囚绿”,但结果是“绿”总是想出去,而且长势越来越糟糕。

五、中考链接

1.有点心急

心情淡然

欢喜欲狂

2.运用了拟人的修辞方法,描写了昙花含苞待放的情态,将昙花人格化,更加生动形象。

3.第⑧段运用了议论、抒情的表达方式:链接材料运用了说明的表达方式。

4.进一步抒发了喜爱昙花的感情;首尾呼应,使文章结构严谨。

5.世上一切美好的事物,只要能坚持,锲而不舍,就能开花结果。

推荐第4篇:九年级第一学期期中教学质量评估

第一学期期中教学质量评估

九年级化学

1.2010上海世博会开幕式上的精彩表演中,发生化学变化的是

A.焰火表演B.霓虹灯表演C.音乐喷泉D.气球升空

2.下列气体中会污染空气的是

A.氮气(N2)B.氧气(O2)C.水蒸气(H2O)D.二氧化硫(SO2)

3.在化学变化中下列哪一种微粒不能再分

A. CO2B.HC. HClD.H

24. 下列各组物质中,前者是氧化物,后者是混合物的是

A.胆矾纯碱B.液氧碘酒

C.冰水共存物洁净的空气D.盐酸加碘食盐

5.集气瓶中装满某气体,可能是下列气体中的某一种:①二氧化碳②氧气③空气④氮气。将燃着的木条伸入瓶中,火焰立即熄灭,则该瓶气体可能是

A.①或②B.②或③C.①或④D.③或④

6. 下列有关化学用语的表述正确的是

A.2N2表示4个氮原子B.氩元素符号:ar

C.MgCl2的名称:氯化镁D.碳酸根的化学式:CO32-

7.科学家用单个分子制成了“纳米车”,它能在人工操纵下运输药物分子到病源处释放以杀死癌细胞。下列有关“纳米车”的说法正确的是

A.“纳米车”的研制成功说明分子要在人为外力作用下才能运动

B.“纳米车”分子不是由原子构成的

C.“纳米车”运输药物分子的过程是肉眼可以直接看见的

D.“纳米车”研制成功说明在一定条件下人可以控制分子运动

8.下列实验的错误操作与“可能产生的后果”不一致的是 ...

A.标签受损B.受热仪器破裂C.溶液蒸不干D.读数不准确

9.已知氮原子核内有7个质子。科学家已研究出高能微粒N5+,则有关说法正确的是

A.N5+是由5个氮原子构成的单质B.N5+中只有质子,没有中子和电子

C.每个N5+中含有35个质子和34个电子D.N5+中含有35个质子和35个电子

10.右图表示封闭在某容器中的少量液态水的微观示意图(该容器的活塞可以

左右移动)。煮沸后,液态水变成水蒸气。在这一过程中,发生的变化是

大 气

B.水分子之间的间隔变大 A.水分子本身变大

大LED大11.

是光源材料,Ga为+3价,As的化合

气 气

AC.水分子的数目增多D.水分子受热都跑到容器的一端 12.下列粒子中属于阴离子的是

13.薄荷醇(化学式为C10H20O)可用于糖果、饮料的香。下列有关薄荷醇的说法正确的是A.薄荷醇不属于化合物

B.薄荷醇由10个碳原子、20个氢原子和1个氧原子构成

C.薄荷醇中碳元素、氢元素和氧元素的质量比为10:20:1D.薄荷醇中碳元素的质量分数约为76.9%

14.右下图是实验室加热高锰酸钾制取氧气的装置图,下列有关实验操作的分析错误的是 ..

A.气密性检查:用手握紧试管,观察到水中导气管有气泡冒出,说明装置不漏气

B.试管口略向下倾斜:防止试管壁上的水倒流入试管底部,导致试管炸裂C.加热:直接用酒精灯的火焰对准药品所在位置加热D.停止加热:先把导管移出水面,再熄灭酒精灯

15.16.与反应时间的关系坐标图,其中合理的是

17.(7分)(1)用化学符号表示:

①2个氢原子:②3个铝离子:

(2)构成物质的微粒有:A.原子;B.分子;C.离子。试判断构成下列物质的微粒,并用相应的序号填空。 ..①氧气:②铁:③氯化钠:(3)下列符号中数字“2”所表示的意义,将其序号填在相应的横线上:

①Ca②NO2③2NO④MgO⑤2PO43-

表示分子个数的是;表示1个离子所带电荷数的是。 18.(4分)下列分别盛有不同物质的容器中,所盛物质混合物的是,纯净物的是 属于单质的是,氧化物的是。(均填容器下的代号)

19.(4分)下面是钠元素和氯元素在元素周期表中的信息和3种粒子的结构示意图。请回答下列问题:

(1)氯原子的核电荷数是;钠原子与氯

原子的最本质区别是;

+

(2)在钠和氯气生成氯化钠的反应中,得到电子的是原子;Na的结构示意图是(填字母)。 20.(4分)2010年上海世博会的主题是“城市,让生活更美好”。

(1)在世博会中应用了下列技术,其中能有效实现“低碳节能环保”的是。

A.采用水循环为场馆降温B.用竹、藤等天然材料建造场馆C.构建生态绿化墙面

(2)由上汽集团自主研发的“叶子”概念车(如右图所示)在中国馆低碳区展出,该车顶部的“大叶子”是一部光电转化器,把能转化为电能。

(3)世博园区内有许多饮水台,可取水直接饮用。其中的饮用水处理步骤如下图所示;

21.(5分)某课外活动小组在学习了书本 “空气中氧气含量的测定”实验,知道P2O5 不能随便排放在空气中,否则会对空气造成污染,所以对该实验进行了改进:

在由两个注射器组成的密闭系统内共有50 mL空气,如右图。然后给装有红磷的玻璃管加热。同时交替推动两个注射器的活塞,至玻璃管内的红磷变成白烟,且较长时间内无进一步变化时停止。停止加热后,待冷却至室温将气体全部推至一个注射器内。 请根据你对上述实验的理解,回答下列问题:

(1)实验结束后,注射器内的气体体积理论上应该减少约mL。

(2)在实验的加热过程中,交替缓慢推动两个注射器的目的是。写出该反应的文字表达式。

(3)上述实验只是粗略测定空气中氧气含量的一种方法,你认为造成该实验不够精确的可能原因是 (写出其中一种)。

三、实验题(本题有2小题 ,共 20分)

2+

222.(12分)实验室中,利用下列装置可以制取某些气体,请回答下列问题。

ABCDE ⑴写出带标号的仪器的名称:①;②;

⑵向气体发生装置内加入药品前,必须; ⑶a实验室若用高锰酸钾制取氧气,选择的制取装置是;

b实验室若用双氧水制氧气的反应文字表达式为, 反应中,MnO2起作用,加快了H2O2的分解。

⑷乙炔(C2H2)又称电石气,是一种无色、无味、密度比空气略小,不溶于水的气体;工业上常用它燃烧产生的高温来切割和焊接金属;实验室用电石(固体)与水反应制取乙炔。你认为制取乙炔应选择的发生装置是;收集装置是;

(5)甲烷(CH4)在实验室里可用排水法或向下排空气法收集,请你根据甲烷的收集方法,推测该气体具有....的性质是_________________________。

23.( 8分)做完“铁在氧气里燃烧”实验后,小冬同学有两个疑惑不解的问题,于是她进行了以下探究活动,请你一同参与。

[问题1]铁燃烧时溅落下来的黑色物质中还有没有铁呢? [查阅资料1] (1)自然界中铁的氧化物主要是Fe3O4(黑色)和Fe2O3(红褐色)两种[FeO(黑色)极易被氧化为Fe2O3]; (2)铁的氧化物均能溶于盐酸或硫酸的溶液。

(3)铁单质也能与盐酸或硫酸的溶液反应,生成氢气。 [实验探究]

将冷却后的黑色物质碾碎,装入试管,加入_____溶液,观察到的现象是_____,说明铁燃烧时溅落下来的黑色物质中还含有铁。[问题2]铁燃烧的产物为什么不是Fe2O3呢?[查阅资料2]

(1)Fe3O4和Fe2O3的分解温度、铁的熔点见表:

(2)Fe2O3高温时分解成Fe3O4。

[理论探讨]

根据实验现象,并结合表中数据,可推知铁在氧气里燃烧时产生的高温应在_____之间,在此温度范围内Fe2O3已分解,所以铁在氧气里燃烧的产生是Fe3O4。 [拓展延伸]

(1)实验中为了防止集气瓶炸裂,必须_____;

(2)Fe2O3高温时会分解成Fe3O4和一种气体,请写出此反应的文字表达式; (3)有些超市的食品密封包装盒的透明盖内放有黑色的FeO粉末,若粉末的颜色____,就说明包装盒破损进气,这样售货员就能及时发现并处理。

四、计算题(本题 8分)

24.(8分)我国从1994年开始强制食盐加碘,即在食盐中加入碘酸钾(KIO3),以防治碘缺乏病的发生,今年将适当下调现行食盐加碘量。各地将根据本地区实际情况作相应调整。深圳已率先作出调整,由原来每千克食盐含碘40mg下调至25mg。

(1)碘酸钾中碘元素、钾元素、氧元素的质量比为;它受热易分解,烹饪时,添

加加碘食盐应注意。

(2)碘酸钾中碘元素的质量分数为(保留至0.1%或用分数表示)(3)请为我市某企业计算:售往深圳的加碘食盐每袋(500g)应比调整前少加碘酸钾多少毫克。

推荐第5篇:九年级语文上册第一单元测试卷

第一单元测试卷

(满分:120分 考试时间:120分钟)

一、积累与运用(共28分)

1.下列加点的字,每对读音都不同的一项是(A)(2分) A.折腰/折腾

今朝/改朝换代

分外/分道扬镳 B.嘶哑/撕咬

风骚/搔到痒处

妖娆/抓耳挠腮 C.冠冕/冠军

鲜妍/鲜为人知

奉还/以牙还牙 D.海峡/脸颊

汹涌/始作俑者

坟墓/朝三暮四

(解析:A.zhé/zhē,zhāo/cháo,fèn/fēn;B.sī/sī,sāo/sāo,ráo/náo;C.ɡuān/ɡuàn,xiān/xiǎn,huán/huán;D.xiá/jiá,yǒnɡ/yǒnɡ,mù/mù。)

2.下列词语中没有错别字的一项是(C)(2分) A.娉婷

遐想

汹涌澎拜

秦皇汉武 B.漂逸

怅惘

一泻万丈

风流人物 C.忧戚

呢喃

原驰蜡象

红装素裹 D.摇曳

腐烂

顿失涛涛

一代天骄

(解析:A.“拜”应写作“湃”;B.“漂”应写作“飘”;D.“涛涛”应写作“滔滔”。)

3.依次填入下面一段文字横线处的语句,衔接最恰当的一项是(B)(2分)

“南国之秋,当然是也有它的特异的地方的……可是色彩不浓,回味不永。比起北国的秋来,正像是黄酒之与白干,稀饭之与馍馍,鲈鱼之与大蟹,黄犬之与骆驼。”这就不但在情调上,而且在语言上把雅趣和俗趣统一起来了。________。________。________。________。________。

①但是,郁达夫却在这里构成了和谐的统一情调

②前面就有茅房、耗子,这里又有稀饭、馍馍、黄犬、大蟹,等等 ③这是因为他的情感特点,本身就是把大雅和大俗融为一体的

④这些话语本来就是缺乏诗意的,用在这充满古典的、高雅趣味的文章中,是要冒不和谐的风险的

⑤郁达夫把秋天写得这么有诗意,赋予它一系列诗意的高雅的话语,然而不时又穿插一些平民的俗语进去

A.②①⑤④③

B.⑤②④①③

C.②⑤①④③

D.③⑤②④① 4.古诗文默写。(8分)

(1)长风破浪会有时,直挂云帆济沧海。[李白《行路难》(其一)] (2)云横秦岭家何在?雪拥蓝关马不前。(韩愈《左迁至蓝关示侄孙湘》) (3)现在人们常用《送杜少府之任蜀州》中的名句“海内存知己,天涯若比邻”来表达对好友的深情厚谊。

(4)明月千里寄相思。“我寄愁心与明月,随风直到夜郎西”(《闻王昌龄左迁龙标遥有此寄》),明月送去了李白对远方朋友的担忧和牵挂;“但愿人长久,千里共婵娟”(《水调歌头》),明月寄托了苏轼对远方亲人的思念和祝福。 5.名著阅读。(任选一题作答)(4分)

(1)“打开它们的关闭了很久的窗子,让我把花束,把香气,把亮光、温暖和露水撒满你们心的空间。”这是艾青《太阳的话》中的诗句,请简要说说艾青《太阳的话》描绘了什么形象?寄托着诗人什么样的思想感情?

诗人描绘了太阳的形象(2分),寄托着诗人追求真理、向往光明的思想感情(2分)。 (2)酒壮英雄胆,酒成(误)英雄事。聚义梁山,“大块吃肉,大碗喝酒”,水浒英雄的豪情似乎总离不开一个“酒”字。请借助你的阅读经历,简述《水浒传》中一个关于“酒”的故事。

【示例一】酒壮英雄胆。“武松醉打蒋门神”。武松发配至孟州牢营,管营施忠之子施恩,慕其名,二人结拜。施恩的酒店被恶霸蒋门神霸占,武松闻之大怒,带酒边走边喝赶至快活林,痛打蒋门神,夺回酒店。

【示例二】酒误英雄事。“鲁智深喝酒误事”。林冲服役前,鲁智深答应林冲照顾他的娘子和家人,可因喝酒迟到一步,没能阻止悲剧发生。林娘子被高俅之子高衙内侮辱了,随后上吊而死。

6.请用简洁的语言概括下列材料的主要内容。(3分)

3月6日,河南省环保厅在郑州举办了全省环保系统“讲忠诚、守纪律、做标杆,持续打赢环境污染防治攻坚战”演讲活动,党组成员、副厅长王喜云,省直纪工委副书记张代波,省环保厅在郑全体人员,省辖市环境保护局分管局长,300多人现场聆听了演讲。

此次活动充分展现了全省环保系统对党忠诚、敬业奉献、严格自律、担当进取、勇于拼搏的良好形象,表达了坚决打赢污染防治攻坚战、持续改善环境质量的坚强决心和坚定信心。 最后,党组成员、副厅长王喜云也以演讲《立足本职忠诚担当——把信送给加西亚》的形式,总结了两年以来环保厅党组在省委省政府领导下持续深入开展环境污染防治攻坚战取得的成绩,并代表厅党组对2018年的环境污染防治攻坚工作提出要求,为进一步持续打赢2018年污染防治攻坚战、建设美丽河南提供坚强有力保证。

河南省环保厅开展“讲忠诚、守纪律、做标杆,持续打赢环境污染防治攻坚战”演讲活动。

7.阅读下面材料,按要求答题。(7分) 材料一:余光中《乡愁》

①小时候/乡愁是一枚小小的邮票/我在这头/母亲在那头 ②长大后/乡愁是一张窄窄的船票/我在这头/新娘在那头 ③后来啊/乡愁是一方矮矮的坟墓/我在外头/母亲在里头 ④而现在/乡愁是一湾浅浅的海峡/我在这头/大陆在那头

材料二:余光中诗中所显露出来的爱国情怀,不像岳飞“三十功名尘与土,八千里路云和月”那样豪放,不似李煜“小楼昨夜又东风,故国不堪回首月明中”这般凄婉,余光中诗中爱国与怀乡融为一体。余光中在其诗歌创作中,还充分表达了对中华传统文化的推崇。余光中说:“只要不放弃自己的文学传统,就可以有自己的文学生命,出现小岛上的文学大师。”

材料三:

(1)仿写材料一第①小节句式,以“童年”为主题,写一节小诗。(2分) 小时候/童年是一条清清的溪流/快乐在里头/烦恼在外头(一空1分) (2)从材料二中提取余光中诗歌内容的两条主要信息。(2分)

①诗中爱国与怀乡融为一体;②充分表达了对中华传统文化的推崇。(一点1分) (3)仔细观察材料三中的漫画,结合漫画内容具体介绍该漫画的寓意。(3分) 【示例】漫画左上角是带有“春节”二字的灯笼和喜庆的鞭炮(1分);主体部分是一个穿着西装的中国人和一个穿着中式服装的外国人用中国传统拱手礼拜年(1分),两个人的对话内容反映了中国文化的世界影响(1分)。(言之成理即可)

二、现代文阅读(共28分)

(一)阅读下文,完成8—11题。(共14分)

雪之梦

毛志成

①每年入冬后,我就格外盼雪,盼那纷纷扬扬、弥天漫地的飘雪,盼那厚厚实实、清清白白的积雪,盼那花花点点、扑朔迷离的残雪。

②中国缺雪已有经年,北京尤甚,好像那雪只飘落在人们的回忆中,消融在人们的遐想里。

③下雪了!窗幔上的微光告诉了我,街上的喊声告诉了我,孩子们跳下床、冲出门的脚步声告诉了我。

④我那颗一下子复活了稚气、复活了回忆的心,似乎顿时浸润在琼花中,净化在玉屑里。

⑤冲出房门,冲上街头,我不禁索然了。

⑥这算得上雪吗?灰蒙蒙的天懒懒散散地洒下几粒近乎尘埃、近乎细沙般的东西,扭扭捏捏地登上楼顶,娇娇滴滴地落在路面,似乎对路旁的枯树、两侧的民宅不屑一顾。它们写在路面上的,也只是一首闪烁其词的晦涩诗,貌似博大而实际浅薄。它们赶不上步履匆匆的行人脚步,更挂不上他们的头顶,染不上他们的眉梢,只在那涂着脂粉的脸上骚动了几下,在那施过铅黛的瘦眉上悬了几星儿。

⑦这哪里是我梦中的雪!

⑧我梦中的雪,纷纷扬扬,铺天盖地,创造着地球上最伟大的宏观美。漫宇琼瑶,满天寒凛,以世上第一流的平等气度博施于山,普赠于涧,广铺于野,慨惠于林。泼辣辣地洒下来,登华厦,覆寒宅,染眉头,醉心间。不弃枯木朽株,不漏病妪衰叟,不能把寒门少女的俭朴衣装染艳,但能把她们的双颊染红。

⑨而眼前的雪,是奢华而悭吝的雪,是徒有虚名的雪。淡淡的、薄薄的、灰灰的,远看有色而近观无形,经不住行人的步履,徒在万千足迹后面遗下了泥泞。好奇的中学生为了验证书上的话——“雪花都是六角形的”,伸出他们的手承接良久,手心里也只是积存了几滴冷露。

⑩雪停了,几乎用不着太阳的帮忙,只需几缕小风,楼顶、屋顶、街头、枝头上的薄粉顿消,路面上那一层薄薄的浊水化而为冰,像是推出了一张古板的脸。

⑪这哪里是我记忆中的雪后!

⑫我记忆中的雪后,是壮丽冬景的最佳镜头,是一幅圣洁绘画的定稿。纯洁、晶莹、清寒的美学元素在大地上铺下了旷远的情怀,铺下了博大胸襟,铺下了冷凝的火焰。步履声声,韵律浑朴,深深浅浅的足迹伸向高山,伸向田野,伸向一切历史车轮在转动的地方。

⑬即使春天的帷幕已经拉开,造物主已经着手打扫冬景的遗迹,舞台上出现的也不是一片空白,而是人们那即将储存在心中的深情回忆。旷野中的雪已经花花点点、斑驳陆离了,但松枝上还有,远山上还有,大道的车迹里还有。远山大道上有雪迹,人的胸襟就会扩展;翠柏苍松上有雪迹,人的情思就会延伸。 ⑭梦中的雪,我多么愿意它变成雪中的梦。

⑮请造物主飘落一场真正有气魄的雪吧,借以成为我展开新梦的襁褓……

8.对全文来说,第①段有什么作用?(3分)

第①段为全文定下了感情基调(1分),表现了作者爱雪、盼雪的急切心情(2分)。 9.请从修辞手法的角度赏析文中画线的句子。(3分)

盼那纷纷扬扬、弥天漫地的飘雪,盼那厚厚实实、清清白白的积雪,盼那花花点点、扑朔迷离的残雪。

运用排比的修辞手法(1分),不仅写出了雪的特点,而且体现出“我”对雪的喜爱和强烈的盼望之情(2分)。

10.通读全文,用简洁的语言说说眼前的雪和“我”梦中的雪各有什么特点。(4分)

【示例】眼前的雪:雪花只有几粒,近乎尘埃和细沙(1分);雪后的地面是淡淡的、薄薄的、灰灰的,地面的雪很快就融化了,让人扫兴(1分)。梦中的雪:纷纷扬扬,铺天盖地(1分);纯洁、晶莹、清寒,激发人的情思(1分)。

11.文章花了不少的篇幅写记忆中的雪和记忆中的雪后,这样写有必要吗?请分别从结构和内容两个方面谈谈你的看法。(4分)

有必要。结构上,写记忆中的雪和雪后,既是对以往那些大雪的追忆、钟情,又很自然地引出作者对当今气候变暖、空气污染的忧虑(2分)。内容上,将记忆中的雪和雪后的美景与现实中的缺雪进行对比,从而深化了文章主旨(2分)。 (二)阅读下文,完成12-15题。(共14分)

第一个告诉你下雪的人

①有没有人告诉你,下雪了?

②大片大片的雪花,像白色的羽毛从天空飘然落下。霎时间,山峦、田野、村庄、小路都陷入白色的忧郁。

③可是对于孩子,这白色永远是快乐的。那是一场罕见的大雪,一夜之间就把村庄包装成了童话世界。童话世界里怎么少得了我们这些可爱的小精灵呢!

④清晨起来,房门已经被大雪封住了,母亲正拿着一把小铁锹从门缝处一点一点把雪挖开,慢慢开辟出一条小路来。我和妹妹也陆续穿好了棉袄棉鞋,戴上了帽子手套,跑到院子里和母亲一起“扫雪”,那是最愉悦的劳动。与其说是在帮母亲,不如说是在给母亲添乱,母亲前脚刚扫出来的小道,我们打闹着,又把雪扬得到处都是。可是母亲并不恼,笑呵呵地看着我们,任凭我们在纯洁的雪里“胡作非为”。 ⑤树上、房顶、鸡舍、草棚,闪着亮晶晶的银光,在与红日的交相辉映中分外妖娆。

⑥妹妹欢快地跳着,和我一起堆雪人.我们用瓶盖做眼睛,用红辣椒做鼻子,而我总怕小雪人会冷,心疼地把自己脖子上的红围巾摘下来给它围上。一会儿,我们的新伙伴就伫立在庭院里,真是可爱极了!这时,母亲已经做好了早饭,开个门缝对着我们高喊:“吃饭啦……吃饭啦……”我和妹妹好像都没有听见,磨磨蹭蹭谁也不愿意回屋。广阔苍穹,母亲是见证这份快乐的唯一观众,望着她的两个白雪公主,母亲的微笑是甜的。

⑦洁白的雪花舞动着我童年的快乐,也翩飞着我少年的轻狂。初三那年寒假,我和同学们约好去县城的书店买些参考书,早上临出门时母亲就叮嘱我穿那件厚一点的棉衣,可是少女的我是爱美的,怎么能把自己变成臃肿的面包?没想到从县城回来时就下起了大雪,汽车晚点了两个多小时,当我背着书包又冷又饿走回家时,看见母亲像一座白色的雕像立在门口,睫毛上沾满了霜白,每眨一下眼睛似乎都能发出吧嗒吧嗒的声响。那天我吃饱喝足就有了精神,母亲却病倒了,高烧40℃。至今我都不知道母亲那天在家门口迎着风雪,望着我回来的方向站了多久。

⑧多年以后,我也成为一位母亲。我的孩子和我一样热爱这北方的大雪。 ⑨春节,我带着孩子回老家。当我们像雪花一样从远方匆匆赶来,哒哒的马蹄声溅起一地飞雪,朵朵如花。那一天,村庄如故,母亲却白发如雪。

⑩年迈的母亲依然站在自家的门口张望着、等待着。仿佛这几十年来,一直站在这里,从未离开过。母亲这几天一直念叨着:“要下大雪了,也不知道通不通车。”母亲是不看天气预报的,但每一次比天气预报还要准确。慢慢我才知道,原来是母亲的老寒腿,每次下雪前都会疼。

⑪母亲的一生几乎都是在劳动中度过的。夏天她忙着菜园子、庄稼地,冬闲时她忙着给人做衣服补贴家用。东北的冬天天寒地冻,上世纪七八十年代我们家住的是简陋的茅草房,虽然窗户外面钉上了塑料布,门上也安上了棉布帘,但墙壁上还是会结霜,四处透风。母亲长时间坐在靠南墙的缝纫机旁做活,两条腿不知不觉中就得了风湿病。母亲腿疼是从来不对我们说的,粗心的我们怎么会知道母亲的腿病这般严重!

⑫雪花落在村庄,落在四野,也落在我隐隐作痛的心里。

⑬这个世界因为有雪,风景如此美丽,空气如此清新,步伐变得轻快,灵魂得到了洗涤。雪,如母亲的爱,时刻安抚我这颗烦躁的心,让我活得简单快乐,就像一朵雪花吹开了另一朵雪花的春天。

⑭传说,第一个告诉你下雪的人是最爱你的人。我相信这个美丽的传说,因为母亲就是最好的例子。电话叮铃铃响了,是母亲在叮嘱我,要下雪了,记得多穿点儿衣服——母亲的老寒腿又在播送“天气预报”了。我喏喏地应着,却透过那场即将到来的雪,看到了母亲的疼痛。 ⑮如果可以,我多愿意让自己变成一贴温暖的膏药,敷在她所有疼痛的关节。 (选自《文苑·经典美文》2017年第2期,有删改)

12.本文写了母亲的哪几次“等待”?请简要概括。(3分)

①“我”与妹妹在外玩雪,母亲做好早饭,等待“我们”吃饭;②母亲在家门口迎着风雪等候迟归的“我”;③年迈的母亲张望、等待春节回家的“我”。(一点1分)

13.结合加点的词语,赏析第④段中画线的句子。(4分)

可是母亲并不恼,笑呵呵地看着我们,任凭我们在纯洁的雪里“胡作非为”。 “胡作非为”本是贬义词(1分),放在此处却生动地写出了“我”和妹妹的淘气以及我们自由自在、无拘无束嬉戏的快乐(2分),同时表现出母亲的温柔及对“我”和妹妹的宠爱(1分)。

14.有人说第⑪段不可或缺,也有人说纯属多余,你的看法如何?请说明理由。(3分)

【示例一】我认为第B11段不可或缺。理由:第B11段交代了母亲得风湿病的原因,表现了母亲的勤劳,丰富了母亲的形象,为下文进一步抒情做铺垫。 【示例二】我认为第B11段纯属多余。理由:第B11段主要表现母亲的勤劳,与全文表现母爱的主旨关系不大;且“雪”是贯穿全文的线索,第⑩段最后一句写“雪”,而第B12段紧承“雪”来写,删去B11段表达更顺畅。(看法1分,理由2分)

15.文中的“雪”内涵丰富,请联系全文谈谈你的理解。(4分)

①“雪”象征着母爱。母亲铲雪、雪中等“我”回家、第一个告诉“我”下雪了等都体现了母亲对“我”浓浓的爱。②“雪”象征着快乐。雪中打闹、雪中堆雪人都表现了童年的快乐,这是让作者念念不忘的。(一点2分)

三、古诗文阅读(共14分)

阅读下面两个语段,完成16-19题。(共10分)

(一)若夫淫雨霏霏,连月不开,阴风怒号,浊浪排空,日星隐曜,山岳潜形,商旅不行,樯倾楫摧,薄暮冥冥,虎啸猿啼。登斯楼也,则有去国怀乡,忧谗畏讥,满目萧然,感极而悲者矣。 至若春和景明,波澜不惊,上下天光,一碧万顷,沙鸥翔集,锦鳞游泳,岸芷汀兰,郁郁青青。而或长烟一空,皓月千里,浮光跃金,静影沉璧,渔歌互答,此乐何极!登斯楼也,则有心旷神怡,宠辱偕忘,把酒临风,其喜洋洋者矣。

嗟夫!予尝求古仁人之心,或异二者之为,何哉?不以物喜,不以己悲,居庙堂之高则忧其民,处江湖之远则忧其君。是进亦忧,退亦忧。然则何时而乐耶?其必曰“先天下之忧而忧,后天下之乐而乐”乎!噫!微斯人,吾谁与归?

(节选自《岳阳楼记》)

(二)普少习吏事,寡学术,及为相,太祖常劝以读书。晚年手不释卷,每归私第,阖①户启箧取书,读之竟日。及次日临政,处决如流。既薨②,家人发箧视之,则《论语》二十篇也。

普性深沉有岸谷,虽多忌克③,而能以天下事为己任。宋初,在相位者多龌龊循默,普刚毅果断,未有其比。尝奏荐某人为某官,太祖不用。普明日复奏其人,亦不用。明日,普又以其人奏,太祖怒,碎裂案牍掷地,普颜色不变,跪而拾之以归。他日补缀旧纸,复奏如初。太祖乃悟,卒用其人。

(节选自《宋史·赵普传》)

【注释】①阖:关上。②薨:去世。③忌克:对人嫉妒刻薄。

16.下列各组句子中,加点词语意思相同的一项是(B)(2分) A.一:一碧万顷/而或长烟一空 B.极:感极而悲者矣/此乐何极 C.辱:宠辱偕忘/祗辱于奴隶人之手 D.异:或异二者之为/属引凄异

(解析:A.一片/全;B.均为“极点、尽头”;C.屈辱/辱没;D.不同/怪异) 17.把语段(一)中画横线的句子翻译成现代汉语。(3分)

沙鸥翔集,锦鳞游泳,岸芷汀兰,郁郁青青。

沙洲上的白鸥时而飞翔,时而停歇,美丽的鱼游来游去,岸上与小洲上的花草茂盛青翠。

18.语段(一)中“古仁人之心”的内涵是从哪些方面阐述的?它的可贵之处是什么?(3分)

“不以物喜,不以己悲”是从博大的胸襟的角度来说的(1分);“居庙堂之高则忧其民,处江湖之远则忧其君”是从考虑国家大事的角度来说的(1分)。可贵之处是不以自己的利益为重,而以天下为己任(1分)。 19.根据语段(一)(二)的内容,结合具体内容分析范仲淹和赵普两人的形象特征的相同之处。(2分)

这两个人都具有心怀天下、坚持原则的性格特征。(2分)

附语段(二)参考译文:

赵普年轻时熟悉官吏(应处理)的事务,学问不多,等到做了宰相,宋太祖常劝说他读书。赵普晚年读书勤奋,每次(退朝后)回到自己的住宅,关上门打开书箱拿出书,读一整天书。等到第二天处理政务,处理决断很快。他死后,家里的人打开书箱看到里面的书籍,原来是《论语》二十篇。

赵普性情沉着且为人严肃刚正,虽然对人嫉妒刻薄,但是他能够以天下大事作为自己的责任。宋朝初年,在宰相职位上的人,大多气量狭小,拘于小节,按常规办事,不多言语,赵普却刚毅果断,没有谁能和他比的。他曾经上奏推荐某人担任某个官职,太祖没有任用。赵普第二天又上奏请这个人(担任某官),太祖还是不用。第三天,赵普又把这个人(担任某官的事)上奏太祖,太祖发怒了,把奏章撕碎了扔在地上,赵普脸色不变,跪在地上把撕碎的奏章拾起来回到家。赵普把撕碎的旧纸片修补连接起来,又像当初一样上奏。太祖这才明白,终于任用了那个人。

阅读下面一首诗,完成20-21题。(共4分)

茅屋为秋风所破歌

杜 甫

八月秋高风怒号,卷我屋上三重茅。茅飞渡江洒江郊,高者挂罥长林梢,下者飘转沉塘坳。南村群童欺我老无力,忍能对面为盗贼。公然抱茅入竹去,唇焦口燥呼不得,归来倚杖自叹息。俄顷风定云墨色,秋天漠漠向昏黑。布衾多年冷似铁,娇儿恶卧踏里裂。床头屋漏无干处,雨脚如麻未断绝。自经丧乱少睡眠,长夜沾湿何由彻!安得广厦千万间,大庇天下寒士俱欢颜!风雨不动安如山。呜呼!何时眼前突兀见此屋,吾庐独破受冻死亦足!

20.诗歌画线部分是怎样描绘风雨肆虐的情景的?(2分)

“风怒号”从听觉上突出了秋风之狂,“卷”从视觉上形象地写出了秋风之肆虐(1分)。“飞”“渡”“洒”“飘转”等词写出了秋风对茅屋的破坏(1分)。 21.“自经丧乱少睡眠”表现了诗人怎样的情感?(2分) 诗人从眼前的处境联想到安史之乱以来的种种痛苦经历,从风雨飘摇中的茅屋扩展到战乱频繁、残破不堪的国家(1分),表达了作者关心民间疾苦、忧国忧民的思想感情(1分)。

四、作文(50分)

22.任选一题作文。(50分) (1)题目:________中的大自然

要求:①先在横线上填写合适的内容,把题目补充完整,然后作文。②除诗歌外,文体不限。600字左右。③文中不得出现真实的人名、校名、地名。 (2)读下面材料,然后作文。

大师指着远处的一座高山,问弟子:“那山为什么能与天相连?”“因为它高呗。”弟子说。大师带弟子来到海边,指着远处与天相连的海面问:“海在低处,没有山的高度,那为什么海也能与天相连、与天等高呢?”弟子支吾着,答不上来。“那是因为海的胸襟宽,它那宽阔的胸襟让它与天相连、与天等高。”大师说,“胸襟,也能成就一种高度,一种抵达天空的高度。”

读了上面的材料,你有什么联想或感悟?请任选角度,自定立意,自拟题目,写一篇文章。

要求:①除诗歌外,文体不限。600字左右。②文中不得出现真实的人名、校名、地名。 作文略

推荐第6篇:九年级语文下册第一单元测试卷

九年级语文下册第一单元测试卷

一、积累·运用

1.给加点字注音或根据拼音写汉字

①xiōng yǒng()②lóu yǐ()③腐烂() ④灰烬() ..

⑤péng hāo ()⑥qiáo cuì() ⑦荇藻() ⑧慰藉() ..

2.用“/ ”为下面的诗句划分节奏。

①为什么我的眼里常含泪水?因为我对这土地爱得深沉„„

②只有那辽远的一角依然完整,温暖,明朗,顽固而蓬勃生春。

③我是干瘪的稻穗,是失修的路基。

④我爱那野火冒起的轻烟,草原上过夜的大队车马。

3.填空。

①小时候,《乡愁》)

②这一角已变成灰烬,。(《我用残损的手掌》)

③我是你河边上破旧的老水车,;(《祖国啊,我亲爱的祖国》)

④我爱祖国,(《祖国》)

4.仿照画线的句子,发挥想象续写两句。

人在生命的旅途中,不能没有朋友的祝福。你的祝福如春天里的一缕清风,为我送来芬芳;如寒夜里的一团火焰,为我送来温暖。如,;如,。我将带着你的祝福,去搏击人生的风雨,拥抱绚丽的彩虹。

5.请指出下面三则材料出自什么作品,文字中的“他”是谁。

⑴他是一个“坚决、沉默而又有生气”的上等车夫,为了争取起码的生存权利,奋斗,挣扎,最终走上了一条自我毁灭的道路。

⑵他是梁山好汉,江湖人称“黑旋风”,挥一双板斧,勇猛无比。找按时,大闹东京城,扯了皇帝的诏书,杀了钦差„„

⑶他出身于英国中产阶级家庭,从小就向往冒险生活。他在一座无人的荒岛上过了长达28年的原始人生活。

⑴ 品:;“他”叫。

⑵ 品:;“他”叫。

⑶ 品:;“他”叫。

6.在“脚踏一方土”综合性学习活动中,同学们搜集到相关的材料。请你按照要求完

成下面两个问题。

(1)下面是我国有关“土地”的诗歌和故事,阅读后从文化层面说说土地的象征意义。

[材料一]假如我是一只鸟,

我也应该用嘶哑的喉咙歌唱:

这被暴风雨所打击着的土地,

这永远汹涌着我们的悲愤的河流,

这无止息地吹刮着的激怒的风,

和那来自林间的无比温柔的黎明„„

——然后我死了,

连羽毛也腐烂在土地里面。

为什么我的眼里常含泪水?

因为我对这土地爱得深沉„„

(艾青《我爱这土地》)

[材料二](重耳)去,过五鹿,饥而从野人乞食,野人盛土器中进之。重耳怒。赵衰曰:“土者,有土也,君其拜受之。”(《史记·晋世家》)

故事大意:晋公子重耳在逃亡途中,疲饿交加,向田间一位耕作的老农乞讨食物,老农捧起一把泥土递给他,一言不发。重耳大怒。随从赵衰说:“送给您土,是说您拥有土地的意思,您跪拜接受吧。”

[材料三]我们共产党人好比种子,人民好比土地。我们到了一个地方,就要和那里的人民结合起来,在人民中间生根开花。(毛泽东)

[材料一]土地象征

[材料二]土地象征

[材料三]土地象征

(2)探究下面材料,把你得出的结论写在横线上。

我国现在荒漠化土地263.6万平方千米,占国土总面积的27.50,主要分布在新疆、内蒙古、西藏、甘肃、青海、陕西、宁夏、河北等地,其荒漠化面积占全国荒漠化总面积的98.5%。在全国的荒漠化土地中,土壤类型以沙为主的有l73.97万平方千米。这就是我们通常所说的沙漠化面积或沙化面积,占我国国土总面积的l8.1%,或者全部荒漠化面积的66%。可见,。

二、诵读·赏析

请阅读下面的古诗,完成后面的问题。

丰乐亭游春(其一)

欧阳修

绿树交加山鸟啼,晴风荡漾落花飞。

鸟歌花舞太守醉,明日酒醒春已归。

【注释】①此诗作于庆历七年,欧阳修被贬为滁州知州的第三年。丰乐亭,在琅琊山幽谷泉上,距醉翁亭不远,为欧阳修所建。

7.诗歌的第

一、二句写了哪些景物?体现了丰乐亭景色怎样的特点?

8.这首诗表达了诗人怎样的思想感情?

三、理解·感悟

阅读下面的诗文,完成后面的问题。

(一)

[甲诗]乡愁四韵 余光中

给我一瓢长江水啊长江水,给我一片雪花白啊雪花白,

酒一样的长江水,信一样的雪花白,

醉酒的滋味,家信的等待, ①

是乡愁的滋味,是乡愁的等待,

给我一瓢长江水啊长江水。给我一片雪花白啊雪花白。

给我一张海棠红啊海棠红,给我一朵腊梅香啊腊梅香,

血一样的海棠红,母亲一样的腊梅香,

沸血的烧痛,母亲的芬芳,

是乡愁的烧痛,是乡土的芬芳,

给我一张海棠红啊海棠红。给我一朵腊梅香啊腊梅香。

[乙诗]乡色酒余光中

三十年前三十年后

你从柳树梢望我你是一杯乡色酒

我正年少我从椰树林望你

你圆你满

人也圆乡愁也满

9.《乡愁四韵》的四个形象有什么相同,有什么不同?

10.《乡色酒》中的“你”指什么?两幅画面有什么不同?

11.《乡愁四韵》与《乡色酒》结构上有什么不同?

12.《乡愁四韵》和《乡色酒》,你更喜欢哪一首?说说喜欢的理由。

(三)

月亮,在半梦半醒之间

迟子建

①太阳是不大懂得养生的,只要它出来,永远圆圆着脸,没心没肺地笑。月亮呢,它修行有道,该圆满时圆满着,该亏的时候则亏。它的圆满,总是由大亏小亏换来的。所以亏并不一定是坏事,它往往是为着灿烂时刻而养精蓄锐。

②故乡的夜晚,一本书,一杯自制的五味子果汁,就会带给我踏实的睡眠。可是到了月圆的日子,情况就大不一样。穿窗而过的月光,进了屋后,招呼也不打,仰面躺在我身旁空下来的那个位置。它躺得并不安分,跳动着,闪烁着,一会儿伸出手抚抚我的睫毛,将几缕月光送入我的眼底;一会儿又揉揉我的鼻子,将月华的芳菲再送进来。这调皮的月光,让我只能睡睡醒醒。

③月光和月光是不一样的。春天的月光,带着股嫩嫩的绿意;夏日的月光,饱满丰腴;秋天的月光,洗尽铅华,安详恬淡;冬天的月光,虽然薄而白,但落到雪地的月光新鲜明媚。

相比较,冬春之交的月光,就没什么特别动人之处了。雪将化未化,草将出未出,此时的月光,也给人犹疑之感,瑟瑟缩缩的。

④今天是满月的日子,回到自己的住处时,月亮已经升起来了。微醺的缘故,未及望月,就熄灯睡了。大约凌晨三点,我被渴醒了。床边的小书桌上,通常放着一杯白开水。室内似明非明,我起身取水杯的时候,发现杯壁上晃动着迎春枝条般的鹅黄光影。喝过这杯被月光点化过的水,无比畅快。

⑤回床的一瞬,我有意无意地望了一下窗外,立时被眼前的情景震住了:天哪,月亮怎么掉到树丛中了?我见过的明月,不是东升时蓬勃跳跃在山顶上的,就是夜半时高高吊在中天的,我还从没见过栖息在林中的月亮。【A】那团月亮也许因为走了一夜,被磨蚀得不那么明亮了,看上去毛茸茸的,更像一盏挂在树梢的灯。【B】那些还未发芽的树,原本一派萧瑟之气,可是掩在林间的月亮,把它们映照得流光溢彩,好像树木一夜之间回春了。

⑥看过了这样的月亮,我再回到床上时,又怎能不被美给惊着呢!虽然接着睡了,可是眯上一段时间,又惦记着什么似的,醒来了。只要睁开眼,朦胧中会望一眼窗外——啊,月亮还在林间,只不过更低了些。再睡,再醒来,再望,也不知循环往复了多少次。【C】月亮终于沉在林地上,由灯的形态,变幻成篝火了„„这是那一夜的月亮,留给我的最后印象。

⑦第二天彻底醒过来时,天已大亮。哪还有满月时的圣景。消尽了白雪而又没有返青的树,看上去是那么的单调。虽然寻不见月亮的踪迹,但我知道它因为燃烧得太忘我了,动了元气,此后的半个月,它将一点点地亏下去。待它枯槁成弯弯的月牙儿,才会真正复苏,把亏的地方,再一点点地盈满。它圆满后,不会因为一次次地亏过,就不燃烧了。因为月亮懂得,没有燃烧,就不会有灰烬;而灰烬,是生命必不可少的养料。

⑧我怎么能想到,在印象中最不好的赏月时节,却看见了上天把月亮抛在凡尘的情景呢。假使我彻头彻尾醒着,这样的风景即使入了眼,也不会摄人心魄。正因为我所看到的一切在黎明与黑夜之间,在半梦半醒之间,那团月亮,才美得夺目。

13.阅读第①段,说说作者认为太阳和月亮有什么不同。

14.请从修辞或词语运用的角度,简要赏析文章第②段中加框语句。

15.文章A、B、C三处画线的语句具体描写了“那团月亮”夺目的美,请任选一处,说说作者是怎样表现那种美的。

16.说说文章第⑦段中“没有燃烧,就不会有灰烬;而灰烬,是生命必不可少的养料”一句给你哪些启示。

四、表达·交流

17.读材料后作文。

一个人在高山之巅的鹰巢里,抓到了一只幼鹰,他把幼鹰带回家,养在鸡笼里。这只幼鹰和鸡一起啄食、嬉闹和休息。它以为自己是一只鸡。

这只鹰渐渐长大,羽翼丰满了,主人想把它训练成猎鹰,可是由于终日和鸡混在一起,它已经变得和鸡完全一样,根本没有飞的愿望了。

主人试了各种办法,都毫无效果,最后把它带到山顶上,一把将它扔了出去。这只鹰像块石头似的,直掉下去,慌乱之中它拼命地扑打翅膀,就这样,它终于飞了起来!

由此我们得出结论:磨炼是召唤成功的力量。

请自选角度写一篇600字以上的文章。

推荐第7篇:九年级语文测试卷

思旸中学九年级语文第一次月考

班级 姓名 学号

一、选择题(共15分,每小题3分)

1、下列词语加点字注音完全正确的一项是()

A、踝骨剽悍狡黠哺育

B、押解瞥见笃信出殡

C、讪笑庇护绮丽车辆

D、猝然孪生吮吸匕首

2、下列词语中没有错别字的一项是()

A、雍容经典韬光养晦筚路蓝缕,以起山林

B、安详遐思薪尽火传二人同心,其利断金

C、跨越振辐秘而不宣合抱之木,生于毫末

D、坐阵砥砺学以致用壁立千仞,无欲则刚

3、下列句中加横线的成语使用不正确的一项是()

A、漫天大雪封住了他们的眼睛,使他们每走一步都忧心忡忡,因为一旦偏离了方向,错过贮藏点,无异于直接走向绝路。

B、要把成都建成世界现代田园城市,环境保护是基础,我们应均衡利用和开发环境资源,合理划分城市生态功能区,因地制宜规划发展。

C、在学校创新作文比赛中,李明的《心中的温暖》一文,构思别具匠心、选材新颖典型,获得了老师和同学们的一致好评。

D、小英在全省中学生舞蹈大赛中,以其优美舒展的舞姿和新颖的编排,征服了评委,获得了一等奖,全家人简直是乐此不疲。

4、下面句子没有语病的一项是()

A.忽视母语修养会让母语中独特的文化意蕴在我们的日常生活中日渐逐步消退。

B.为提高市民行车的舒适度,有关部门对多处路井部位进行了整修,解决了颠簸现象。

C.教育行政部门提出,要制定进城务工人员随迁子女义务教育后在当地参加升学考试。

D.日前,分藏两岸六十余年的国画名作《富春山居图》在台北故宫博物院“合璧”展出。

5、下列作家、作品搭配错误的一项是()

A、《雨说》——郑愁予《星星变奏曲》——江河

B、《夜》——叶塞宁《致女儿的信》——苏霍姆林斯基

C、《蝈蝈与蛐蛐》——济慈《敬业与乐业》——鲁迅

D、《傅雷家书两则》——傅雷《蝈蝈与蟋蟀》——济慈

6、按要求填空。(共8分)

(1)无可奈何花落去,(《浣溪沙》)

(2)晴川历历汉阳树, 。

?烟波江上使人愁。(《黄鹤楼》)

(3),路上行人欲断魂。(《清明》)

(4)王维的《使至塞上》以传神的笔墨,刻画了奇特壮美沙漠景象的两句诗是:,。 cùluánjiâhuáipiāoxiãbǔpiēdǔbìnshànpìqǐliàngyǔnbǐ

(5)《论语》中“”一句阐释了学习和思考的辩证关系。

7、仿写句子(4分)

若要有明亮的眼睛,请多看到别人的长处;若要有姣好的面容,请多展示真诚地微笑;,;若要有宽广的胸怀,请多接纳别人的忠言。

二、阅读(53分)

(一)(13分)

陈胜者,阳城人也,字涉。吴广者,阳夏人也,字叔。陈涉少时,尝与人佣耕,辍耕之垄上,怅恨久之,曰:“苟富贵,无相忘。”佣者笑而应曰:“若为佣耕,何富贵也?”陈涉太息曰:“嗟乎!燕雀安知鸿鹄之志哉!

8、解释下列加横线的字(4分)

(1)陈胜者,阳城人也

(2)尝与人佣耕

(3)苟富贵,无相忘

(4)若为佣耕,何富贵也

9、用现代汉语翻译下列语句。(6分)

(1)陈涉少时,尝与人佣耕,辍耕之垄上,怅恨久之

(2)嗟乎,燕雀安知鸿鹄之志哉

10、陈胜为什么会发出“燕雀安知鸿鹄之志”的感叹?(3分)

(二)

沁园春.雪(13分)

北国风光,千里冰封,万里雪飘。望长城内外,惟余莽莽;大河上下。顿失滔滔。山舞银蛇,原驰蜡象,欲与天公试比高。须晴日,看红装素裹,分外妖娆。

江山如此多娇,引无数英雄竞折腰。惜秦皇汉武,略输文采;唐宗宋祖,稍逊风骚。一代天骄,成吉思汗,只识弯弓射大雕。俱往矣,数风流人物,还看今朝。

11、“山舞银蛇,原驰蜡象”采用了什么手法?(3分)

12、说说“江山如此多娇,引无数英雄竞折腰”一句,在文中起什么作用。(3分)

13、如何理解“惜秦皇汉武,略输文采;唐宗宋祖,稍逊风骚。一代天骄,成吉思汗,只识弯弓射大雕。”(3分)

14、诗中点明题旨的句子是哪一句?联系诗中提到的历史人物,体会主旨句的含义。(4分)

(三)

美丽如初(12分)

月色皎洁,一如闪亮的白绸,宁静而安详地弥漫。我握着母亲的手站在街口,等放晚学的弟弟归家。并不冷,然而街静人空,我等得焦急不耐烦,母亲却等得耐心又耐心,遥望着那条很宽很白的路,母亲说:“一直这样等,惯了。”我的心梀然一动,目光在母亲单薄的身影里模糊了。

我也曾让母亲这样的等待过来,并不是小的时候,女儿大了,反而更让母亲牵挂。那些个月朗星稀的夜晚,和学友们一路高歌神侃地回家,一个人转进偏僻的街口,却望见母亲的衣衫和着树影飘动,一样的迷离,心忽地跳快了,跑过去,却只叫了一声“妈妈!”母亲也不说什么,很欣慰的笑着,拍拍我的手,一起走回家去。

而今我去了异地,只在假期里归来,母亲的身影却依然准时地站在街口树下,等待不久也要离家求学的弟弟。我忽然很羡慕母亲,可以把那么深沉的爱包容在静静的等待中。

清脆的铃声响过来,弟笑嘻嘻立在我和母亲面前:“妈!姐!”“怎么才回来,让妈等那么久?”我半是欢喜半是埋怨。“回家吧!”母亲还是那么欣慰的笑着,拍拍弟的手。弟冲我做了个鬼脸。看地高大的身影在母亲的身边,我忽然觉得失去了什么,起航的船只能留恋温暖的港湾,却不能永远停泊。

返校之前,母亲安静地替我整理行囊,见我跟着她走来走去,却不开口,母亲说:“从前你外婆也是这样送我走。”啊,我默默地望着母亲,仿佛看见外婆的双手在忙碌。我忽然明白了,从前外婆一定也曾站在街口,等母亲回家,就像母亲今天等我们回家,而卧也会有那么一天,让深深的四溶在等待中。

眼前晃动着月光里母亲静立的身影,才知道不论经过什么,我记忆中的那些夜晚永远美丽如初。

15、文章首段的景物描写写出了景物的什么特点?在文中起什么作用?(4分)

16、首段中,在等放晚学的弟弟时,为什么我会“等得焦急不耐”,母亲却“等得耐心又耐心”?(2分)

17、在第四段中写到,“我忽然觉得失去了什么”,“我”觉得失去了什么呢?(2分)

18、纵观全文,写出在全文结构上与段末中“眼前晃动着月光里母亲静立的身影”这个句子相呼应的一句。然后说说为什么“我记忆中的那些夜晚永远美丽如初”?(4分)

(四)

枪打落尾鸟(15分)

或许是囿于中庸之道几千年来的思想束缚,许多国人认为“木秀于林,风必摧之;名高于世,众必毁之”,认为“出檐的椽子先烂掉”,认为“夫唯不争,故莫能与之争。”为人处

事只要随大流,中规中矩,亦步亦趋,就能安身立命,明哲保身;而如果遇事喜欢挑头的话,就会被别人抓住把柄,落得个“枪打出头鸟”的下场。

然而,在当下这个竞争日趋激烈,各种挑战纷至沓来的时代,如果你选择做“落尾鸟”,结局又会怎样呢?

很遗憾地告诉大家,时代在发展,鸟儿也要进化。当枪声响起的时候,只要鸟儿飞得够快,决不会被猎人打死,还能吃到新鲜的食物。而那些落尾的鸟最有可能被猎枪击中,从而惨遭淘汰。

1回想一下鸦片战争后中国遭受的一百多年屈辱,○2回想一下当今世界上某些霸权主○

义国家对其他国家的指手画脚,3回想一下世界经济大潮中因工艺落后而破产的企业,4○○回想一下因业务不精而下岗的工人,○5我们不得不再次承认——落后就要出局,落后就要挨打。○6鸟儿如此,一个人、一个团队、一个民族也是如此。

遇到危险时不敢出头、不愿出头的人,最后往往会退化成没有能力出头的人;而这种人,面临的就是被淘汰的命运。

许多千里马一直都在埋怨没有伯乐赏识,但如果你不出头,即便你真的是匹千里马,也只能穷其一生去拉盐车,这样一来,伯乐又怎能发现你?

所以,如果你不想成为被枪打的落尾鸟,就要敢于出头,勇于出头。虽然出头鸟也有被打的可能,但这种可能仅仅说明你还不够成熟,如果你够成熟,尽管成为“出头鸟”吧,因为出众的人,只有经过“枪打”才会更加成熟,更加被人认可!

人生就是赛场,当枪声响起的时候,你做好振翅高飞的准备了吗?

19、本文的中心论点是。(2分)

20、文章第一段主要用了什么论证方法?有何作用?(4分)

答:

21、做落尾鸟,有何弊端?(3分)

答:

22、文章第四段的六句话,请你重新排列句序,并说明理由。(4分)

句序:,理由是

23、文章结尾一段能否去掉?为什么?(2分)

答:

三、综合性学习(10分)

24、县里将举行运动会,学校将选派几名同学参加校拉拉队为运动员呐喊助威。假设你参加拉拉队的竞选,请你完成以下工作:(5分)

(1)请你为拉拉队设计一个呐喊助威的口号,要求是用两个句子,朗朗上口,有鼓动性。

(2)拉拉队需做好哪些工作?请你提出两点建议。

25、近年来,在东亚“汉字文化圈”中,“年度汉字”的评选已成为一个热门的话题。如日本2008年的年度汉字是“变”,2009年地年度汉字是“新”,这两个字均反映了日本近两年间政坛演变的特征,具有高度的概括力和较强的象征性;中国2008年当选的年度汉字是“雷”,2009年“被”字的拥护者颇多,这两个字具有鲜明的中国网络文化特征,在一定程度上反映了当前的民情民意。(5分)

请根据你对时事的了解为今年的中国遴选一个“年度汉字”,并简要说明你遴选的理由。年度汉字

遴选的理由

四、作文(60分)

有人憧憬雪花飞扬的冬,有人心仪草长莺飞的春,有人喜欢阳光灿烂的夏,有人钟情天高云淡的秋。岁月更迭,四季交替,总有一个季节让人期盼,总有一片天空让人翱翔,总有一段往事让人回味,总有一份精彩属于自己。

请以“总有属于我的季节”为题写一篇文章。

要求:1将题目抄在答题卡上;2除诗歌、剧本外文体不限;3不要少于600字;4○○○○文中不要出现(或暗示)本人的姓名、校名

1——5 BBDDC

6、(1)似曾相识燕归来

(2)芳草萋萋鹦鹉洲日暮乡关何处是

(2)清明时节雨纷纷

(3)大漠孤烟直长河落日圆

(4)学而不思则罔思而不学则殆

7、略

8、(1)判断词,

(2)曾经

(3)如果

(4)你

9(1)陈胜年轻的时候,曾经跟别人一道被雇佣耕地。(有一天,)他停止耕作走到田边高地(休息),因失望而叹息了好长时间以后

(2)唉,燕雀怎么能知道鸿鹄的凌云壮志啊

10、用形象的比喻说明了他非凡的个性和远大的报负

11、“山舞银蛇,原驰蜡象”中“舞”、“驰” 化静为动、动静相衬。运用了比喻和对偶的修辞手法。将披雪的群山比作“银蛇”飞舞,将白雪覆盖的丘陵比作巨象奔驰。

12、由描写雪景过渡到直接地抒情、议论的

13、论今,引出人们对今朝风流人物的赞颂。

14、“俱往矣,数风流人物,还看今朝。” 诗人自况;诗人所代表的领导人民群众进行斗争的无产阶级革命家。他们都是新时代在文治和武功方面都有更杰出才能、更伟大的抱负的人。还有历史真正的主宰——人民群众。他们一定能为社会的建设、历史的发展,贡献出自己全部的力量。

15、宁静平和展示人物活动环境特点;衬托人物美丽和充满母爱的性格

16、因为母亲能把“深深的爱包容在静静的等待中”,而我还未能

17、应为我已经长大独立,所以我已经失去了已在母亲怀抱里感受母爱的机会,失去了享受母亲等待的满足感和幸福感的机会

18、目光在母亲单薄的身影里模糊了因为那些夜晚凝聚着母亲深沉美好的爱

19、中心论点是“枪打落尾鸟”或“勇做出头鸟”。

20、道理论证(或引证法)(对比论证亦可)是为了引出传统观点:枪打出头鸟。

21、做落尾鸟会退化成没有能力出头的人,面临被淘汰的命运。

22、(3分)⑥④③①②⑤理由:⑥句是承上启下的过渡句;⑥句与④③①②是总分、对应关系。

23、(2分)不能去掉;因为结尾一段强化了观点(或警示读者勇做出头鸟)。

推荐第8篇:九年级语文测试卷

九年级测试卷

一、语言积累与运用(30分)

1 .下列加点字注音完全正确的一项:()(2分)

A、秀颀‧ qí婆挲‧suō阔绰‧chuò‧

恹恹欲睡yān

B‧皲裂jūn‧漪郁yī‧谄媚chǎn瘦骨嶙峋‧‧

línxún

C、熠熠‧‧yì‧渚崖zhǔ‧矜持 jīn颓垣‧

断壁yuán

D、狡黠‧jié穷匮‧kuì哽咽‧yè‧

锲而不舍qiè2.下列词语中没有错别字的一项是()(2分) A.殉职腼腆言简意赅人事沧桑 B.揶揄震摄毛骨悚然名副其实 C.噩耗仓皇神彩奕奕眼花缭乱 D.宣泄希冀坦荡如砥不屑臵辨

3、下列句子中加点的成语运用错误的一项是()(2分) A、为了把自然灾难的损失降到最低,我市许多学校未雨绸缪,举行了灾难求生救生大演练。

B、对于日本在钓鱼岛问题上歪曲事实的做法,中国人民无不义愤填膺。 C、古往今来成大事者,不惟有惊世之才,更兼有坚韧之志,因此,面对学习上的困难,我们应有锲而不舍的精神。 D、中考结束后,李明打算和几个同学结伴出游,到大自然中去共享天伦之乐。

4、下列句子没有语病的一项是()(2分) A、由于杨建一校长勇斗歹徒以身殉职,被网友誉为“最美校长”。

B、为了防止校车安全事故不再发生,国务院法制办起草公布了《校车安全条例草案征求意见稿》。

C、实现中华民族的伟大复兴,就是中华民族近代以来最伟大的梦想。

D、纪录片《舌尖上的中国》不仅引发了人们对“文化认同”和“软实力输出”的思考, 而且让人怀念童年时的美味。

5、下列文学常识表达有错误的一项是()(2分) A、《繁星》《春水》是冰心平时随手记下的“随时随地的感想和回忆”,主要表现了“母爱”和“童心”两大主题。

B、郭沫若的《石榴》、朱自清的《背影》、茅盾的《白杨礼赞》都是脍炙人口的散文名篇。

C、柳宗元是唐代著名文学家,唐宋八大家之一。我们学过他的《捕蛇者说》。 D、英国作家丹尼尔〃笛福在他的小说《鲁滨逊漂流记》中,为我们讲述了主人公鲁滨逊在小人国、大人国历险多年,吃尽千辛万苦,终于得到了可观的财富,回到家乡的故事,

6、为下面这则消息拟写的标题最恰当的一项是()(2分) 本报讯5月13日下午3时,娄底城区应急疏散演练警报声响起。娄底市星星实验学校7-9年级学生有组织地迅速疏散到盛世华庭防空地下室指定区域。这是我市开展应急疏散演练活动的一个场景。这一天,除娄底中心城区外,涟源、冷水江、新化、双峰等县市也都举行了应急疏散演练活动。

根据省政府办公厅《关于印发〈湖南省应急疏散演练活动方案>的通知》,为了进一步提高公众防灾减灾意识和应急避险能力,完善相关应急预案,我市制定了《娄底市应急疏散演练活动方案》。疏散演练分为社区疏散演练和学校疏散演练.涟源市应急疏散演练设涟源市三一学校,演练现场活动情况通过移动应急小平台将信号传送到我市指挥中心,我市相关单位负责人和工作人员在指挥中心观摩了现场演练情况。通过演练让师生们从汶川、雅安地震惨痛的事件中吸取血的教训,当灾难来临时快速疏散,懂得科学自救,珍爱生命.

A.我市城区应急疏散演练警报声响起B.我市举行应急疏散演练

C.我市制定了《娄底市应急疏散演练活动方案》D.珍爱生命,科学自救

7、用正楷或行楷将下列古诗文默写在田字格中(10分,默写每空1分,书写2分) (1)在《使至塞上》中,描写落日时分边关所见,以其写景壮观,气势流畅,被王国维称之为“千古壮观”的名句是:

“,。”

(2)使用比喻,曲折丰富地表达两个有情人之间生死不渝的爱情的句子是:“,。”

3、苏轼的《水调歌头》中体现作者深挚崇高的思想境界和乐观思想,表达对离别的人们的美好的祝福的名句是: __________________,___________________。 (4)在《浣溪沙》中,被誉为“天然奇偶”,传诵千古的佳句是:,。

8.根据下面提供的情境回答问题(2分)

2013年国庆节,清洁工人在**广场扫出近8吨垃圾。假如你是在场的一位小游客,发现身旁一位叔叔正淮备把香蕉皮扔到地上,你应该怎么对他说? 答:

9.完成下面两道综合性学习题(6分)

(1)下面是九年级

(一)班班长李强留给学校后勤管理员的借条,在内容和格式上均有一处错误,请找出并加以修改。(3分)

借条

今借到后勤管理处活动经费258元,用于五四青年节合唱排练。

九年级

(一)班 2013年4月18号

答:

(2)最近,餐桌上的浪费现象成为了人们热议的话题。在政府狠刹公款吃喝风的同时, 民众也自发掀起了“光盘”运动。请就餐桌浪费现象谈谈你的想法。(3分) 答:

二、阅读(40分) .蒋氏大戚,汪然涕曰:“君将哀而生之乎?则吾斯役之不幸,未若复吾赋不幸之甚也。向吾不为斯役,则久已病矣。自吾氏三世居是乡,积于今六十岁矣,而乡邻之生日蹙,殚其地之出,竭其庐之入,号呼而转徒,饥渴而顿踣,触风雨,犯寒暑,呼嘘毒疠,往往而死者相藉也。曩与吾祖居者,今其室十无一焉;与吾父居者,今其室十无二三焉;与吾居十二年者,今其室十无四五焉。非死则徙尔。而吾以捕蛇独存。悍吏之来吾乡,叫嚣乎东西,隳突乎南北,哗然而骇者,虽鸡狗不得宁焉。吾恂恂而起,视其缶,而吾蛇尚存,则弛然而卧。谨食之,时而献焉。退而甘食其土之有,以尽吾齿。盖一岁之犯死者二焉;其余,则熙熙而乐。岂若吾乡邻之旦旦有是哉!今虽死乎此,比吾乡邻之死则已后矣,又安敢毒耶?”

10、解下面加点词的意思正确的一项。()(2分)

(1)蒋氏大戚‧( 悲伤)(2)向‧

吾不为斯役(假使)

(3)曩‧与吾祖居者( 以前)(4)时‧

而献焉( 时时)

11、用∕给下列句子划分节奏:(2分)

而乡邻之生日蹙,殚其地之出

12、句子翻译(4分)

(1)则吾斯役之不幸,未若复吾赋不幸之甚也.___________________________________________________________________ (2)比吾乡邻之死则已后矣,又安敢毒耶?

___________________________________________________________________ 13.对“又安敢毒邪?”的理解正确的一项是()(2分)

A、蒋氏不敢怨恨捕蛇这件事,因为这是太医以皇帝的命令发的。

B.此句与上段作者的问话“若毒之乎”相呼应,使一篇血泪斑斑的控诉,带上“哀而不伤”的色调,蕴含更加深刻。

C.蒋氏在控诉捕蛇给他一家三代带来的不幸。

D、蒋氏不敢怨恨“复若赋”这件事,因为他知道“复赋比捕蛇的毒害更深”。 14.选文叙述蒋氏和乡邻的遭遇,主要运用了对比的手法,请从中找出一组对比句,结合选文简要分析其作用。(3分)

(二)泊秦淮杜牧

烟笼寒水月笼沙,夜泊秦淮近酒家。

商女不知亡国恨,隔江犹唱后庭花。

15、“烟笼寒水月笼沙”这句诗中的这句诗描绘了一幅什么样的画面(2分)

16、真正“不知亡国恨”的是什么人,这首诗表现了诗人什么思想感情?(2分)

(三)让我们相信人性的正能量①必须承认,这个世界并不总是那么的美好,我们的人性也并不总是那么的高尚。“小悦悦事件”、“彭宇案”都曾经一度让人们感到道德的迷茫:为什么好人得不到好报,为什么周围都是冷漠的目光?于是,有人沮丧,有人彷徨,似乎人们已经找不到方向,似乎社会已经失去了力量。但张丽莉、王世伟等六位龙江英雄的事迹一次次告诉我们:世界的真相,并不是这样!社会也好,人性也罢,都有着像硬币一样的正反面,我们不要以为看到了一面就看到了答案,这个世界,有黑暗就有光亮,有失望也有希望。而这光亮与希望,其实,就在我们每个人身上。

②我们在报道中看到,当护士拉开窗帘,张丽莉看见照进病房的阳光,不禁由衷地感叹:“活着真好。”而在张丽莉、王世伟、荆百岁、谢尚威、高铁成、郭肖岐等人的身上,也让我们看到了同样的阳光,那是人性散发出来的希望光芒,也会

让我们感叹世界的美好与生命的坚强。正如本报在27日推出的《发现黑龙江之美〃大爱读本》对于六位英雄的解读,在他们身上,都有着热爱生活、与人为善、心怀责任、目标坚定、乐观坚强、平凡可爱的特质,这些都是人性在日常生活中最美好的微茫,而在危难时刻,便会转化成闪电一样的力量。诗人说:“在没有英雄的年代,我只想做一个人。”但其实,能把人做好了,就是最大 的英雄,就有无穷的能量。

③我们相信,无论人的本性原来怎样,但最终的方向一定是向善向上,而人性中最重要的品质,也一定是善良。正如法国作家雨果所说:“善良是历史中稀有的珍珠,善良的人几乎优于伟大的人。心怀善良的人,总是在播种阳光和雨露,医治人们心灵的创伤。同善良的人接触,智慧得到启迪,灵魂变得高尚,襟怀更加宽广。”而在前几天,著名教授易中天先生在来哈的讲座中,也一再强调,与其空谈信仰,不如坚守道德底线,而这底线的标准之一,就是恻隐之心,而恻隐之心的实质,就是善良。这种人性的善良,用易中天先生曾经的比方,“就是全人类都能输的0型血”,能够拯救道德的沦丧,能够温暖世道的寒凉。

④无论是英雄挺身而出的义举,还是民众爱心涌动的景象,这一切的一切,都让我们相信,只要社会中的“正能量”充足,就一定会形成道德与爱的强大气场。而人性的美好和善良,也最终一定能给这个世界带来无尽的力量,无尽的希望,像水融入着水,像光簇拥着光。

17.本文的中心论点是什么?(2分)

18.为了论证中心论点,作者主要采用了哪些论证方法?(2分)

19.选文第①段中列举“小悦悦事件”、“彭宇案”的事例有什么作用?(2分)

20.文章第②段写道:“能把人做好了,就是最大的英雄,就有无穷的能量。”请结合你的学习与生活,谈谈我们应该聚积和传递哪些“正能量”?(3分)

(四)

新的学期,刘老师到一个班当班主任。她发现班上有一个奇怪的男生,每天中午放学后,别的学生一窝蜂似的涌出教室,只有他,坐在教室里,拿出书本来看,或者拿出作业来做。他的名字叫张晓军。

一天中午,刘老师走进了教室,她对张晓军说:“张晓军,放学了,快回家吃

饭吧!”张晓军说:“老师,我知道,我想等会儿再回家!”刘老师问道:“你不饿吗?”张晓军说:“饿!”刘老师说:“那就快回家吧!”张晓军说:“我等会儿就回家!”说完,张晓军又埋头做作业。

这是一个固执的孩子,刘老师想,他肯定是想把成绩赶上去,便忍不住对张晓军说:“饿着肚子可是学不好的,你回去吃饭吧,吃了饭到学校找我,我可以帮你补习功课!”

张晓军说:“老师,您误会了。其实,我迟些回家,并不是为了抓紧时间争分夺秒地学习……”刘老师睁大眼睛:“那你是为了什么啊? ”

张晓军告诉刘老师,他的父亲早年去世了,母亲为了供他和妹妹读书,便外出打工,家里只有他和妹妹。每天早上,他和妹妹一早就起床做好饭菜,中午回家热一下就可以吃。由于饭量大,他担心自己吃多了,妹妹就吃不饱,便迟些回家。这样,妹妹就可以先吃饭,剩下的,就全是他的,他就可以放开肚皮吃。

原来是这样。刘老师问张晓军为何不多做些饭菜,张晓军说他们每个月的钱只有那么多,要是多做些,钱就不够花。他不忍心告诉母亲家里的情况。他说要是让母亲多寄钱回家,母亲就会起早贪黑地干活,会很辛苦.张晓军说他少吃一点没什么,不能苦了母亲,饿了妹妹。

刘老师默默地听著,湿了眼睛。①她掏出钱包,拿出一百元钱,递给张晓军.张晓 军连连后退,说:“老师,我不能要您的钱!您放心,我的问题我能解决。现在,我每天都在悄悄地捡废品,我能挣钱,很快就能吃饱饭了。”

刘老师知道,这是一个要强的孩子。她只好作罢,收起了钱。

以后的每个中午,刘老师依然发现张晓军迟迟不回家.每次看到教室里埋头学习的这个孩子,刘老师就叹息不已。

下一学期,刘老师因为工作需要,离开了这个班级,当了另一个班的班主任。让刘老师没想到的是,在新的这个班,她又遇到了一个奇怪的女生。这个女生每天中午放学铃声一响,她还没走出教室,女生就第一个冲出了教室。女生的脚步匆匆忙忙,似乎有什么急事。有时候踉踉跑抢,几欲跌倒,让人好不担心。

一天两天有急事没什么,可是总是有急事,那就不正常了。有一天中午,刘老师在放学之前点了女生的名,让她放学后留下来。没想到,女生竟然站了起来,说:“老师,放学我不留下来,行吗?”刘老师说:“不行,我有事找你!”女生说:“老师,等我下午来上学的时候,我到办公室找你,行吗? ”看着女生乞求的目光,刘老师点头答应了。

下课铃声一响,女生又是第一个冲出了教室,她的脚步依然是那样匆匆忙忙,依然是那样跟踉跑跄。

下午,女生走进了刘老师的办公室,她说:“老师,我来了。您找我有什么

事? ”刘老师问她:“为什么每天中午放学后你总是第一个冲出教室? ”女生说:“我急着回家……” 刘老师说:“急着回家?肚子再饿,也用不着急那一会儿。你跑那么快,很危险的,你知不知道? ”

女生说:“老师,对不起,让您担心了。”刘老师让女生以后放学别急着回家,可女生却说不行,刘老师问她为什么,女生说:“为了我哥!”

女生告诉刘老师,她的父亲早年去世,母亲为了供她和哥哥读书,外出打工,家里只有她和哥哥。每天早上,她和哥哥一早就起床做好饭菜,中午回家热一下就可以吃。哥哥饭量大,她担心哥哥吃不饱,便先到家吃饭,这样,剩下的,就全归哥哥,他就可以放开肚皮吃饱饭了。

刘老师想起了什么,她突然想到女生的名字叫张晓燕,不用说,她就是张晓军的妹妹.刘老师问张晓燕:“这么说,你为了让你哥吃饱饭,从来就没有吃饱过?张晓燕连连摇头,她说:“不,我吃饱了,吃饱了!”刘老师知道,张晓燕没有说实话.刘老师掏钱给张晓燕,却被拒绝了。张晓燕说她在悄悄捡废品,等她挣到钱,她就可以买很多米和菜,就可以和哥哥一起回家吃饭了 。

后来的日子里,每天中午放学的时候,刘老师看到张晓燕第一个冲出教室,她就禁不住双眼潮湿。②她知道,那匆匆忙忙、踉踉跄跄的脚步里,跑出的是爱的直线。

21、从文中找出一个恰当的短语作为选文的标题.(2分)

22、选文①②两处画线的句子,表现了刘老师怎样的心情?(3分)

23、请结合全文,分析妹妹张晓燕的形象。(3分)

24、刘老师先后担任兄妹俩的班主任,这是一种巧合。这种巧合在文中有何作用?(2分)

25、结合生活实际,谈谈选文给了你什么启迪。(4分)

三、作文(50分)

26、根据下面的题目,按要求作文。

电影《致青春》真实地再现了

八、九十年代的校园生活,展示了莘莘学子的人生观和价值观。此电影一上映,就引起了广大观众的共鸣。

同学们,也许在你的生活中也有拨动心弦的一件物品、一个人物、一道风景或一段情感……并让你深有感悟。请以“致______”为题写一篇文章。

附:答案

一、1.D 2.C 3.D 4.C5.D6.B 7.大漠孤烟直,长河落日圆;春蚕到死丝方尽,蜡炬成灰泪始干;但愿人长久,千里共婵娟;门前流水尚能西,休将白发唱黄鸡;8.言之成理即可9.㈠内容:258元应该大写;格式:九年级一班后面加上李强。㈡围绕爱惜粮食,不要攀比等方面展开均可。

二、阅读

110.㈠11.而/相邻之生日/蹙,殚/其地之出

12.㈠我干这差事遭受的不幸,是远不如恢复租赋遭受的不幸厉害啊㈡比起我的乡邻们已经死在他们后面了,又怎么敢怨恨他呢? 13.( B)

14.任选一个对比句,例如:曩与吾祖居者,今其室十无一焉;与吾父居者,今其室十无二三焉;与吾居十二年者,今其室十无四五焉。非死则徙尔。而吾以捕蛇独存。通过邻人与蒋氏的对比,充分体现苛政猛于

虎的社会现实,此题言之成理,抓住重点即可。 阅读

2父母担忧,又是一个爱家人的孝女。

24.前后照应,使人为之一震,让人不觉感叹,引出文章主题。15.此句写景,“烟”、“水”、“月”、“沙”由两个“笼”字联系起来,融合成一幅朦胧冷清的水色夜景,为下文写商女做铺垫。 16.真正不知亡国恨的是指那些只知征歌征舞,卖笑逐欢的统治者和官宦子弟,体现了作者内心深深的愤懑和谴责。 阅读三

17.中心论点是让我们相信人性的正能量。18.举例论证、引用论证、比喻论证

19.使论据更有说服力、是论点更信服,增加感染力。

20.此题灵活,只要抓住重点就行,比如:帮助别人,传递正能量;遵守社会秩序,尊老爱幼,传递正能量;危难时刻的挺身而出等等,都是正能量。 阅读4 21.爱的直线

22.第一次也许只是感动,同情这个孩子,第二次则是心灵的震撼 23.是一个理解家庭,理解兄弟的懂事的女孩子,并且好好学习,不让

25.此题灵活,只要抓住主题即可。

三、作文,

26.作文满分60分,要求如下:切合题意,主题明确,语句流畅,表达有创意,600左右,不能出现真实的人名和校名。

推荐第9篇:九年级语文测试卷

九年级语文测试卷

-、语文知识积累与运用(26分)

1、下面一段文字,请根据具体语境,根据拼音写出相应的汉字或给加点字注音(4分)。

那是一片候春的秋叶。尽管它早已不能为树木光全作用,对新一代的花朵无法作出实际奉献,但它那为新一轮生命的dàn()生,„„

秋叶候春竟然不落只是个案。抚养和shàn()养,应该是人生美丽的循环。一片玉兰秋苦苦候春,直到新一轮花苞膨胀欲放前夕才悄()然离枝,这一悲壮雅丽的个案,启示我们更深入地体味天下父母心。我们或许都能“可怜天下父母心”,我们一定也能“可怜天下父母心”吗?生生不息的人类啊,在你栖()息的大地上,有多少这样的细节,值得你用心灵支亲近„„

2.读下边一则新闻消息,用一句话概括其要点(不超过15个字)。(4分) 本报北京9月21日电(记者王立) 为改善首都及周边生态环境,涵养和保护北京水源,防治沙尘暴,为北京成功举办绿色奥运作贡献,一项大型公益项目——京水源头林公益工程今天在北京正式启动。国家林业局、北京市有关领导及京水源头所在地领导、粤港企业家等近百人出席启动仪式。

北京人民赖以生存的密云、官厅水库的水源都在河北省承德和张家口。经论证,要更好地保护、涵养和清洁北京的水源,最直接有效的办法就是在京水源头实施大规模的植树造林。据介绍,京水源头林公益工程的63万亩森林,将涵养和保护两亿多吨用水,每年可减少大量的泥沙流入这两座水库。此项重大公益工程由中国新闻文化促进会、广州宏宇集团等单位发起主办,并成立了组委会。

3、下列句子没有语病的一句是 () (2分)

A、学校希望通过开展节约用电教育,防止同学们不浪费用电。

B、能否推进素质教育,是保证青少年健康成长的条件之一。

C、我们要引导青少年用美的眼光去看世界,用美的心灵去感受世界。

D、通过这次社会实践活动,使我们磨练了意志,增长了见识。

4、古诗文默写。(10分)

①西北望,射天狼。(苏轼《江城子》)

②鸡声茅店月,。(温庭筠《商山早行》)

③零落成泥碾作尘,。(陆游《卜算子》)

④莫道不消魂,___________,人比黄花瘦。(李清照《醉花阴》)

⑤《武陵春》是南宋婉约派女词人李清照的作品,词中最为人称道的写“愁”颇

有新意的句子是,。

⑥范仲淹在《渔家傲? 秋思》中,表现自己和征夫们想家却不甘无功而返的矛盾

心理的句子是,。

⑦诸葛亮鞠躬尽瘁地辅佐刘备父子,得到了人们极高的赞誉,这可借用辛弃疾的

词句“,”来概括。

5、《三国演义》和《水浒传》中,各塑造了一个聪明机智的“军师”形象。

写出他们的姓名及用计的一个事例。(4分)

《三国》中:

《水浒》中:

6、依照下面“自然的启示”类警句,自选一种事物,写一个托物抒怀、富含哲

理的拟人句。(2分)

示例: 稻穗——空虚者的头总是昂得很高。

篮球——容易坠入圈套,因为有人拍你。

7、阅读下面两则材料,回答问题。(4分)

如果说外语是人生的工具,那么,母语毫无疑问是掌握工具的手。可是,当中国

人在投入巨大精力学习外语、应付外语考试的同时,却尴尬地发现,自己的母语

水平正在明显下降。

据调查发现,中学生母语水平不容乐观,他们连深奥一点的现代文也理解不了,

普遍存在词汇贫乏,言不达意的情况。

(1)两则材料反映了一个什么现象?请用一句话概括。

(2)简要谈谈你对这一现象的看法。

二、现代文阅读(30分)忍耐与宽容(15分)

①你爱吃鱼,我爱吃肉,虽然嗜好各有不同,但缘分安排我们一桌共食。我

们也都吃到了自己喜欢的东西,这很好。

②如果我们能承认品质各有差异的客观存在,便会对彼此的差异感到快乐。

你有你的思考方式,我有我的思考方式,若是我们都能互相学习,彼此宽容,

就能一团和气。

③无论彼此有何不同,你我都有长处与缺点,如果我们能学习别人的长处,赞美别人的长处,努力改正自身的缺点,含蓄地指出别人的缺点,即可共同提高水平。不必去批评责难,也不必互相排斥,更不用怀疑别人是否出了毛病,、。能做到此境界者,才是真正的君子。

④砂糖是甜的,精盐是咸的。它们是味道的两极,互为正反,如果想要使食物尝起来是甜的,只要加点糖就可以了。然而事实上若我们再加入些盐,反而更能增强砂糖的甜度与味道。这是因为调和了互为正反的两种味道而产生的一种新鲜滋味,这正是造物主绝纱的安排。

⑤事物都有对立,都有正反。有对立的关系,我们才能感受到自己的存在,才能体会得出那种类似砂糖里加入了盐的滋味。

⑥所以,与其苦思如何去排除那些挥之不去的东西,还不如苦思如何去接纳、调和它们。如此,必能产生新的天赐美味,而康庄大道也就在我们面前展开了。

⑦一般人往往认为人与人之间的关系,可以凭自己的意志来促成或断绝。但事实并非如此,人与人之间的关系,并不是个人的“意志”或“希望”所能左右的,而是由一种超越个人的意志或希望的力量来决定的。

⑧明白了这个道理,就应该珍惜自己的人际关系,心中常怀感激之情,在任何不平或不满之前,先以谦虚的态度想到彼此的缘分,然后以喜悦的心情,热忱的态度对待对方。如果每个人都能这样,必然可以产生坚强无比的力量,使社会由黑暗变为光明。

⑨人与人相互依靠而生活,而从事工作。这世界上各类人都有,因此,惟有养成忍耐与宽容的品性,才能适应这个社会。

8、围绕“忍耐与宽容”作者主要论述的观点是什么?(3分)

9、一位哲人说过:“学会宽容,世界会变得更为广阔;忘却计较,人生才能永远快乐。”这句话和文中哪段所论述的观点是一致的呢?(3分)

10、怎样理解文中加线句的含议呢?(3分)

11、古今中外,有许多名人都具有忍耐和宽容的品质,你能说出其中的一位及他

们的事迹吗?

(3分)

12、阅读本文后,你对“忍耐和宽容”还有怎样的理解?结合自己对人生的感悟来谈一谈。(3分)

二、文言文阅读(15分)

出师表

① 亲贤臣,远小人,此先汉所以兴隆也;亲小人,远贤臣,此后汉所以倾颓也。先帝在时,每与臣论此事,未尝不叹息痛恨于桓、灵也。侍中、尚书、长史、参军,此悉贞良死节之臣,愿陛下亲之信之则汉室之隆可计日而待也。 ② 臣本布衣,躬耕于南阳,苟全性命于乱世,不求闻达于诸候。先帝不以臣卑鄙,猥自枉屈,三顾臣于草庐之中,咨臣以当世之事,由是感激,遂许先帝以驱驰。后值倾覆,,尔来二十有一年矣。

③ 先帝知臣谨慎,故临崩寄臣以大事也。受命以来,夙夜忧叹,恐托付不效,以伤先帝之明,故五月渡泸,深入不毛。今南方已定,兵甲已足,当奖率三军,北定中原,庶竭驽钝,攘除奸凶,兴复汉室,还于旧都。此臣所以报先帝而忠陛下之职分也。至于斟酌损益,进尽忠言,则攸之、袆、允之任也。

13、解释下列加横线词语在句子中的含义。(4分) ①躬耕于南阳() ②先帝不以臣卑鄙() ③臣本布衣 () ④故五月渡泸 ()

14、用“/”给第①段画横线的句子断句(断两处)。(2分)

愿陛下亲之信之则汉室之隆可计日而待也。

15、在原文的横线上填空。(2分)

16、翻译下列句子。(2分)

此臣所以报先帝而忠陛下之职分也。

17、在第①段中,诸葛亮向后主提出的建议是(用四字成语回答)(2分)

18、在②、③两段中,表达了诸葛亮怎样的情感?(3分)

参考答案

-、语文知识积累与运用

1、略

2、逶迤 磅礴

3、B

4、C ⒌①-⑤略; ⑥只恐双溪舴艋舟,载不动许多愁 ⑦燕然未勒归无计,浊酒一杯家万里 ⑧了却君王天下事,赢得生前身后名

6.略

7、竹 —每攀登一步,就做一次小结。 沙发—柔软不一定就是无能。气球—总是爆烈,因为有人吹你。

8.⑴不少中国人在重视外语学习的同时,轻视了母语的学习。(意近即可)⑵学好母语是热爱祖国的表现。要学好外语,必须首先学好母语。(从这两个角度任何一方谈都可,但必须突出学习母语的重要性)

二、现代文阅读

(一) 算命

9、表明母亲到得及时、快;“只要他一到,第一个去算命的便是我母亲”相照

应。

10、先“多次规劝”,再“继续开导”,最后“假扮农民去算命”。

11、通过算命来帮助算命先生,从而既帮助他人,又照顾到他人的尊严。

12、①是愚昧 ②自作聪明却不懂得同情弱者,帮助弱者。

13、略

(二)忍耐与宽容

14、养成忍耐与宽容的品质,能和谐人际关系,使自己愉陪,适应

15、和第⑧段内容相一致

16、人与人之间关系应建立在相互理解、宽容的基础上,以及其他社会因素等。(重在对“超越自然”的理解上,只要言之成理即可)

17、略

18、此题为开放题,可从正面揭示它的好处,也可从反面指出“忍耐和宽容”是有限度的。

三、文言文阅读

19、略

20、略

21、略

22、这就是我用来报答先帝和忠于陛下的职责和本分啊。

23、亲贤远佞

24、表达了诸葛亮报先帝和忠陛下的情感

25、①„①守边兵力薄弱,只有紧闭城门。②反映守军将士戍边决心大,紧闭城门死守孤城。③反映朝廷没有投放大量兵力,因而有势单力薄,孤寂之感。④边城紧张的战争气氛。(只要答出两点即可)

②.这是千古戍边将士的命运的真切写照,表现了他们壮志难酬的感慨和思乡忧国的情怀

推荐第10篇:九年级语文测试卷

民乐二中2012~2013学年

1(7)庭下如积水空明,,。(苏轼《记承天寺夜游》)

二、阅读部分(48分)

(一)阅读下文,完成9—13题。(共14分)

我是一片瓦 乔叶

①夏夜,我和几个朋友相约于郑州东区的“瓦库5号”。坐在露天的最顶层,清风在身,明月在上,红酒在口,香茶在壶,眼里是朋友的笑意,耳里是隐隐的乐声——我不禁惊异,居然有这样一个地方。

②客人不多,有的清谈,有的下棋,有的打麻将,还有的,只是静静地坐着。

③最多的,就是瓦。青瓦、红瓦、灰瓦,大瓦、小瓦,一扇一扇的瓦窗,整面整面的瓦墙,我们所处的顶层,则有成片成片的瓦顶„„不期然间,在任何一个角落里,你都会看到瓦的身影。它静静地待在那里,温和地沉默着。

④还有专门用来签名和题字的瓦,内容各种各样:上房揭瓦;美丽的瓦;来看瓦吧„„ ⑤忽然想,如果让我写,我写的也许是:我是一片瓦。

⑥五间青砖灰瓦的房子,曾经是我们家最重要的不动产,它如一件巨大的粗布衣衫,给我们全家以最简陋的也是最坚实的温暖包裹。生活在瓦下,但平时感觉不到瓦的存在。只有下雨的时候,我在屋檐下玩耍,伸出双手,任落雨在掌心汇聚如歌,偶尔会听到母亲叹息:该揭瓦了。我便知道:房顶某个地方漏雨了。于是,天晴以后,父亲便会找来泥瓦匠上房,揭开某个部分的瓦,在瓦下搪上一些泥巴,再把那些瓦盖上去。雨再来的时候,我们的房子便没有任何破绽可寻。

⑦渐渐长大,到了调皮的年龄,有一次,曾悄悄顺着院墙爬到房顶,去采摘一棵已经长成的胖胖瓦松,被母亲发现后自然是一顿狠狠的呵斥:小女孩家家的,怎么那么野呢?!

⑧但是感觉真的很好——那是我

屈,三顾臣于草庐之中,咨臣以当世之事,由是感激,遂许先帝以驱驰。后值倾覆,受任于败军之际,奉命于危难之间,尔来二十有一年矣 。先帝知臣谨慎,故临崩寄臣以大事也。受命以来,夙夜忧叹,恐付托不效,以伤先帝之明。故五月渡泸,深入不毛。今南方已定,兵甲已足,当奖率三军,北定中原,庶竭驽钝,攘除奸凶,兴复汉室,还于旧都。此臣所以报先帝而忠陛下之职分也。

18.解释下列加点词的意思。(4 分)

①将军既帝室之胄胄:②躬耕于南阳躬:③先帝不以臣卑鄙卑鄙:④ 故五月渡庐故:19.用现代汉语翻译下面的句子。(4 分) ①苟全性命于乱世,不求闻达于诸侯。

② 此臣所以报先帝而忠陛下之职分也。

20.联系两段文字看,甲段中的“将军”也就是乙段中的(1分) 21.甲、乙两段文字所展示共同目标是什么?(用原文或自己的话概括均可)(2分)

22.乙段文字回顾了对作者人生有重大影响的两件往事,它们分别可以概括为 什么 ?作者回顾这些往事的目的又是什么?(用自己的话概括作答)(3分)

23.在公众眼里,诸葛亮是智慧的化身,历来被人们所称颂,有关他的典故、成语、名言警句以及俗语等有很多,请列举两个(句、对)(2分)(不必每项都列出,只要写出两个即可)

(四)阅读下面这首诗,然后回答问题。(4分)

山居秋暝唐王维

空山新雨后,天气晚来秋。明月松间照,清泉石上流。 竹喧归浣女,莲动下渔舟。随意春芳歇,王孙自可留。

24.“诗中有画”是王维诗歌的特点。请用简洁的语言描绘“明月松间照,清泉石上流”所表现的画面。

第11篇:学年度第一学期九年级历史期中测试卷

_____ _:题号位座 答 ___________要____:名姓不 _______内___:级班 线 _______订__________装校:学2017~2018学年度

12.工业革命中交通运输领域的突破首先是在

A.美国

B.德国

C.法国

D.英国

13.中国铁道博物馆收藏着一台中国现存最古老的机车──“0”号机车。该机车1882年从英国购进,

运用于唐胥铁路。对这段材料认识不正确的是

A.洋务运动受到英国近代化进程的影响 B.“0”号机车可能以内燃机作为动力

C.“0”号机车是中国近代交通发展的见证 D.从社会发展角度看,这是近代中国的进步

14.英国是老牌的殖民国家之一,曾在世界各大洲建立了殖民地,号称“日不落帝国”。英国成为世

界上最强大的殖民国家是在

A.17世纪中期

B.18世纪初

C.18世纪后半期

D.19世纪末期

15.有一段史料记载:“从1816年以后十年间,他率领起义军英勇作战,由北向南,横扫南美大陆上

的西班牙军队,解放了西班牙在南美洲的殖民地。”这里的“他”是指

A.华盛顿

B.玻利瓦尔

C.圣马丁

D.史蒂芬孙

16.1836~1848年间,英国工人掀起了一场规模宏大.持续时间长的运动。这次运动的斗争目标是

A.反对殖民压迫

B.改善劳动和生活条件 C.要求取得普选权

D.建立自己的政权 17.之所以说南北战争是美国历史上的

三、材料解析题(本大题2小题。每小题10分,共20分) 25.阅读下列材料:

材料一:如图

材料二:他(伏尔泰)‚把教皇比作‘两只脚的禽兽’,

把教士比作‘文明恶棍’,还说天主教就是

‘一群狡猾的人布置的一个最可耻的骗人的

罗网’。‛

──摘引自《世界历史》九年级上册

材料三:路易十六在狱中阅读了启蒙思想家的著作,

感叹道:‚原来是伏尔泰和卢梭毁了法国!‛

请回答:

法国社会等级制度

(1)根据材料一中反映的内容,请你描绘出18世纪法国社会等级状况?(4分)

(2)材料二反映出伏尔泰的思想立场是什么?(2分)

(3)结合以上材料和所学知识,说说为什么路易十六会发出这样的感叹?(4分)

26.阅读下列材料:

材料一:明治政府兴建工厂,修筑铁路,举办邮政、电讯事

业,扶植、保护私人企业的发展。为‚求知识于世

界‛,日本还请来大批欧美专家和技师。

材料二:(明治政府)规定了统一的学制;仿照欧美,设立了

新式学校,普及初等教育。政府提倡学习欧美的资

本主义文明,盖洋楼,吃洋餐,穿西服,改变传统

的日本发型。 材料三:如图

──以上材料均摘引自《世界历史》九年级上册 请回答:

(1)以上材料一和材料二是关于明治维新内容的介绍。请你用自己的语言,归纳概括出材料主要内 容。(6分)

(2)以上三则材料中,哪些内容充分体现了当时世界科技发展的领先成就?(2分)

(3)通过以上材料,你认为明治维新这段历史给我们的启示应当是什么?(2分)

四、问答题(本大题2小题。其中

(1)你能介绍一些有关这首歌曲诞生的背景知识吗?(答案要求包括它创作的历史背景、词曲作者) (4分)

(2)你能否解释一下,歌词中“奴隶”是指哪个阶级?歌词中的“旧世界”代表的是什么制度? (4分)

(3)联系历史与现实,请告诉我们,你是如何理解“要为真理而斗争!”这句歌词的?(2分)

28.战争的发生,总是因为经济、政治、军事、民族等各种矛盾的激化而引发的。一位美国历史学家

在谈到一战前的欧洲形势时就说过:“欧洲变成一只‘火药桶’,只等一粒火星将它引爆。” 请回答:

(1)根据一战前的欧洲形势,请你说说,欧洲是怎样逐步变成一只“火药桶”的?(8分)

(2)最后引爆“火药桶”的“火星”是什么?(2分)

(3)一战对欧洲的政治格局产生了哪些深远影响?(2分)

五、活动与探究(本大题1小题,共10分)

29.在主题为“世界联成一体”的历史活动课上,一个小组的同学向其它同学展示了四幅选自历史教

科书上的图片,他们提出挑战,请其他组同学派出一个代表,上台回答他们的问题。如果你也是其中一员,你能顺利完成这场挑战吗?

图1 哥伦布

图2 克伦威尔

图3 瓦特

图4 林肯

(1)首先,他们要求同学写出以上四幅图片内容反映的重大历史事件名称?(4分)

图1:__________________________________________________________________________ 图2:__________________________________________________________________________ 图3:__________________________________________________________________________ 图4:__________________________________________________________________________ (2)接着,他们要求上台的代表任选一幅图,然后说出图中人物在历史上所起的作用。你会选哪一

幅呢?(2分)(请注明你选择的人物,再作答)

(3)最后,他们请每个组的代表说出这些图片反映的历史事件之间的联系。请你也按以下提示,选

出其中一组进行简要说明。(提示:图1与图2;图2与图3;图3与图4)(4分)

装订线内不要答题

第12篇:九年级第一学期二、三单元测试卷

——最专业的中小学教学资源共享平台

人教版2011-2011年度九年级政治

人的身智力、有文 ——最专业的中小学教学资源共享平台

8.下列关于我国当前各种经济成分的叙述,不正确的是

A.集体经济直接体现共同富裕的原则 B.公有制经济在我国经济中占主体地位

C.非公有制经济是社会主义经济制度的基础

D.国有经济是引导、推动经济和社会发展的主导力量

9.要实现共同富裕,最根本的途径是

A.加强教育,提高人们的思想境界

B.树立榜样,提倡过俭朴的生活

C.发展经济,提高人民生活水平

D.消除矛盾,让社会更加和谐

10.2011年1月14日,中共中央、国务院在北京隆重举行国家科技奖励大会。师昌绪、王振义获得国家最高科技奖金500万元。这500万元奖金属于

A.按劳动分配

B.按技术分配

C.按资本分配

D.按管理分配

11.中学生因社会阅历不够丰富,自己选择商品的能力还较弱,在消费时更应该多一些理性,少一些盲目。下列行为不符合这一要求的是

A.消费前作出预算,把钱花在关键的地方

B.在可承受的范围内购买经济实用的商品 C.经常购买名牌服装

D.勤俭节约,更注重精神上的需求

12、下列能直接体现现阶段我国社会主要矛盾的有

①中国救援队赴日参与灾后救援 ②两会时,住房、医改、物价等问题仍然引起百姓的极大关注 ③我国经济建设与环境保护之间的矛盾 ④中考后只有一部分学生进入重点中学读书 A.①②

B.②④

C.①③

D.②③

非选择题56分

13、【家庭一幕】有一天,小明一家三口边散步边聊天……小明:“爸,你经常忘记切断计算机的电源,水龙头的水流又开得很大,真浪费。”爸爸:“地大物博的大中国,一点点水电,何足挂齿!再说我们又不是没出钱。”妈妈和小明听了齐声说:“反对!……“ 请你至少从二个方面帮母子俩反驳爸爸(4分)

【社会一隅】低碳生活,是一种时尚。株洲公共自行车租赁系统自2011年5月正式启动以来, 502个已施工的公共自行车租赁点中,目前已有474个开通并投入使用,共投放自行车8000辆。已有超过300万人次使用,单日使用量最高突破8万人次。平均每辆车每天使用至少10次。每几百米设一个公共自行车亭,不少拥有轿车的市民也骑着公共自行车上街。 (1)材料反映了市民什么意识在提高?(2分)

(2)践行低碳生活有什么重要意义?(4分)

(3)我们还可选择哪些低碳生活方式?(4分)

14、(14分)材料一:2010年10月18日,胡锦涛总书记在党的十七届五中全会上提出,要认真落实国家中长期教育改革和发展规划纲要,落实促进教育公平、提高教育质量重大举措,办好人民满意的教育。12月10日,在中央经济工作会议上,胡锦涛总书记再次强调,全面落实国家中长期教育改革和发展规划纲新课标教学网(www.daodoc.com)--海量教学资源欢迎下载!

——最专业的中小学教学资源共享平台

要,重在促进教育公平、提高教育质量。

材料二:中国生产的DVD每台批发价为40美元,其中专利费却占到了21.3美元;中国是计算机的生产大国,但同时也在为微软及英特尔打工赚钱。在高新技术领域,美国、日本拥有的专利占世界专利总量的90%左右,而包括中国在内的其他国家仅仅占有10%左右。 (1)材料一体现了党和政府在落实什么战略?(2分)

(2)“强国必先强教”,说明我国把教育搞上去的决心,请问这样做有何意义?(4分)

(3)科技创新能力已经成为综合国力竞争的决定性因素。对此你是怎样理解的?(4分)

(4)在大力提高我国的科技自主创新能力的今天对我们中学生提出了怎样的要求?(4分)

15..

(图二为2011年6月20日,江西瑞金市沙洲坝村选区人大代表候选人与村民见面会会场。) 结合上图,运用所学知识,思考下列问题

(1)图

一、图二说明我国坚持什么政治制度?(2分)

(2)这一政治制度在我国政治体制中具有什么地位?(2分)

(3)结合图一谈谈全国人大在我国国家机构中居于什么地位,享有哪些职权?(4分)

新课标教学网(www.daodoc.com)--海量教学资源欢迎下载!

图一

图一 图二

图二 ——最专业的中小学教学资源共享平台

(4)请你结合图二简述人民和人大代表的关系。(2分)

16.中共中央总书记胡锦涛在庆祝中国共产党成立90周年大会上发表讲话时指出,要全面落实依法治国基本方略,在全社会大力弘扬社会主义法治精神,不断推进科学立法、严格执法、公正司法、全民守法进程,实现国家各项工作法治化。

结合上述材料,运用所学知识,思考下列问题:

(1)我国为什么要坚持依法治国基本方略? (一点原因即可。2分)

(2)全面落实依法治国基本方略有何意义? (两个方面即可。4分)

(3)落实依法治国基本方略,弘扬社会主义法治精神,中学生该怎么做?(两个方面即可。4分)

17.2011年9月14日,温家宝总理在

——最专业的中小学教学资源共享平台

(2)践行低碳生活,有利于落实保护环境和节约资源的基本国策(或:有利于增强人们保护环境、节约资源的意识。或:有利于提高人们的节能减排意识);有利于全面贯彻落实科学发展观(或:有利于促进人与自然和谐相处);有利于实施可持续发展战略(或:有利于走可持续发展道路):有利于建设资源节约型和环境友好型社会;有利于在全社会倡导文明健康的生活方式:等等。

(3)用节能灯代替白炽灯;尽量不使用电梯,多走楼梯,多步行,多做公交车,少开私家车;减少看电视的时间

14、(1)科教兴国和人才强国战略。

(2)只有把教育搞上去,才能化人口大国为人才强国,化人口压力为才优势,才能从根本上提高中华民族的整体素质,增强我国的综合国力,才能在激烈的国际竞争中取得战略主动地位。

(3)①当今世界,科学技术日益渗透到经济发展,社会进步和人类生活的各个领域,科学技术成为生产力中最活跃的因素。科学技术是

——最专业的中小学教学资源共享平台

新课标教学网(www.daodoc.com)--海量教学资源欢迎下载!

第13篇:吴中区学年第一学期期末调研测试卷 七年级语文

七年级期末教学质量调研测试

语文

一、基础知识(26分)

1.根据拼音写出汉字。(4分)

①安xiáng()②jí()取③轻飞màn()舞④一视同rén()

2.找出下列语段中的四个错别字并加以改正。(4分)

现代人对狼的厌恶之情似乎根深蒂故,狼心狗肺、狼狈为奸、豺狼当道、狼子野心…… 关于狼的成语令人眼花暸乱,但主要情感倾向却不言而谕。其实在古代,人们对狼充满了崇敬。狼虽然常独自活动,但攻击比自己强大的敌人时,总是同心协力,异常团结;攻击前,狼总要反复观察对手,做到知己知彼,充满智慧;狼不像狗一样容易被驯服,几千年来不曲不挠,始终保持着自己的本性。

3.默写古诗文名句,并在括号内的横线上填写相应的作家、篇名或词牌名(10分)①海日生残夜,_______________。(王湾《次北固山下》)

② _______________?门前流水尚能西,休将白发唱黄鸡。(苏轼《_____________》)③四面湖山归眼底,。(《湖南岳阳楼对联》)

④但愿人长久,_______________。(苏轼《水调歌头》)

⑤最是一年春好处,_______________。(韩愈《早春呈水部张十八员外》)⑥ _______________,听取蛙声一片。(_______________《西江月》)⑦子曰:“_______________,思而不学则怠。”(《论语》)。

⑧ _______________,随风直到夜郎西。(李白《闻王昌龄左迁龙标遥有此寄》)

4.依据课本上学到的作家作品,仿照填空前边的句式,再补写两个句子。(4分) 文学,是巴金对“繁星”的寄语,是安徒生对“皇帝的新装”的嘲讽,①, 情感的真实流露。

5.《汤姆·索亚历险记》是“批判现实主义”文学的重要作品,马克·吐温借引人入胜的儿童历险故事,批判了现实生活中的不合理现象,表达了对美好社会、美好人性的追求。请根据小说中的有关故事和人物,回答下列问题。(4分)

①故事的主人公汤姆身上不仅有追求自由、喜欢历险的勇敢,更有正直善良、乐于助人的责任心。请依据具体故事情节,对汤姆的后一种特点作简要说明。(2分)

②故事中最主要的反派人物叫什么?他的结局是怎样的?(2分)

二、阅读理解(34分)

阅读下面两段文言文,完成6—9题。(13分)

【甲】郑人有欲买履者,先自度其足,而置之其坐。至之市,而忘操之。已得履,乃曰: “吾忘持度!”反归取之。及反,市罢,遂不得履。人曰:“何不试之以足?”曰:“宁信度,无自信也。”

【乙】曩①有愚者,常于户外县②履为志③。一日出户,及午,忽暴雨。其妻收履。至薄暮,愚者归,不见履,讶曰:“吾家徙乎?”徘徊不进。妻见之,曰:“是汝家,何不入? ”愚者曰:“无履,非吾室。”妻曰:“汝何以不识吾?”愚者审视之,乃悟。

①曩(náng):从前。②县:同“悬”,挂。③志:标记。

6.下面各项加点字的意思相同的两项是()()(4分)

A. 先自度其足B.遂不得履 ..

五忘持度扒履其板(《梵天寺木塔》) ..

C.何不试之以足D.已得履,乃曰 ..

徐喷以烟(《幼时记趣》)愚者审视之,乃悟(选文【乙】) ..

E.至之市 .

春冬之时(《三峡》) .

7.【甲】文中“郑人”在别人提醒他可以用脚试鞋子后,还说“宁信度,无自信也”,说 明他是一个怎样的人?(2分)

8.我国古代有不少富含哲理的寓言故事,你还喜欢哪一个?说说喜欢的理由。(3分)

9.用现代汉语翻译下列句子。(4分)

①愚者归,不见履,讶曰:“吾家徙乎?”

②妻见之,曰:“是汝家,何不入?”

阅读屠格涅夫《蔚蓝的王国》选段,完成10—13题。(9分)

①我们轻快的小船在微波中飘荡,时起时伏。并不是风在推动它前进,驾驶它的是我 们自己无忧无虑的心灵——我们稍一动念,想到哪里去,小船就飘到哪里去,它像是有生 命的东西,完全听从我们使唤。

②我们看到一些岛屿,晶莹明澈的仙岛,岛上的璧玉宝石光艳夺目。——仙岛隆起的 岸上飘来醉人的芳香;一些岛上,像下雨似的朝我们身上飘洒白玫瑰花和铃兰花;另一些岛上忽然飞起一群五彩缤纷的长翼的鸟儿。

③鸟儿在我们头顶上回翔,铃兰花和玫瑰花同沿着平滑的船舷翻滚的珍珠似的浪花融 合在一起,分辨不清了。

④随同花儿和鸟儿一起飘来一阵甜滋滋的声音……其中仿佛有女人的声音……周 围的一切:天空、海洋、微微飘动的风帆、船尾潺潺的水流——一切都在诉说着爱情,诉 说着无比幸福的爱情!

⑤,她,我们每个人都爱着的那个人——她就在这儿……虽然看不见,但近在咫尺。 不消。片刻,她的眼睛就会闪出亮光,她的脸上就会现出笑容……她的手将会拉着你的手一一拉着你一起进入永不衰败的天堂!

⑥蔚蓝的王国呀!我看见过你……在梦里。

10.作者在选文第①节中说“我们稍一动念,想到哪里去,小船就飘到哪里去”,仔细阅 读选文,你认为作者想到哪里去,希望见到什么?(2分)

11.选文中,作者重点写到了一个“女人”。从第④节中,你觉得这个女人指什么?读了 第⑤节内容,你觉得这个女人还可能指什么?至少说出两种含义。(3分)

12.第⑥节为全文的结尾,表达了作者怎样的思想感情?(2分)

13.在翻译外国作品的时候,不同翻译者的译文语言会有差异。下边是另一位翻译者翻译 的第③节内容。试比较一下,说说你更喜欢课文还是下边的译文?为什么?(2分)

“鸟儿在我们头顶盘旋。铃兰和玫瑰消溶在珍珠般的泡沫之中,沿着我们光滑的船舷 漂流而去。”

阅读《我交给你们一个孩子》,完成下边l 4—18题(12分)

我交给你们一个孩子

张晓风

①小男孩走出大门,返身向四楼阳台上的我招手,说:

②“再见!”

③这是他开始上小学的第二天。

④我其实仍然可以像昨天一样,再陪他一次,但我还是狠下心来,看他自己单独去了。 他有属于他的一生,是我不能相陪的,母子一场,只能看作一把借来的琴弦,能弹多久, 便弹多久,但借来的毕竟是有其归还期限的。

⑤他欢然地走出长巷,很听话地既不跑也不跳,一副循规蹈矩的模样。我怔怔地望着 巷子下细细的朝阳而落泪。

⑥想大声地告诉全城市,今天早晨,我交给你们一个小男孩,他还不知恐惧为何物。我却是知道的,我开始恐惧自己有没有交错。

⑦我把他交给马路,我要他遵守规矩沿着人行道而行,但是,匆匆的路人啊,你们能够小心一点吗?不要撞倒了我的孩子。我把我的至爱交给了纵横的道路,容许我看见他平平安安地回来。

⑧我不曾搬迁户口,我们不要越区就读,我们让孩子读本区的国民小学而不是某些私立的明星小学,我努力去信任自己的教育当局,而且,是以自己的孩子为赌注来信任——,但是,学校啊,当我把我的孩子交给你,你保证给他怎样的教育?今天早晨,我交给你一个欢欣、诚实又颖悟的小男孩,多年以后,你将还给我一个怎样的青年?

⑨他开始识字,开始读书,当然他也要读报纸、听音乐或看电视、电影,古往今来的撰述者啊,我的孩子会因为你们得到什么呢?你们将饮之以琼浆,灌之以醍醐,还是哺之以糟粕?他会因而变得正直、忠信,还是学会奸猾、诡诈?

⑩世界啊,今天早晨,我,一个母亲,向你交出了她可爱的小男孩,而你们将还给我一个怎样的人呢?

14.作者文中说,孩子“还不知恐惧为何物,我却是知道的\"。请你用简明的话概括作者“恐惧”些什么。(3分)

15.作者既然有那么多的“恐惧”,为什么还要把孩子交给“世界”?(2分)

16.第⑨节中的“醍醐”两个字你可能不识,但是,你能否根据前后文推断一下,“醍醐”和“琼浆”的意思相近呢,还是和“糟粕”的意思相近?它在文中具体指什么?(2分)

17.第⑤节划线句包含着作者怎样的情感?(2分)

18.联系自己的生活体会,说说孩子的成长环境中还有哪些令人担心的事。(3分)

三、作文(40分)

19.生命是一趟旅行,不能错过沿途的风景;人生是一场历练,在喜怒哀乐中走向成熟;生活是一种领悟,懂得收藏领悟才会有滋有味。你进入中学已经一个学期了,今日回首,你会收藏什么?

请你以《学期回眸》为题,写一篇记叙文,记述本学期中给你印象特别深的往事,表达自己对初中生活或青春岁月的思考和领悟,不少于500字。文中不要出现(或暗示)本人的姓名、校名。

第14篇:安庆市学年度第一学期期末教学质量调研检测九年级英语试题

开场白

许多学生认为数学是枯燥的、乏味的。一些非数学老师在听完一堂数学课后,往往这样评价:思路清晰、语言精练、解题严谨,就是太乏味、缺少趣味性,让人昏昏欲睡。那么,如何调动学生的上课积极性,引发他们的好奇心?设计好“开场白”,非常关键。下面是数学课的几个片断:

动手实验式“开场白”:桌上摆满了切成各种形状的萝卜,大伙好像还在热列地讨论着什么。老师微笑问:“同学们,用一个平面去截一个正方体,截出的面可能是什么形状?”

悬念式“开场白”:老师一上讲台,故意神神秘秘地说“你们每人随便想一个自然数,将这个数乘5减7,再把结果乘2加14”。……“你们算得的结果个位数字一定是0”。顿时教室里象炸了锅似的,……“等你学了字母表示数,你也会算了”。……

故事式“开场白”:为了让学生体会图形的边长、周长、面积在变化过程中的关系,领会列方程解应用题时,关键是捕捉到不变的量。老师先给学生讲了一个故事:父亲的羊越来越多,想拆旧羊圈扩大面积,可是没有多余的篱笆,怎么办呢?他叫来了儿子,儿子不慌不忙地说:“爸,我有办法”。“你看,旧羊圈长70米,宽30米,面积2100平方米。如果改成50米见方的新羊圈,不用添篱笆,羊圈面积就有2500平方米”。

诸如此类的还有:“贴近生活式”开场白;“设疑式”开场白;“名言式”开场白;“趣味式”开场白;“实例式”开场白;“比喻式”开场白等等。向学生提出恰当的问题,激发起学生的兴趣,提高他们学习的积极性。

第15篇:学年第一学期期末调研测试题九年级语文

2012-2013学年第一学期期末调研测试题

九年级语文

说明:1.全卷共8页,满分120分,附加题可以计入总分,但总分不得超过120分。2 考试用时为120分钟。:

一、基础(25分)

1.根据课文默写古诗文。(10分)

(l)口口口,口口口,此后汉所以倾颓也。(诸葛亮《出师表》)(1分)

(2)蒹葭苍苍,白露为霜。口口口口,口口口口。(《诗经·蒹葭》)(2分)

(3)影片中革命烈士们面对敌人的残酷迫害视死如归,看到同学们对此无法理解时,班主 任语重心长地说:“在当时的情况下,贪生怕死就意味着放弃革命大义,所以烈士们践行了 孟子

(4)军听了军愁,民听了民怕,口口口口口口口口口!(王磐《朝天子》)(1分)

(5)把辛弃疾的《破阵子·为陈同甫赋壮词以寄之》下片默写完整。(4分)

马作的卢飞快,__________._____________,______________.______________!

2.根据拼音和语境写出相应的词语。(4分)

(1)我希望他们不再像我,又大家ge m6()起来。

(2)据说他当初品行不正,zao ta()钱。

(3)探索的过程不是消极的xiu shou pang guan(),而是有计划的探索。

(4) nao xiu cheng nll()的伏尔泰总会让位于心平气和的伏尔泰。

3.下列病句修改不正确的一项是((3分)

A.由于运用了科学的复习方法,他的学习效率有了很大改进。(改“改进”为“提高,,)

B.支气管炎、过敏性鼻炎吃龙虾,会导致病情加重。(在“过敏性鼻炎”后加“患者”)c.自去年9月钓鱼岛附近撞船激发了两国主权纠缠后,中国宣布将把钓鱼岛附近海域 的巡逻常态化。(改“激发”为“引发”)

D.随着秦兵马俑在沉寂24年后的再次发掘,引起国内外媒体的关注。(删掉“随着”)

4.从下列A组词语中至少选用两个,并从B组关联词语中任选一组写一段连贯的话,描写 某人的神情或动作。(80字之内)(4分)

A组:煞白嬉闹忐忑茶饭无心唯唯连声对答如流

B组:不但„„而且„„之所以„„是因为„„

________________________________________________

__________________________________________________

5.请根据材料

一、二拟出30字以内的新闻,并根据材料三分析产生的主要原因。(4分)材料一:近日,央视曝光了山东多个地方的养鸡场,对白羽鸡喂食抗生素或抗病毒药物。 这种肉鸡生长速度快、一般40天左右出栏,平均体重5.5斤。有的甚至给鸡喂食国家明令 禁用的激素类药物。而这些抗生素鸡在没有经过检验捡疫的情况下,配送给肯德基门店。材料二:愈演愈烈的“速生鸡”问题,让“洋快餐”肯德基正面临一轮新的信任危机。 一直以来,肯德基始终以健康安全的形象自我标榜,但面对媒体的一再追问,肯德基的态度却始终暧昧不清。这已经不仅仅是一个简单的道德问题,更涉嫌违法。

材料三:今年8月底,长沙市卫生局在抽检的肯德基香辣鸡腿堡中,发现大肠菌群含量超标的现象,远高于国家规定的糕点类食品标准。但肯德基方面坚持认为,检测汉堡的标准不对,汉堡不是面包,不能以面包糕点的标准来衡量,要求卫生局重新考量。考虑到肯德基身份特殊、品牌影响力巨大,长沙市卫生局不得不将其临时撤出了不合格名单。

二、阅读(45分)

阅读下面文段,完成第6-18题。

(一)生于忧患死于安乐(12分)

舜发于吠亩之中,傅说举于版筑之间,胶鬲举于鱼盐之中,管夷吾举于士,孙叔敖举于 海,百里奚举于市。

故天将降大任于是人也,必先苦其心志,劳其筋骨,饿其体肤,空乏其身,行拂乱其所 为,所以动心忍性,曾益其所不能。

人恒过然后能改,困于心衡于虑而后作,征于色发于声而后喻。入则无法家拂士,出则 无敌国外患者,国恒亡。然后知生于忧患,而死于安乐也。

6-下而加点词语的解释有误的一项是()(3分)

A.必先苦其心忐苦:使„痛苦B.征于色发十声而后喻喻:明白,知晓。c.人恒过然后能改恒:偶尔,有时D..出则无敌国外患者出:在国外

7.用现代汉语翻译文中划线的句子。(3分)

8上天让人才在担当大仟前历尽磨难是为了“口口口口,口口口口口口”,

开头六位名人的史例是为了直接印证“口口口口”的观点。

9.根据文意,下面的理解不正确的一项是()(3分)

A.开头列举六位历史名人的事例形成排比,为下文的进一步分析提供了充分的依据。

B.第二段的6个“其”字表明了“上天”为了磨练、锻造可以担当大任的一代圣君或一代贤臣的良苦用心。

c.第三段从正反两方面论证经受艰苦磨炼的必要性,说明人处困境能激发斗志,而处于安逸之中则使人丧失斗志。

D.材料的详略安排取决于表达中心的需要。本文详细论述了“生于忧患”而简略阐述“死于安乐”,可见作者极力主张生活的简朴而鄙弃安逸享乐的人。

(二)人和机会(10分)

郭恢扬

①弱者待候机会,而强者创造机会。

②怯懦动摇者常常用没有机会来原谅自己。其实,生活中到处充满着机会!学校的每一门课程,报纸的每一篇文章,每一个客人,每一次演说,每一项贸易,全都是机会。这机会带来教养,带来勇敢,培养品德,制造朋友。对你的能力和荣誉的每一次考验都是宝的机会。如果像道格勒斯这样的奴隶都能使自己成为演说家、著作家和政治家,那么,我们应该怎么办?要知道,道格勒斯连身体都不属于自己!

③没有谁,在他的一生中,

④年轻的医生经过长期的学习和研究,他碰到了第一次复杂的手术。主治大夫不在,时 间又非常紧迫,病人处在生死关头。他能否经得起考验?他能否代替主治大夫的位置和他的 工作?机会和他面面相对。他是否能够否定自己的无能和怯懦,走上幸福和荣誉的道路?这 就要他自己做出回答了。

⑤对重大的机会你做过准备吗?

⑥除非你做好准备,否则,机会只会使你显得可笑。

⑦“从这条路走过去可能吗?”拿破仑问那些被派去探测“死亡之路”的工程技术人员。“也许吧。”回答是不敢肯定的,“它在可能的边缘上。那么,前进!”拿破仑不理会工程人员讲的困难,下了决心。

③出发前,所有的士兵和装备都经过严格细心地检查。走破的鞋、穿破的衣服、坏了的 武器,都马上修补或调换。一切就绪,然后部队才前进。统帅的精神鼓舞着战士们。战士刺 刀的闪光,出现在阿尔卑斯山高高的陡壁上,出现在高山的云雾中。每当部队遇到特殊困难 的时候,雄壮的冲锋号就会响彻群山之巅。尽管在这些危险的攀登中到处充满了障碍,致使

部队行程延长到30公里,但是他们一点不乱,也没有一个人掉队1 4天之后,这支部队就 突然出现在意大利平原上了。

⑨当这“不可能”的事情完成以后,其他人才看到,这件事其实是早就可以做到的。许 多统帅都具有必要的设备、工具和强壮的士兵,但他们缺少毅力和决心。拿破仑不怕困难, 在前进中聪明地抓住了自己的命运。

⑩机会,也许就在你的面前。

(节选自《阅读与鉴赏》2006年第21期)

10.下列理解符合原文意思的一项是()(3分)

A.开篇用对比的方法突出了不同的人与机会的关系,统领全文,发人深省。

B.道格勒斯的史例从正面论证了“强者把握机会”的观点,年轻医生的事例则从方面阐 明了机会往往与没有准备的人擦肩而过的观点。

C.拿破仑取得胜利的原冈在于他有必要的设备、工具和强壮的士兵,还有超出常人的毅 力和决心。

D.本文紧扣论题,采用了总分总式的论证方式,逐层深入地阐述了观点,结尾催人警醒。

11.本文依次着重阐述了哪两个观点?(4分)

12.请结合上下文说说文中划线句子的含义。(3分)

(三)有本事,你也撕(1 5分)

马德

(1)我高中时候,有个数学老师,姓王。

(2)他上课,常常讲不到十分钟时间,就会有细小的白沫子,挂在两嘴角边,像刷完牙忘记了擦一样,很扎眼。有捣蛋的学生,给他起了个外号,叫“王白沫”。那时候,他住在校里,经常自己做饭吃。晚自习辅导的时候,给我们讲题,常见他的手背上残留着面渍。每每这时候,女生就偷偷地笑,然后问他,老师,你才做过饭吧。他似乎意识到了什么,就赶紧把那双刚刚和过面的手,藏起来。

(3)印象中,开始他还显得很窘迫,后来,再遇到类似情况,笑笑,就过去了。

(4)王老师的数学课上得不错,纵横捭阖的,好多题,都给我们讲好几种解法。他说,他有一个同学,数学很厉害,上高中的时候,老师讲一章,他撕一章的书,老师讲完了,书也撕完了。而且,他的这个同学数学精通到什么程度呢,据说,选择题,不用算,只看选项, 就知道哪一个是答案。总之,他说得神乎其神。每次讲到这个故事的时候,他都神采飞扬, 嘴角的白沫也愈发地多,仿佛那个人不是他的同学,就是他自己。

(5)我们有一个数学顶好的同学,很狂妄。王老师有些看不惯他,就说,你要哪一天也能一边学一边撕书,我就服你。后来,这个故事变成了他的一句口头禅:有本事,你也撕!有事没事的,他都用这句话来鞭策我们。

(6)我那时候数学很差。上他的课,总不敢看黑板。尤其是他提问的时候,更是把头扎得低低的,生怕他问到自己。即便这样,有一次,他还是问到了我。他喊我名字的那一刻,我吓得仿佛在黑黢黢的夜里,被魔鬼一把抓住。我脸色苍白,哆哆嗦嗦站起来。王老师见我这样,很严厉地批评我,说,怕什么,谁也不是天生就什么都会的!不会答不可怕,可怕的是不敢答。我的同学为什么敢撕书,那是一种胆量和勇气。我教你们,希望你们也能像他一样,在学习上,有这么一股子闯劲和霸气。

(7)那节课,他在台上唾沫星子飞舞,我们在下边大气也不敢出。下课后,同学们都埋怨我,说因为我而挨了批。

(8)我有一个同学叫李军,在一次考试结束之后,到办公室,他见办公室无人,就从王老师的办公桌上,找到自己的数学卷,把第二卷的分数改了,把“61”,分改成了“87”分。然而,改完之后,从办公室跑出来没多久,他傻眼了,因为那次考试,第二卷满分才85分。

(IO)李军吓得惶惶不可终日,他不知道接下来会发生什么。

(10)第二天发卷的时候,李军假装肚子疼,没敢上课去,他怕王老师会当着全班的面,揭发他的“罪恶”行径。然而,卷子发下来,王老师什么也没说。李军的卷子上,在被他改过的“87”分上面,轻轻地画了两个斜杠,然后,在旁边,重新写上了61分。难道老师没有发现?可是,没有发现,他怎么会重新改过来呢?那么发现了,他怎么没有任何反应呢?

(11)这件事很蹊跷,即便是李军,也觉得不可思议。李军偷偷地和我们说,管他呢,这事,就算稀里糊涂过去了,逃过这一劫,以后,打死我也不干了,。

(12)毕业的那一年,我们班的数学考得很好。成绩下来的那一天,王老师高兴得喝了不少酒,他说,你们考得不错,虽然没撕书,但都算是有本事的学生。然后,他又在李军的肩膀上拍了一下,说,你本来也是最棒的嘛„„

(13)李军的脸“腾”一下子红了,他尴尬地等在那里,以为老师还要说什么,结果,王老师什么也没说,只是满脸灿烂的笑。

(14)那笑,被酒燃烧过,红红的额,让人一辈子难忘。

13.下列对于文章的理解不正确的一项是()

A、本文采用了典型的先抑后扬的写法,第(2)段以漫画式笔法交代了老师外号的来历和不休边幅的特点,文字简练而人物个性鲜活,为下文刻画其美好品质作好铺垫。

B、作者对王老师的感情很复杂,既有对其鄙视的外表的不屑,又有对其美好品行的赞许。

C、本文善于紧扣中心选择材料,并通过典型事件以小见大地反映人物的精神品质。不仅表现王老师高明的教育教学技能,更突出其言传身教的育人艺术。

D、文末对王老师外貌的刻画形神兼具,形象地表现了好老师对学生深远的影响。

14、请联系原文内容,简要说说王老师是一个怎样的形象?

______________________________________________________________________________

15、请联系上下文,品味文中划线的句子。

16、本文“有本事,你也撕”为题,有哪些好处?

(四)

夏洛克把我的本钱还我,放我去吧。

巴萨尼奥钱我已经预备好在这儿,你拿去吧。

鲍西亚他已经当庭拒绝了;我们现在只能给他公道,让他履行原约。

葛莱西安诺好一个但尼尔,一个再世的但尼尔!谢谢你,犹太人,你教会我说这句话。 夏洛克难道我单单拿回我的本钱都不成吗?

鲍西娅犹太人,除了冒着你自己生命的危险割下那一磅肉以外,你不能拿一个钱。 夏洛克好,那么魔鬼保佑他去享用吧!我不打这场官司了。

鲍西娅等一等,犹太人,法律上还有一点牵涉你。威尼斯的法律规定:凡是一个异邦人企图用直接或间接手段,谋害任何公民,查明确有实据者,他的财产的半数应当归受害的一方所有,其余的半数没入公库,犯罪者的生命悉听公爵处置„„快快跪下来,请公爵开恩吧。

葛莱西安诺求公爵开恩,让你自己去寻死吧;可是你的财产现在充了公,一根绳子也买不起啦,所以还是要让公家破费把你吊死。

公爵让你瞧瞧我们基督徒的精神,你虽然没有向我开口,我自动饶恕了你的死罪。你的财产一半划归安东尼奥,还有一半没入公库;要是你能够诚心悔过,也许还可以减处你一笔较轻的罚款„„

夏洛克不,把我的生命连着财产一起拿了去吧,我不要你们的宽恕。你们拿掉了支撑房子

的柱子,就是拆了我的房子;你们夺去我养家活命的根本,就是活活要了我的命。 鲍西娅安东尼奥,你能不能够给他一点慈悲?

葛莱西安诺白送给他一根上吊的绳子吧;看在上帝的面上,不要给他别的东西! 安东尼奥要是殿下和堂上愿意从宽发落,免予没收他的财产的一半,我就十分满足了.„一可是还要有两个附带的条件:第一,他接受了这样的恩典,必须立刻改信基督教;第二,他必须当庭写下一张文契,声明他死了以后,他的全部财产传给他的女婿罗兰佐和他的女儿。

17.从选段和全书其他情节看,夏洛克为什么起先一定要置安东尼奥于死地?后来为什么一再让步直至放弃打官司和视若生命的财产?(4分)

18.选段属于断案的最后环节。联糸原著中整个断案过程,从鲍西亚和夏洛克两人的表现中, 你得到了哪些启发或感悟?(4分)

三、作文(50分)

19.根据下面题目和要求作文。

懂得欣赏使生活更加美好:日月星辰原来如此动人,山川大地竟然这么迷人。善于欣赏, 使自己充满活力,使被欣赏者萌生斗志,互相欣赏使人际关系更加和谐,使双赢局面应运而 生„„关于欣赏,你有什么难忘的经历?或者有什么独到的见解?或者有什么强烈的感悟?请自拟一个包含“欣赏”这个词语的标题,写一篇不少于500字的文章,文体不限,文 中不得出现真实姓名和学校名。

四、附加题(10分)

阅读下面文言文,回答后面问题。

王烈字彦方,太原人也。少师事陈寔,以义行称。乡里有盗牛者,主得之。盗请罪日: “刑戮是甘,乞不使王彦方知也。”烈闻而使人谢之,遗布一端。或问其故,烈日:“盗惧 吾闻其过,是有耻恶之心。既怀耻恶,必能改善,故以此激之。”

后有老父遗剑于路,行道一人见而守之,至暮,老父还,寻得剑,怪而问其姓名,以事 告烈。烈使推求,乃先盗牛者也。诸有争讼曲直,将质之于烈,或至途而返,或望庐而还。 其以德感人若此。

(选自南朝宋范晔(《后汉书·独行列传》)

1.解释下列句子中加点词语的意思。(3分)

(1)少师事陈寔,以义行称称:_______________

(2)烈闻而使人谢之,遗布一端遗:_____________

(3)烈使推求,乃先盗牛者也推:_______________

2.结合上下文翻译文中划线的句子。(2分)

3.为了表现王烈的可贵品质,本文用了哪些描写方法?(3分)

4.范晔记述本文的故事,想要表达怎样的思想情感?(2分)

第16篇:——学年度第一学期语文测试卷(一)

2018——2019学年度

摊主见我还在翻看,便跺着脚催问买不买。他急着要走,因为冷,因为要吃饭,因为到了散摊的正午时光。于是询价,掏钱,打包,在柔和清冷的阳光抚摸中,我把那摞枣红色书籍带回了家。

用白菜价格买来的那排枣红色,宛如钱塘江涌出的一线潮,整齐醒目地摆在书橱中央。然而,望着捡漏得来的书籍和由它们簇成的风景,我却怎么也高兴不起来。

文化和市场一接轨,就成为买卖文化的市场,一切与文化沾边的标识,比如青铜、陶瓷、字画、家具、钱币、刺绣、木雕等等,都贴着文化的标签,以古董的身份在不同的位置出现和交易。至于真假,新旧,年代,那就考验淘宝者的眼力、胆识和运气。书,自然是文化的,可是,在这交易文化的市场里,除了古籍和稀有版本受到收藏者淘宝者的青睐外,其他都白菜萝卜般地拥挤在地摊上。一堆一堆任人翻,任人找,任人看的书,在这里成为真正的落魄者和流浪者,既失去了“书中自有黄金屋”和“书中自有颜如玉”的高贵价值,也失去了可以归属的身份证。它们的高贵去哪儿了呢?

寻找被遗失被冷落的那些高贵书籍,常成为我周末的主题。 在不同城市的文化市场和相同模样的地摊书摊上,我寻找到许多高贵的书籍,如四卷本《柳宗元集》,四卷本《乐府诗集》,三卷本《白居易集》等;八十年代初影印的《详解聊斋志异图咏》让我高兴了许多个夜晚,书中那些绘于光绪十二年的上百幅精致插图,是那样惟妙惟肖,清晰可人。稀有的图画,让蒲松龄笔下的聂小倩、崂山道士、席方平都有了可以直视的生命画像。还有四十年代末、五十年代初标着金圆券价格的鲁迅的、巴金的、吕叔湘的书籍;还有六十年代山东师范学院研究老舍、夏衍、郭沫若的书籍,在那些跳跃着细小草根显露的纸面上,尽管造纸的师傅想用绿色黄色淡青色来掩饰,依然如劳作者的皮肤那样粗糙。凹凸不平的汉字们倔强地排在那里,一言不发地留着自然灾害年代的苦涩和艰辛。上世纪七十年代出版的鲁迅着作单行本竟也在地摊上一一买齐。这些绝对正版,品相绝对在九品以上的不同思想的高贵者,各自在书橱的上上下下站位,摇曳着厚重如山的独有风采。

坚强人格让我敬仰,倔强书格让我生出无边的敬意。

淘来的书群既有卓然自尊的高贵身份,又有正版合法的身份证和居住证。书中一枚枚不同模样不同规格的印章,还有不同方式的图书编码,都载着岁月痕迹与书一同存在,成为书无法分离的命运共同体。这些后来附加在书中的手写体和深深浅浅的印泥,证明这些书们曾经驻留的基地或者居住的地址。有的书中还有硬硬的或白或黄的借书卡,卡上还有借书人书写的潦草姓名和时间。这一切都成为书的正统履历。

好玩和好奇心曾促使我按图索骥,为书籍寻找曾经的家门,结果出奇一致,你寻找的单位不存在。 书大概在破产、改组、或者拍卖过程中,被迫卖掉的,或者任意让人拿走的。我见过无数书们被秤钩子称过以后,被塞进麻袋的狼狈和无奈。书的高贵身份瞬间被秤钩子剥夺,从高高的书架沦为不值钱的“废品”,然后到地摊上去流浪。

名著,尽管流浪过,也是流浪者中的高贵者。

每次抚摸,阅读这些血统正宗、出身名门的流浪书,感觉面对的似乎已经不是简简单单的书本,而是一位位经过生命体验的长者。当手指划过,眼光走过,书籍也默默收藏起你的体温、感知和眼神,就像收藏每天的阳光那样,把发生的一切悄悄地融注在经过岁月沧桑洗礼的纸面里,成为你什么时候需要就什么时候出现的最好朋友与知己。

每一部书,特别是那些经过书摊地摊流浪后的书,都会成为一座有秘密有故事的房子,无论横版竖版精装简装简体繁体。书的一个钤印,一个刻本,一个年份,一行字迹,都在文化字面的脊梁上,留下烧不掉、丢不了、碾不碎、化不去的文化记忆。

(4)下列对作品相关内容和艺术特色的分析鉴赏,不正确的一项是(

) A.“蹲下。捡起。端看。”这三句描写了“我”发现那几本高贵的书时的动作,短促的句式表现了“我”内心的激动。

B.这几本名著已经出版了三十多年且已不成套,对于书贩来说已经没有太大的价值,难怪他显得那么的不耐烦并急于出手。

C.文中写了书的种种遭遇,比如“白菜萝卜般地拥挤在地摊上”“被秤钩子称过以后,被塞进麻袋”,隐含着作者的痛心。

D.本文记述了“我”发现散落的名著进而收集名著并寻找这些名著“身份”的经历,表达了对名著经久不衰的价值的崇敬之情。

(5)文章开头写出了环境的什么特点?这样写有什么作用?

(6)文中写到:“坚强人格让我敬仰,倔强书格让我生出无边的敬意。”结合全文,谈谈你对“倔强书格”的理解。

3.阅读下面的文字,完成小题。 材料一:

日前,中国科学院在京召开新闻发布会对外宣布,“墨子号”量子科学实验卫星提前并圆满实现全部既定科学目标,为我国在未来继续引领世界量子通信研究奠定了坚实的基础。

通信安全是国家信息安全和人类经济社会生活的基本要求。千百年来,人们对于通信安全的追求从未停止。然而,基于计算复杂性的传统加密技术,在原理上存在着被破译的可能性。随着数学和计算能力的不断提升,经典密码被破译的可能性与日俱增。中国科学技术大学潘建伟教授说:“通过量子通信可以解决这个问题。把量子物理与信息技术相结合,利用量子调控技术,用一种革命性的方式对信息进行编码、存储、传输和操纵,从而在确保信息安全、提高运算速度、提升测量精度等方面突破经典信息技术的瓶颈。”

量子通信主要研究内容包括量子密钥分发和量子隐形传态。量子密钥分发通过量子态的传输,使遥远两地用户可以共享无条件安全的密钥,利用该密钥对信息进行一次一密的严格加密。这是目前人类唯一已知的不可窃听。不可破译的无条件安全的通信方式。量子通信的另一重要内容量子隐形传态,是利用量子纠缠特性,将物质的未知量子态精确传送到遥远地点,而不用传送物质本身,通过隐形传输实现信息传递。

(摘编自吴月辉《“墨子号”,抢占量子科技创新制高点》,《人民日报》2017年8月10日)

材料二:

潘建伟的导师安东•蔡格林说,潘建伟的团队在量子互联网的发展方面冲到了领先地位。量子互联网是由卫星和地面设备构成的能够在全球范围分享量子信息的网络。这将使不可破解的全球加密通信成为可能,同时也使我们可以开展一些新的控制远距离量子联系的实验。目前,潘建伟的团队计划发射

潘建伟是一个有无穷热情的乐观主义者。他低调地表达了自己的信心,称中国政府将会支持下一个宏伟计划——一项投资20亿美元的量子通信、量子计量和量子计算的五年计划,与此形成对照的是欧洲2016年宣布的旗舰项目,投资额为12亿美元。

(摘编自伊丽莎白•吉布尼《一位把量子通信带到太空又带回地球的物理学家》,《自然》2017年12月) 材料三:

日本《读卖新闻》5月2日报道:中国实验设施瞄准一流(记者:莳田一彦,船越翔)

在中国南部广东省东莞市郊外的丘陵地带,中国刚刚建成了大型实验设施“中国散裂中子源”。该实验设施建设费用达到23亿元人民币,3月正式投入运行。中国是继美国、英国、日本之后

四、诗歌鉴赏。

野 歌 ·李 贺

鸦翎羽箭山桑弓,仰天射落衔芦鸿。麻衣黑肥冲北风,带酒日晚歌田中。 男儿屈穷心不穷,枯荣不等嗔天公。寒风又变为春柳,条条看即烟濛濛。

(14)下列对这首诗的赏析,不正确的一项是(

A.弯弓射鸿,麻衣冲锋、饮酒高歌都是诗人排解心头苦闷与抑郁的方式。

B.诗人虽不得不接受生活贫穷的命运,但意志并未消沉,气概仍然豪迈。

C.诗中形容春柳的方式与韩愈《早春呈水部张十八员外》相同,较为常见。

D.本诗前半描写场景,后半感事抒怀,描写与抒情紧密关联,脉络清晰。 (15)诗中最后两句有何含意?请简要分析。

五、默写。

(16)刘禹锡《陋室铭》中体现陋室环境清幽,突出景色之雅的句子是:________,________。 (17)《荀子·劝学》中螃蟹虽“六跪而二螯”,但是用心浮躁导致“________”的状况。 (18)曹操慨叹人生短暂“譬如朝露”,苏轼在《赤壁赋》中以“________,________”表达了相近的意思。

六、综合题。

戏曲既需传承也需创新,这是业内的基本共识。然而,近年来由于一些创新尝试未收到理想效果,有人就讲创新和继承对立开来,认为戏曲不必创新。尤其是昆曲等戏曲艺术进入世界非物质文化遗产名录之后,创新在某些人那里几乎成了贬义词。(____________)

随着时代的发展变化,戏曲艺术不断被赋予新的内涵。如果一直固守原有形态,只强调复制和模仿,戏曲恐怕早在数百年前就____________了。突破前人、大胆创新,这是各个时代取得伟大成就的艺术家的共性,诚如某戏刷评论家所言,没有一位____________的京剧名伶是靠模仿或重复而成就自己的。京剧大师梅兰芳,以坚定的信念和博大的胸怀为京剧改革作出巨大贡献。他眼界开阔,________,除唱腔、表演技巧之外,还从化妆、灯光、服装、舞蹈、剧目创作等多个方面进行了大量的探索,可谓“剧剧有创新,剧剧有新腔”。尚小云、荀慧生,于连泉等人,也是因为具有超越前人的理想和切实的努力,不满足于停留在雷池之内_________,才能够在强大的保守情堵的笼罩下突破藩篱,从而成为新流派的创始人。当然,戏曲的创新必须以传承为基础,是传承中的创新,而不是眼花缭乱甚至任性妄为的创新,才能探索出一条能够被大多数观众接受的创新之路来。

(19)下列在文中括号内横线上补写的语句,最恰当的一项是(

) A.当代戏曲的发展,被创新精神的缺失所制约

B.当代戏曲的发展,因创新精神的缺失而被制约

C.创新精神的缺失,制约了当代戏曲的发展

D.创新精神的缺失,对当代戏曲发展起了制约作用

(20)依次填入文中横线上的成语,全都恰当的一项是(

) A.寿终正寝 名噪一时 兼容并蓄 照猫画虎

B.无疾而终 名垂青史 兼容并蓄 按图索骥

C.寿终正寝

名垂青史

博采众长

照猫画虎

D.无疾而终

名噪一时

博采众长

按图索骥

(21)文中画横线的部分有语病,下列修改最恰当的一项是(

A.而不是眼花缭乱甚至任性妄为的创新,这样才能探索出一条能够被大多数观众接受的创新之路来。

B.而不是令人眼花缭乱甚至任性妄为的创新,这样才能探索出一条能够被大多数观众接受的创新之路来。

C.而不是令人眼花缭乱甚至任性妄为的创新,才能探索出一条能够被大多数观众接受的创新之路来。

D.而不是眼花缭乱甚至任性妄为的创新,这样我们才能探索出一条能够被大多数观众接受的创新之路来。

七、语言表达。

22.下面是某校中学生暑假社会实践活动计划的初步构思框架,请把这个构思写成一段话,要求内容完整,表述准确,语言连贯,不超过85个字。

23.下面是某校一则启事初稿的片段,其中有五处不合书面语体的要求,请找出并作修改。

我校学生宿舍下水道时常堵住。后勤处认真调查了原因,发现管子陈旧,需要换掉。学校打算7月15日开始施工。施工期间正遇上暑假,为安全起见,请全体学生暑假期间不要在校住宿。望大家配合。

八、写作题。

①《送东阳马生序》中有言:“同舍生皆被绮绣,戴朱缨宝饰之帽,腰白玉之环,左佩刀,右备容臭,烨然若神人;余则缊袍敝衣处其间,略无慕艳意,以中有足乐者,不知口体之奉不若人也。” ②据《中国青年报》报道:高考以后,准大学生掀起了整容热,整容项目主要集中在五官上,如割双眼皮、隆鼻、改脸型等。去做手术的极少是为“修残补缺”,更多的是为“锦上添花”。面对整容,学生说,整容是为了漂亮,上大学后有一个全新的形象。 ③美国《福布斯》杂志这样介绍马云:“这个长相怪异的人有拿破仑一般的身材,深凹的面颊,扭曲的头发,淘气的露齿笑,一个5英尺高、100磅重的顽童模样。”

要求:综合材料内容及含意,选好角度,确定立意;明确文体,自拟标题;不要套作,不得抄袭;不少于800字。

第17篇:高二第一学期语文期末测试卷

语文试题

(时间150分钟,满分150分)

注意事项:

1.答卷前,考生务必用黑色字迹的钢笔或签字笔将自己的姓名和考生号、试室号、座位号填写在答题卷的相应位置上。

2.考生务必用黑色字迹的钢笔或签字笔作答,答案不能答在试卷上,必须写在答题卷的各题目指定区域内相应位置上;如需改动,先划掉原来的答案,然后再写上新的答案;不准使用铅笔和涂改液。

一、现代文阅读(9分,每小题3分)

阅读下面的文字,完成1—3题。

戏曲,是中国人最典型的传统审美方式。

中国戏曲雅俗共赏,选材宽泛,自古以来,既有帝王将相的阳春白雪,也有寻常百姓的家长里短;既登得了庙堂,又入得了街巷。中国戏曲深蕴传统文化的精髓,舞台之上,一桌二椅,唱念做打,节奏唱腔,全凭演员高度程式化的表演营造具体的戏剧情境和表现鲜活人物。而中国戏曲本质上是写意的:三五步走遍天下,六七人千军万马。这与整个中国的传统艺术,如文学、绘画、书法、诗歌等,都一脉相通,代表着中国人独特的审美特质。在很长一段时间里,戏曲一直都在中国人的社会生活和审美活动中占据着最为重要的地位。 中国戏曲历经千年流传,“通变”早已融入自身血液之中,善于继承传统,也很擅长吸收和创新。戏曲代表中国人传统的审美,沉淀了中国人千年的心灵世界,而地方戏更是凝聚了千姿百态的地域文化。在全球化大趋势下,文化的多样性需要民族文艺的坚守和创新。戏曲在当代的发展,就具有了在全球化背景下保持民族特性、弘扬民族艺术不可取代的价值。因此,戏曲艺术是传统的、独特的,更是面向未来的。

戏曲是古老而又面向未来的艺术,保存个性和特性的传承,扬长避短、顺应时代要求的创新是其生命力所在。这就如同书法艺术需要发展创新,但不管如何创造,终归是点画线条的艺术,是汉字的艺术。中国戏曲整体的写意性、程式性要保留,切忌盲目采用现代舞台声光电技术“大制作”,破坏戏曲的本质。地方剧种的个性和风格也只能在创新中得到强化。 戏曲要走向未来,剧本是基础。曹禺大师早就说过,“繁荣的关键在于创作”,“剧本的生命在于演出”。目前戏曲事业发展遇到了一些瓶颈,“剧本荒”尤为突出。没有纯净坦荡、自由放飞的心态是写不出好剧本的。剧作家要静下心来,独立思考,有感而发,才能创作出形象生动、内涵深刻的剧本来。

戏曲的题材要创新。今人看古人的戏,看什么?传统戏曲应该开辟题材创作的新领域,关注现实题材的创作。题材可以是历史的,但创作理念和立意必须是现代的。站在现代人的立场上再现经典,适当取舍,挖掘经典中历久弥新的永恒主题,厚重的经典就能为现代人的生活提供源源不断的心灵慰藉。

戏曲的未来在年轻人,戏曲的观众是需要培养的,这是因为戏曲是高度艺术化的样式,“形式即内容”。现在的小孩子自小就看得懂动画片、电视剧,但让他去看戏就不一定能看懂。一是内容看不懂,二是形式看不懂,比如为什么要勾脸谱?水袖表达了些什么?戏曲有一定的欣赏门槛,观众要经历一个从看热闹到看门道的过程。因此,要创造机会让年轻人接触戏曲。现在时尚的东西太单薄,当年轻人腻烦了感官刺激,有了一定审美修养和内涵时,中国戏剧是最好的选择之一。

戏曲是创造性的艺术,是与观众当场双向交流的艺术。如果我们重视剧本创作,扬长避

短,好戏频出,老百姓怎么可能不喜欢看?新的观众不断成长,我们的戏曲怎么可能不繁荣?戏曲里有中国文化的根,是中国人最后的审美家园。

(摘编自 2010年10月22日《 人民日报 》)

1.下列有关“中国戏曲”的表述,不符合原文意思的一项是 ...

A.中国戏曲选材丰富,帝王将相、普通百姓都可以成为其表现内容。

B.中国戏曲的人物形象主要靠唱念做打等高度程式化的表演来体现。

C.中国戏曲与传统的中国文学、绘画、书法、诗歌在本质上都崇尚简约。

D.中国戏曲曾在中国人的生活和审美活动中占据着最为重要的地位。

2.下列理解,不符合原文意思的一项是 ...

A.继承传统就是要保留属于中国戏曲本身的东西,就是要弘扬独特的民族文化。

B.古老的戏曲艺术要想面向未来,既要保持自身的个性特点,也要善于创新。

C.戏曲创新可以使地方戏种的特色得到更好的发挥,从而赢得更多的年轻观众。

D.戏曲发展创新不能靠现代的声光电等舞台艺术,而应向传统的书法艺术学习。

3.下列对原文内容的理解和分析,正确的一项是

A.制约中国戏曲发展的主要问题是没有好剧本,好剧本的产生需要剧作家心无旁骛的创作,需要剧作家的独立思考。

B.看腻了动画片、电视剧的年轻人会对审美有更高的要求,而戏曲中的脸谱和水袖就可以满足他们的审美需求。

C.现在的年轻人喜欢时尚,而时尚的东西又都比较单薄缺少内涵,让年轻人接触戏剧就可以使他们变得厚重起来。

D.传统戏曲很少关注社会现实,这是观众不喜欢戏曲的根本原因。戏曲要进行题材创新,让现代观众看古人写的戏、看写古人的戏都是要不得的。

二、古代诗文阅读(36分)

(一)文言文阅读(19分)

阅读下面的文言文,完成4~7题。

王安石,字介甫,抚州临川人。父益,都官员外郎。安石少好读书,一过目终身不忘。...

其属文动笔如飞,初若不经意,既成,见者皆服其精妙。友生曾巩携以示欧阳修,修为之延誉。擢进士上第,签书淮南判官。旧制,秩满许献文求试馆职,安石独否。再调知鄞县,.

起堤堰,决陂塘,为水陆之利;贷谷与民,出息以偿,俾新陈相易,邑人便之。通判舒州。文彦博为相,荐安石恬退,乞不次进用,以激奔竞之风。寻召试馆职,不就。修荐为谏官,以祖母年高辞。修以其须禄养言于朝,用为群牧判官,请知常州。移提点江东刑狱,入为度支判官,时嘉祐三年也。

安石议论高奇,能以辩博济其说,果于自用,慨然有矫世变俗之志。于是上万言书,以为:“今天下之财力日以困穷,风俗日以衰坏,患在不知法度,不法先王之政故也。法先王之政者,法其意而已。法其意,则吾所改易更革,不至乎倾骇天下之耳目,嚣天下之口,而固已合先王之政矣。因天下之力以生天下之财,取天下之财以供天下之费,自古治世,未尝以财不足为公患也,患在治财无其道尔。在位之人才既不足,而闾巷草野之间亦少可用之才,社稷之托,封疆之守,陛下其能久以天幸为常,而无一旦之忧乎?愿监苟且....

因循之弊,明诏大臣,为之以渐,期合于当世之变。臣之所称,流俗之所不讲,而议者以为迂阔而熟烂者也。”后安石当国,其所注措,大抵皆祖此书。

安石性强忮,遇事无可否,自信所见,执意不回。至议变法,而在廷交执不可,安石傅经义,出己意,辩论辄数百言,众不能诎。甚者谓“天变不足畏,祖宗不足法,人言不

足恤”。罢黜中外老成人几尽,.

不复召凡八年

节选自(《宋史》列传第八十六),有删改

4.文中画波浪线的部分,断句最恰当的一项是(3分)

A.久之/以旱引去/洎复相/岁余罢/终神宗世不复召/凡八年/

B.久之以旱/引去/洎复相/岁余/罢终神宗世/不复召凡八年/

C.久之以旱/引去/洎复相/岁余罢终/神宗世不复召/凡八年/

D.久之/以旱引去/洎复相/岁余罢终神宗世/不复召/凡八年/

5.下列对文中加点词语的相关内容的解说,不正确的一项是(3分) ...

A.字介甫:古人重视礼仪,男子成人,不便直呼其名,所以另取一与本名涵义相关的别名,称之为字,以表其德。平辈之间,相互称字,则认为是有礼貌的表现。

B.陛下:“陛下”原指站在台阶下的侍者。臣子向天子进言时,不能直呼天子,必须先呼台下的侍者而告之。后来“陛下”就成为与帝王面对面应对的谦称。在此文中可译为“您”。

C.调:指一般的职务调动,类似的还有“转”、“移”、“徙”等,但文章结尾处的“黜”则是降级或免职。

D.社稷:土神和谷神的总称。由于古时的君主为了祈求国事太平,五谷丰登,每年都要祭祀土地和五谷神,后来人们就用“社稷”来代表国家。

6.下列对原文有关内容的概括和分析,不正确的一项是(3分) ...

A.王安石记忆力超群,文思敏捷,写的文章精妙,而且口才好,在辩论时常常旁征博引,把对手说得心服口服。

B.王安石任签书淮南判官期满后,并没有像其他官员一样,按照惯例进献文章要求考试馆阁职务,这表现出他特立独行的性格。

C.王安石任地方官时就采用把官谷借贷给百姓,百姓秋后加息偿还等做法,积累了许多变法的经验,这体现了王安石务实的工作作风。

D.王安石很早就有变法的思想,他在当政后就推行变法,变法虽遭到了朝廷中许多官员的反对,但这并没有动摇他变法的决心。

7.把文中画横线的句子翻译成现代汉语。(10分)

(1)父益,都官员外郎。安石少好读书,一过目终身不忘。(5分)

(2)风俗日以衰坏,患在不知法度,不法先王之政故也。(5分)

(二)古代诗歌鉴赏(11分)

阅读下面这首宋词,完成8~9题。

好事近·梦中作

①秦观

春路雨添花,花动一山春色。行到小溪深处,有黄鹂千百。

飞云当面舞龙蛇,夭矫转空碧。醉卧古藤阴下,了不知南北。

[注]①秦观,字少游。在哲宗绍圣元年(1094)被贬为监处州酒税,三年徙郴州。这首词大约作于二年的春天。

8.词的上片运用了视听结合的手法写景,请简要分析。(5分)

9.“醉卧古藤阴下,了不知南北。”表达了一种什么样的思想感情?(6分)

(三)名篇名句默写(6分)

10.补写出下列句子中的空缺部分。(6分)

(1)孔子在《论语》中,以“_________,_________”两句说明学习与思考是相辅相成的,只有把学习和思考结合起来,才能学到切实有用的真知。

(2)在《陈情表》中,作者以乌鸟反哺作比喻表达孝道的句子是“______,______”。

(3)王安石在《游褒禅山记》中用 “______________,往往有得,______________”,表明古人游赏有所收获不是随意的,而是经过深思而悟出来的。

三、现代文阅读(25分)

11.阅读下面的文字,完成(1)~(4)题。(25分)

“青蒿素之母”:屠呦呦

1930年,屠呦呦出生于浙江省宁波市。“呦呦鹿鸣,食野之苹”,父母选取《诗经·小雅》的名句为她取了这个名字。她自幼耳闻目睹中药治病的奇特疗效,小时候就对中药有了深刻印象,认为药物是治疗疾病的主要手段与工具,并决定为此奋斗终身。

屠呦呦1951年如愿考入北京大学医学院药学系,所选专业正是当时一般人缺乏兴趣的生药学。她觉得生药专业最可能接近探索具有悠久历史的中医药领域,符合自己的志趣和理想。在大学4年期间,屠呦呦努力学习,取得了优良的成绩。在专业课程中,她尤其对植物化学、本草学和植物分类学有着极大的兴趣,并打下了扎实的基础。

大学毕业后,她被分配到卫生部直属的中医研究院工作。当时,设备简陋,科研人员不足。但是,刚刚走上工作岗位的屠呦呦虽身患结核等慢性疾病,但仍坚持工作,到野外一根根采集标本,在室内夜以继日地进行实验研究,各种工作她都积极主动地完成,她是《中药炮炙经验集成》一书的主要编著者之一;她研究半边莲和中药银柴胡的成果被相继收入《中药志》。

1969年,中国中医研究院接受抗疟疾药研究任务,屠呦呦任科技组组长。从1969年1月开始,历经380多次实验,制出190多个样品,制作了2000多张卡片。其间,她查阅大量文献,借鉴古代用药的经验,设计了多种提取的方法,经过艰苦的试验、千百次的试错,终于在1971年的一次尝试中从黄花蒿中发现抗疟有效提取物,他们将这种无色的结晶体物质命名为青蒿素。1973年,屠呦呦合成出了双氢青蒿素。1981年10月,在北京召开的由世界卫生组织等主办的国际青蒿素会议上,屠呦呦以首席发言人的身份作《青蒿素的化学研究》的报告。青蒿素的研制成功被中华医学会等评为“建国35年以来20项重大医药科技成果”之一,屠呦呦也被称为“青蒿素之母”。据世卫组织统计数据,世界上约有2.5亿人感染疟疾,将近100万人因感染疟原虫而死亡。如果没有屠呦呦发现的青蒿素,那么2.5亿疟疾感染者中将有更多的人可能无法幸存下来。青蒿素是当前中国被国际承认的唯一创新药物,现在世界多国已广为应用,已经产生很大的社会效益和经济效益。

“七五”期间,屠呦呦参与国家攻关项目“常用中药材品种质量研究”中“青蒿品种整理和质量研究”课题,对青蒿进行系统研究,分离鉴定了17个化合物,其中5个为新化合物,并修正了《中国药典》长期沿用的谬误。

“屠呦呦的贡献值得获取诺贝尔奖!青蒿素经受了时间的考验,并挽救了大量患者的生命,从而证明了从传统药物获得确定化学成分药物的价值。”北大生科院院长饶毅这样评价。但屠呦呦认为自己只是推开了青蒿素的一道“门缝”,其他人合力才打开了这扇“门”。

有趣的是,屠呦呦生活上是个粗线条,一心扑在工作上,不太会照顾自已。因为工作忙,她的私人东西总放得乱七八糟的,不像一般女性收拾得那么停当。但工作中的屠呦呦十分严谨,兢兢业业,在接到“523”项目时,她女儿才三岁,为了不影响研究,她把孩子交给老母亲抚养。为了检验青蒿素治疗作用于人类身上是否安全有效,她勇敢地充当了首批志愿者,在自己身上进行实验,屠呦呦后来身体不太好,与当时长期做实验有关。

谈及自己的梦想是什么,她说,“我的梦想是用古老的中医药,促进人类健康。”

(1)下列对材料有关内容的分析和概括,最恰当的两项是(5分)

A.屠呦呦出生于浙江省宁波市,父母选取《诗经·小雅》的名句为她取“呦呦”这个名字,希望她如鹿一样能出人头地。

B.在专业课程中,屠呦呦唯独对植物化学、本草学和植物分类学有着极大的兴趣,可以说这为她以后的研究和成功打下了坚实的基础。

C.在1981年北京召开的国际青蒿素会议上,屠呦呦以首席发言人的身份作《青蒿素的化学研究》的报告,足以说明她在这一领域具有举足轻重的地位。

D.文章叙述屠呦呦在生活上的粗线条,更加突出了她为了事业所作出的牺牲,使传主的形象更丰满。

E.屠呦呦发现的青蒿素,让2.5亿疟疾感染者中更多的人幸存下来,作为当前国际承认的独特的创新药物,它产生的社会效益和经济效益难以估量。

(2)这篇传记运用了正面侧面结合的方法刻画屠呦呦形象。请结合材料简要分析。(6分)

(3)作为一名药学家,屠呦呦在医学上有哪些贡献?请结合材料简要概括。(6分)

(4)屠呦呦2015年10月获得了诺贝尔科学奖,她成功的原因有哪些?请结合材料,谈谈你的看法。(8分)

四、语言文字运用(20分)

12.下列各句中,加点的成语使用不恰当的一句是(3分) ...

A.在第二个千年的最后几十年,中国各族人民励精图治,投身于改革开放的伟大事业中。 ....

B.1997年,亚洲出现了一次金融危机,至今令人谈虎色变。 ....

C.以上所述,正是美国在经济发展方面能够长期卖弄风骚的重要原因。 ....

D.一些领域市场经济秩序还比较混乱,这不能不让人忧心忡忡。 ....

13.下列各句中,没有语病的一句是(3分)

A.我们希望通过举办纪念抗战胜利70周年活动,唤起每一个善良的人对和平的向往与坚守,避免历史悲剧不再重演。

B.按照五中全会精神落实并制定好“十三五”规划,奋力弥补生态环境方面的短板,相信到2020年,小康“木桶”里盛满的甘甜之水,将提升人们的发展获得感和生活幸福感。

C.要把“全面二孩”这一利国利民的好事办好,还需要多部门联动,充分考虑公众的诉求,尽早出台相关细则和各种配套措施。

D.在中国女药学家屠呦呦获诺贝尔奖后,罗浮山管委会将在景区内恢复“洞天药市”盛景,不但能弘扬优秀的中草药传统文化,而且能开发罗浮山的旅游资源。

14.填入下面一段文字横线处的语句,最恰当的一项是(3分)

人在世上都离不开朋友,但是,最忠实的朋友还是自己,就看你是否善于做自己的朋友了。_____________,从而能够从人生的前景出发给自己以提醒、鼓励和指导。事实上,在我们每个人身上,除了外在的自我以外,都还有着一个内在的精神性的自我。可惜的是,许多人的这个内在自我始终是昏睡着的,甚至是发育不良的。

A.必须比那个外在的自己站得更高,看得更远,你才能做自己的朋友

B.要能够做自己的朋友,你就必须比那个外在的自己站得更高,看得更远

C.不能比那个外在的自己站得更高,看得更远,就不能做自己的朋友

第二篇:《广东省惠州市年高二上学期期末语文试卷》

(时间150分钟,满分150分)

注意事项:

1.答卷前,考生务必用黑色字迹的钢笔或签字笔将自己的姓名和考生号、试室号、座位号填写在答题卷的相应位置上。

2.考生务必用黑色字迹的钢笔或签字笔作答,答案不能答在试卷上,必须写在答题卷的各题目指定区域内相应位置上;如需改动,先划掉原来的答案,然后再写上新的答案;不准使用铅笔和涂改液。

一、现代文阅读(9分,每小题3分)

阅读下面的文字,完成1—3题。

戏曲,是中国人最典型的传统审美方式。

中国戏曲雅俗共赏,选材宽泛,自古以来,既有帝王将相的阳春白雪,也有寻常百姓的家长里短;既登得了庙堂,又入得了街巷。中国戏曲深蕴传统文化的精髓,舞台之上,一桌二椅,唱念做打,节奏唱腔,全凭演员高度程式化的表演营造具体的戏剧情境和表现鲜活人物。而中国戏曲本质上是写意的:三五步走遍天下,六七人千军万马。这与整个中国的传统艺术,如文学、绘画、书法、诗歌等,都一

脉相通,代表着中国人独特的审美特质。在很长一段时间里,戏曲一直都在中国人的社会生活和审美活动中占据着最为重要的地位。

中国戏曲历经千年流传,?通变?早已融入自身血液之中,善于继承传统,也很擅长吸收和创新。戏曲代表中国人传统的审美,沉淀了中国人千年的心灵世界,而地方戏更是凝聚了千姿百态的地域文化。在全球化大趋势下,文化的多样性需要民族文艺的坚守和创新。戏曲在当代的发展,就具有了在全球化背景下保持民族特性、弘扬民族艺术不可取代的价值。因此,戏曲艺术是传统的、独特的,更是面向未来的。

戏曲是古老而又面向未来的艺术,保存个性和特性的传承,扬长避短、顺应时代要求的创新是其生命力所在。这就如同书法艺术需要发展创新,但不管如何创造,终归是点画线条的艺术,是汉字的艺术。中国戏曲整体的写意性、程式性要保留,切忌盲目采用现代舞台声光电技术?大制作?,破坏戏曲的本质。地方剧种的个性和风格也只能在创新中得到强化。

戏曲要走向未来,剧本是基础。曹禺大师早就说过,?繁荣的关键在于创作?,?剧本的生命在于演出?。目前戏曲事业发展遇到了一些瓶颈,?剧本荒?尤为突出。没有纯净坦荡、自由放飞的心态是写不出好剧本的。剧作家要静下心来,独立思考,有感而发,才能创作出形象生动、内涵深刻的剧本来。

戏曲的题材要创新。今人看古人的戏,看什么?传统戏曲应该开辟题材创作的新领域,关注现实题材的创作。题材可以是历史的,但创作理念和立意必须是现代的。站在现代人的立场上再现经典,适当取舍,挖掘经典中历久弥新的永恒主题,厚重的经典就能为现代人的生活提供源源不断的心灵慰藉。

戏曲的未来在年轻人,戏曲的观众是需要培养的,这是因为戏曲是高度艺术化的样式,?形式即内容?。现在的小孩子自小就看得懂动画片、电视剧,但让他去看戏就不一定能看懂。一是内容看不懂,二是形式看不懂,比如为什么要勾脸谱?水袖表达了些什么?戏曲有一定的欣赏门槛,观众要经历一个从看热闹到看门道的过程。因此,要创造机会让年轻人接触戏曲。现在时尚的东西太单薄,当年轻人腻烦了感官刺激,有了一定审美修养和内涵时,中国戏剧是最好的选择之一。

戏曲是创造性的艺术,是与观众当场双向交流的艺术。如果我们重视剧本创作,扬长避

短,好戏频出,老百姓怎么可能不喜欢看?新的观众不断成长,我们的戏曲怎么可能不繁荣?戏曲里有中国文化的根,是中国人最后的审美家园。

(摘编自 2010年10月22日《 人民日报 》)

1.下列有关“中国戏曲”的表述,不符合原文意思的一项是 ...

A.中国戏曲选材丰富,帝王将相、普通百姓都可以成为其表现内容。

B.中国戏曲的人物形象主要靠唱念做打等高度程式化的表演来体现。

C.中国戏曲与传统的中国文学、绘画、书法、诗歌在本质上都崇尚简约。

D.中国戏曲曾在中国人的生活和审美活动中占据着最为重要的地位。

2.下列理解,不符合原文意思的一项是 ...

A.继承传统就是要保留属于中国戏曲本身的东西,就是要弘扬独特的民族文化。

B.古老的戏曲艺术要想面向未来,既要保持自身的个性特点,也要善于创新。

C.戏曲创新可以使地方戏种的特色得到更好的发挥,从而赢得更多的年轻观众。

D.戏曲发展创新不能靠现代的声光电等舞台艺术,而应向传统的书法艺术学习。

3.下列对原文内容的理解和分析,正确的一项是

A.制约中国戏曲发展的主要问题是没有好剧本,好剧本的产生需要剧作家心无旁骛的创作,需要剧作家的独立思考。

B.看腻了动画片、电视剧的年轻人会对审美有更高的要求,而戏曲中的脸谱和水袖就可以满足他们的审美需求。

C.现在的年轻人喜欢时尚,而时尚的东西又都比较单薄缺少内涵,让年轻人接触戏剧就可以使他们变得厚重起来。

D.传统戏曲很少关注社会现实,这是观众不喜欢戏曲的根本原因。戏曲要进行题材创新,让现代观众看古人写的戏、看写古人的戏都是要不得的。

二、古代诗文阅读(36分)

(一)文言文阅读(19分)

阅读下面的文言文,完成4~7题。

王安石,字介甫,抚州临川人。父益,都官员外郎。安石少好读书,一过目终身不忘。...

其属文动笔如飞,初若不经意,既成,见者皆服其精妙。友生曾巩携以示欧阳修,修为之延誉。擢进士上第,签书淮南判官。旧制,秩满许献文求试馆职,安石独否。再调知鄞县,.

起堤堰,决陂塘,为水陆之利;贷谷与民,出息以偿,俾新陈相易,邑人便之。通判舒州。文彦博为相,荐安石恬退,乞不次进用,以激奔竞之风。寻召试馆职,不就。修荐为谏官,以祖母年高辞。修以其须禄养言于朝,用为群牧判官,请知常州。移提点江东刑狱,入为度支判官,时嘉祐三年也。

安石议论高奇,能以辩博济其说,果于自用,慨然有矫世变俗之志。于是上万言书,以为:?今天下之财力日以困穷,风俗日以衰坏,患在不知法度,不法先王之政故也。法先王之政者,法其意而已。法其意,则吾所改易更革,不至乎倾骇天下之耳目,嚣天下之口,而固已合先王之政矣。因天下之力以生天下之财,取天下之财以供天下之费,自古治世,未尝以财不足为公患也,患在治财无其道尔。在位之人才既不足,而闾巷草野之间亦少可用之才,社稷之托,封疆之守,陛下其能久以天幸为常,而无一旦之忧乎?愿监苟且....

因循之弊,明诏大臣,为之以渐,期合于当世之变。臣之所称,流俗之所不讲,而议者以为迂阔而熟烂者也后安石当国,其所注措,大抵皆祖此书。

安石性强忮,遇事无可否,自信所见,执意不回。至议变法,而在廷交执不可,安石傅经义,出己意,辩论辄数百言,众不能诎。甚者谓?天变不足畏,祖宗不足法,人言不足恤?。罢黜中外老成人几尽,多用门下儇慧少年。久之以旱引去洎复相岁余罢终神宗世.

节选自(《宋史》列传第八十六),有删改

4.文中画波浪线的部分,断句最恰当的一项是(3分)

A.久之/以旱引去/洎复相/岁余罢/终神宗世不复召/凡八年/

B.久之以旱/引去/洎复相/岁余/罢终神宗世/不复召凡八年/

C.久之以旱/引去/洎复相/岁余罢终/神宗世不复召/凡八年/

D.久之/以旱引去/洎复相/岁余罢终神宗世/不复召/凡八年/

5.下列对文中加点词语的相关内容的解说,不正确的一项是(3分) ...

A.字介甫:古人重视礼仪,男子成人,不便直呼其名,所以另取一与本名涵义相关的别名,称之为字,以表其德。平辈之间,相互称字,则认为是有礼貌的表现。

B.陛下:“陛下”原指站在台阶下的侍者。臣子向天子进言时,不能直呼天子,必须先呼台下的侍者而告之。后来“陛下”就成为与帝王面对面应对的谦称。在此文中可译为“您”。

C.调:指一般的职务调动,类似的还有“转”、“移”、“徙”等,但文章结尾处的“黜”则是降级或免职。

D.社稷:土神和谷神的总称。由于古时的君主为了祈求国事太平,五谷丰登,每年都要祭祀土地和五谷神,后来人们就用“社稷”来代表国家。

6.下列对原文有关内容的概括和分析,不正确的一项是(3分) ...

A.王安石记忆力超群,文思敏捷,写的文章精妙,而且口才好,在辩论时常常旁征博引,把对手说得心服口服。

B.王安石任签书淮南判官期满后,并没有像其他官员一样,按照惯例进献文章要求考试馆阁职务,这表现出他特立独行的性格。

C.王安石任地方官时就采用把官谷借贷给百姓,百姓秋后加息偿还等做法,积累了许多变法的经验,这体现了王安石务实的工作作风。

D.王安石很早就有变法的思想,他在当政后就推行变法,变法虽遭到了朝廷中许多官员的反对,但这并没有动摇他变法的决心。

7.把文中画横线的句子翻译成现代汉语。(10分)

(1)父益,都官员外郎。安石少好读书,一过目终身不忘。(5分)

(2)风俗日以衰坏,患在不知法度,不法先王之政故也。(5分)

(二)古代诗歌鉴赏(11分)

阅读下面这首宋词,完成8~9题。

好事近〃梦中作

①秦观

春路雨添花,花动一山春色。行到小溪深处,有黄鹂千百。

飞云当面舞龙蛇,夭矫转空碧。醉卧古藤阴下,了不知南北。

[注]①秦观,字少游。在哲宗绍圣元年(1094)被贬为监处州酒税,三年徙郴州。这首词大约作于二年的春天。

8.词的上片运用了视听结合的手法写景,请简要分析。(5分)

9.“醉卧古藤阴下,了不知南北。”表达了一种什么样的思想感情?(6分)

(三)名篇名句默写(6分)

10.补写出下列句子中的空缺部分。(6分)

(1)孔子在《论语》中,以“_________,_________”两句说明学习与思考是相辅相成的,只有把学习和思考结合起来,才能学到切实有用的真知。

(2)在《陈情表》中,作者以乌鸟反哺作比喻表达孝道的句子是“______,_

_____”。

(3)王安石在《游褒禅山记》中用 “______________,往往有得,______________”,表明古人游赏有所收获不是随意的,而是经过深思而悟出来的。

三、现代文阅读(25分)

11.阅读下面的文字,完成(1)~(4)题。(25分)

“青蒿素之母”:屠呦呦

1930年,屠呦呦出生于浙江省宁波市呦呦鹿鸣,食野之苹?,父母选取《诗经〃小雅》的名句为她取了这个名字。她自幼耳闻目睹中药治病的奇特疗效,小时候就对中药有了深刻印象,认为药物是治疗疾病的主要手段与工具,并决定为此奋斗终身。

屠呦呦1951年如愿考入北京大学医学院药学系,所选专业正是当时一般人缺乏兴趣的生药学。她觉得生药专业最可能接近探索具有悠久历史的中医药领域,符合自己的志趣和理想。在大学4年期间,屠呦呦努力学习,取得了优良的成绩。在专业课程中,她尤其对植物化学、本草学和植物分类学有着极大的兴趣,并打下了扎实的基础。

大学毕业后,她被分配到卫生部直属的中医研究院工作。当时,设备简陋,科研人员不足。但是,刚刚走上工作岗位的屠呦呦虽身患结核等慢性疾病,但仍坚持工作,到野外一根根采集标本,在室内夜以继日地进行实验研究,各种工作她都积极主动地完成,她是《中药炮炙经验集成》一书的主要编著者之一;她研究半边莲和中药银柴胡的成果被相继收入《中药志》。

1969年,中国中医研究院接受抗疟疾药研究任务,屠呦呦任科技组组长。从1969年1月开始,历经380多次实验,制出190多个样品,制作了2000多张卡片。其间,她查阅大量文献,借鉴古代用药的经验,设计了多种提取的方法,经过艰苦的试验、千百次的试错,终于在1971年的一次尝试中从黄花蒿中发现抗疟有效提取物,他们将这种无色的结晶体物质命名为青蒿素。1973年,屠呦呦合成出了双氢青蒿素。1981年10月,在北京召开的由世界卫生组织等主办的国际青蒿素会议上,屠呦呦以首席发言人的身份作《青蒿素的化学研究》的报告。青蒿素的研制成功被中华医学会等评为?建国35年以来20项重大医药科技成果?之一,屠呦呦也被称为?青蒿素之母?。据世卫组织统计数据,世界上约有2.5亿人感染疟疾,将近100万人因感染疟原虫而死亡。如果没有屠呦呦发现的青蒿素,那么2.5亿疟疾感染者中将有更多的人可能无法幸存下来。青蒿素是当前中国被国际承认的唯一创新药物,现在世界多国已广为应用,已经产生很大的社会效益和经济效益。

?七五?期间,屠呦呦参与国家攻关项目?常用中药材品种质量研究?中?青蒿品种整理和质量研究?课题,对青蒿进行系统研究,分离鉴定了17个化合物,其中5个为新化合物,并修正了《中国药典》长期沿用的谬误。

?屠呦呦的贡献值得获取诺贝尔奖!青蒿素经受了时间的考验,并挽救了大量患者的生命,从而证明了从传统药物获得确定化学成分药物的价值北大生科院院长饶毅这样评价。但屠呦呦认为自己只是推开了青蒿素的一道?门缝?,其他人合力才打开了这扇?门?。

有趣的是,屠呦呦生活上是个粗线条,一心扑在工作上,不太会照顾自已。因为工作忙,她的私人东西总放得乱七八糟的,不像一般女性收拾得那么停当。但工作中的屠呦呦十分严谨,兢兢业业,在接到?523?项目时,她女儿才三岁,为了不影响研究,她把孩子交给老母亲抚养。为了检验青蒿素治疗作用于人类身上是否安全有效,她勇敢地充当了首批志愿者,在自己身上进行实验,屠呦呦后来身体不太好,与当时长期做实验有关。

谈及自己的梦想是什么,她说,?我的梦想是用古老的中医药,促进人类健康

(有删改)

[相关链接]

2015年10月,屠呦呦因发现青蒿素治疗疟疾的新疗法,获诺贝尔生理学或医学奖,她是第一位获得诺贝尔科学奖项的中国本土科学家、第一位获得诺贝尔生理医学奖的华人科学家。(百度百科)

(1)下列对材料有关内容的分析和概括,最恰当的两项是(5分)

B.在专业课程中,屠呦呦唯独对植物化学、本草学和植物分类学有着极大的兴趣,可以说这为她以后的研究和成功打下了坚实的基础。

C.在1981年北京召开的国际青蒿素会议上,屠呦呦以首席发言人的身份作《青蒿素的化学研究》的报告,足以说明她在这一领域具有举足轻重的地位。

D.文章叙述屠呦呦在生活上的粗线条,更加突出了她为了事业所作出的牺牲,使传主的形象更丰满。

E.屠呦呦发现的青蒿素,让2.5亿疟疾感染者中更多的人幸存下来,作为当前国际承认的独特的创新药物,它产生的社会效益和经济效益难以估量。

(2)这篇传记运用了正面侧面结合的方法刻画屠呦呦形象。请结合材料简要分析。(6分)

(3)作为一名药学家,屠呦呦在医学上有哪些贡献?请结合材料简要概括。(6分)

(4)屠呦呦2015年10月获得了诺贝尔科学奖,她成功的原因有哪些?请结合材料,谈谈你的看法。(8分)

四、语言文字运用(20分)

12.下列各句中,加点的成语使用不恰当的一句是(3分) ...

A.在第二个千年的最后几十年,中国各族人民励精图治,投身于改革开放的伟大事业中。 ....

B.1997年,亚洲出现了一次金融危机,至今令人谈虎色变。 ....

C.以上所述,正是美国在经济发展方面能够长期卖弄风骚的重要原因。 ....

D.一些领域市场经济秩序还比较混乱,这不能不让人忧心忡忡。 ....

13.下列各句中,没有语病的一句是(3分)

A.我们希望通过举办纪念抗战胜利70周年活动,唤起每一个善良的人对和平的向往与坚守,避免历史悲剧不再重演。

B.按照五中全会精神落实并制定好“十三五”规划,奋力弥补生态环境方面的短板,相信到2020年,小康“木桶”里盛满的甘甜之水,将提升人们的发展获得感和生活幸福感。

C.要把“全面二孩”这一利国利民的好事办好,还需要多部门联动,充分考虑公众的诉求,尽早出台相关细则和各种配套措施。

D.在中国女药学家屠呦呦获诺贝尔奖后,罗浮山管委会将在景区内恢复“洞天药市”盛景,不但能弘扬优秀的中草药传统文化,而且能开发罗浮山的旅游资源。

14.填入下面一段文字横线处的语句,最恰当的一项是(3分)

人在世上都离不开朋友,但是,最忠实的朋友还是自己,就看你是否善于做自己的朋友了。_____________,从而能够从人生的前景出发给自己以提醒、鼓励和指导。事实上,在我们每个人身上,除了外在的自我以外,都还有着一个内在的精神性的自我。可惜的是,许多人的这个内在自我始终是昏睡着的,甚至是发育不良的。

A.必须比那个外在的自己站得更高,看得更远,你才能做自己的朋友

B.要能够做自己的朋友,你就必须比那个外在的自己站得更高,看得更远

C.不能比那个外在的自己站得更高,看得更远,就不能做自己的朋友

D.做自己的朋友,只要比那个外在的自己站得更高,看得更远,就可以做到

第三篇:《惠州市高二年级第一学期期末考试语文试题)》

(本试题满分150分,考试用时150分钟。)

注意事项:

考生务必用黑色字迹的钢笔(或圆珠笔、签字笔)填写相关栏目(如学校、姓名、考生号等)并在试卷上直接答题。答案如需改动,先划掉原来的答案,然后再写上新的答案;不准使用铅笔和涂改液。不按以上要求作答的答案无效。

一、语言知识及运用(12分,每小题3分)

1.下列各项中加点字读音无误的一项是(3分)

A.反哺(bǔ) 籼稻(xiān) 崤山(xiáo) 毋庸置疑(mù) 潜移默化(qiǎn) .....

B.契约(qì) 履行(lǚ) 即使(jí) 商贾云集(gǔ) 管中窥豹(guī) .....

C.眺望(tiào) 俯瞰(kàn) 阐明(chǎn) 炯炯有神(jiǒng) 头晕目眩(xuàn) .....

D.证券(quàn) 梵语(fán) 卑鄙(pì) 载歌载舞(zài) 潇洒倜傥(tì) .....

2.下面各句中加点的成语,使用恰当的一项是(3分)

A.德国《经济新闻报》文章称中国2014年研发投入将超越欧洲,有望于5年后超越美国。近年来,中国正不遗余力地转型经济模式,希望把国家打造成为技术创新大国。 ....

B.在第十届珠海航展上,六机筋斗、三机交叉、半滚倒转着陆??“八一”飞行表演队鬼斧神工的表演让世界看到了歼-10的优异性能,展示了中国空军近年来的迅速成长。 ....

C.一场拿破仑主题拍卖会16日在法国枫丹白露市举办,吸引了来自世界各地众多“皇帝粉”的热情关注和参与,一顶拿破仑曾经戴过的帽子更是拍出了180万欧元的高价,令人高山仰止。 ....

D.今年下半年以来,国际金价持续下挫,目前已跌至四年新低。去年刷新购金记录的中国大妈,并未再度慷慨解囊,而是捂金待涨。 ....

3.下列各项句子中,没有语病的一项是(3分)

A.我国针对一直把“秦淮线”作为冬季集中供暖南北分界线的现实情况,有学者认为应以室外温度作为判断当地是否应该集中供暖的决定因素。

B.恐怕很少

有人想到,以卖国求荣、屈膝投降而被世人所不齿的奸臣秦桧早年还一度是个挺有主见、口碑不错的主战派人士。

C.由陈可辛执导,赵薇、黄渤、佟大为等主演的电影《亲爱的》以“寻子”为主题凸显了法律与伦理的矛盾,引得众多观众潸然泪下。

D.根据世界卫生组织(WHO)14日发布的最新报告显示,埃博拉疫情已造成至少5177人死亡,感染及疑似病例达14413例,利比里亚、塞拉利昂和几内亚3国成为核心疫区。

4.把下列句子组成语意连贯的一段文字,排序最恰当的一项是(3分)

①我国已基本形成了市场经济框架,但规则和信用这两块基石还不太牢固,存在破损。 ②统而言之,市场经济实质上是法治与德治统一的经济。

③市场经济是按规则运作的经济,因而市场经济是法治经济。

④规则是市场经济的法制基石,信用是市场经济的道德基石。

⑤市场经济又是讲信用的经济,因而市场经济又是德治经济。

⑥在这里,规则和信用是市场经济的两大基石。

A.①②③⑤④⑥ B.②③⑤①⑥④ C.①④⑥③⑤② D.③⑤②⑥④①

二、古诗文阅读(35分)

李密,字令伯,犍为武阳人也,一名虔。父早亡,母何氏改醮。密时年数岁,感恋弥至,烝烝之性,遂以成疾。祖母刘氏,躬自抚养。密奉事以孝谨闻,刘氏有疾,则涕泣侧息,未尝解衣,饮膳汤药必先尝后进。有暇则讲学忘疲,而师事谯周,周门人方之游夏。

少仕蜀,为郎。数使吴,有才辩,吴人称之。蜀平,泰始初,诏征为太子洗马。密以祖母年高,无人奉养,遂不应命。乃上疏曰:“臣以险衅……”

武帝览之曰:“士之有名,不虚然哉!”乃停召。后刘终,服阕,复以洗马征至洛。司空张华问之曰:“安乐公何如?”密曰:“可次齐桓。”华问其故,对曰:“齐桓得管仲而霸,用竖刁而虫流。安乐公得诸葛亮而抗魏,任黄皓而丧国,是知成败一也。”次问:“孔明言教何碎?”密曰:“昔舜、禹、皋陶相与语,故得简雅;《大诰》与凡人言,宜碎。孔明与言者无己敌,言教是以碎耳。”华善之。

出为温令,而憎疾从事,尝与人书曰:“庆父不死,鲁难未已。”从事白其书司隶司隶以密在县清慎弗之劾也密有才能常望内转而朝廷无援乃迁汉中太守自以失分怀怨。及赐饯东堂,诏密令赋诗,末章曰:“人亦有言,有因有缘。官无中人,不如归田。明明在上,斯语岂然!”武帝忿之,于是都官从事奏免密官。后卒于家。

(《晋书·李密传》)

[注]①醮(jiào):改嫁。 ②烝烝(zhēng):热切的样子,形容李密对母亲思念之深。 ③游夏:指孔子的学生子游和子夏,他们在文学上皆有造诣。

5.对下列句子中加点的词的解释,不正确的一项是(3分)

A.孔明言教何碎?碎:琐碎 B.后刘终,服阕。服:服丧 ..

C.庆父不死,鲁难未已。已:表语气,同“矣” D.武帝忿之 忿:对??生气 ..

6.下列各组句子中,加点的词的意义和用法都相同的一组是(3分)

A.密奉事以孝谨闻 险以远,则至者少 ..

B.士之有名,不虚然哉 苏子与客泛舟,游于赤壁之下 ..

C.人亦有言,有因有缘 因利乘便,宰割天下 ..

D.后卒于家 某所,而母立于兹 ..

7.下列各项中对原文中未加标点部分内容断句正确的一项是(3分)

A.从事白其书司隶/司隶以密在县清慎/弗之劾也/密有才能/常望内转/而朝廷无援/乃迁汉中太守/自以失分怀怨

B.从事白/其书司隶/司隶以密在县/清慎弗之/劾也/密有才/能常望内转/而朝廷无援/乃迁汉中/太守自以失分怀怨

C.从事白其书司隶/司隶以密在县清慎/弗之劾也/密有才能常望内/转而朝廷/无援乃迁/汉③②①

中太守自以失分/怀怨

D.从事白/其书司隶/司隶以密在县/清慎弗之劾也/密有才能/常望内转而朝廷/无援乃迁汉中/太守自以失分怀怨

8.以下各项中,句式与“言教是以碎耳”不同类的一项是(3分)

A.州司临门,急于星火 B.姜氏何厌之有

C.邠人偷嗜暴恶者 D.吾属今为之虏矣

9.以下各项表述与原文内容不符的一项是(3分)

A.李密父亲早亡,母亲改嫁,自小体弱多病,其祖母刘氏照顾他长大。刘氏生病后,李密细心服侍,体贴备至。

B.李密年轻时曾在孙权属下为官,颇有才能,时人多称之。泰始初年,李密被征召为太子洗马,他上书《陈情表》辞不赴命。

C.司空张华问李密他怎样评价刘禅,李密认为刘禅在用人而使国家成败之上与齐桓公相仿。

D.李密在被晋武帝召见时赋诗一首,语含讥讽,都官从事上奏要求罢免李密官职。

10.请将下列句子翻译为现代汉语(6分)

①此所以学者不可以不深思而慎取之也。(3分)

②若阙地及泉,隧而相见,其谁曰不然?(3分)

11.阅读下面两首诗,回答问题。(6分)

题乌江亭

杜牧

胜败兵家事不期,包羞忍耻是男儿。

江东子弟多才俊,卷土重来未可知。

乌江亭

王安石

百战疲劳壮士哀,中原一败势难回。

江东子弟今虽在,肯为君王卷土来?

(1)以上两首诗在类型和题材上有何相同之处?(2分)

(2)试赏析两诗在思想内容上的不同。(4分)

12.请默写下列句子中的空缺部分(任选4题作答,全选按前4题给分,每空1分,共8分)

(1)臣欲奉诏奔驰,_______;_______,则告诉不许。(《陈情表》)

(2)有志与力,而又不随以怠,_____________,_______。(《游褒禅山记》)

(3)亦欲以究天人之际,_______,_______。(《报任安书》)

(4)大行不顾细谨,_______。如今人方为刀俎,_______,何辞为?(《鸿门宴》)

(5)_______,_______;吾尝跂而望矣,不如登高之博见也。(《劝学》)

三、阅读下面一段材料,完成13-16题(16分)。

地球表面首次发现彗星尘埃

研究人员日前发现了保存于南极冰雪中的彗星尘埃,这是科学家第一次在地球表面发现这样的微粒。这一研究成果开启了寻找这种物质的一条前所未有的新途径。一旦彗星尘埃这种最古老的天文学微粒能够被用于研究,将为搞清太阳系如何形成提供线索。

并未参与该项研究的美国华盛顿哥伦比亚特区卡内基研究所地磁学部行星科学家Larry Nittler表示:“对于那些研究地外物质的科学家而言,这确实非常令人激动,因为这为获取这些物质打开了一扇大门。”他说:“研究人员找到了一些非常有趣和非常罕见的物质的新来源。”

直到最近,除了飞向太空,科学家收集“球粒状陨石多孔星际尘埃粒子”,或者说彗星尘埃的唯一途径便是驾驶研究飞机在平流层高度飞行。这是一项非常艰苦的工作:几个小时的飞行通常只能找到一颗尘埃微粒。这项研究的共同作者、夏威夷大学及夏威夷地球物理与行星学研究所星际物质学家John Bradley表示,这么少的样本极大限制了科学家对物质进行的测试和分析研究。

Bradley指出,研究人员现在在南极发现了更多的彗星尘埃微粒。他说:“通过这种方式有可能采集到2到4个数量级质量的尘埃微粒。”“我相信这样采集的微粒有可能促成研究范式的转换。”

与此同时,采集自南极的尘埃还更加干净。目前,科学家用飞机采集彗星尘埃往往都是使用一个涂抹着硅油的盘子捕捉微粒,就像用捕蝇纸逮苍蝇一般。这就使得尘埃微粒会被硅油以及随后用于清洗它们的有机化合物所污染,从而让想要研究其有机物质构成的科学家感到一筹莫展。

Nittler指出,比较在南极和在平流层收集的彗星尘埃微粒可以帮助科学家找出哪些成分是尘埃的自然化学组成,而哪些成分来自污染物。

2010年,一个法国研究小组报告说他们在南极冰雪中发现了一些致密且富含碳的彗星微粒,但最新的研究则第一次发现了更为典型的彗星尘埃并得到了证明。科学家曾认为,高孔隙度的、极其脆弱的粒子无法在地球上存在。

为了找到它们,研究人员从2000年开始,在南极的两个不同地点采集了冰雪样本。通过将冰雪融化并过滤融水,他们采集到3000多个微小陨石——直径为10微米或更大的来自空间的微粒。

在5年的时间里,通过在立体显微镜下一个接一个地分析这些微小陨石,研究人员发现了40多个具有彗星尘埃特征的微粒。更进一步的分析表明,它们与采集自平流层的彗星尘埃几乎无法区分,并且它们与美国宇航局(NASA)的星尘号任务于2006年采集的彗发样本也完全匹配。

研究人员日前在《地球与行星科学快报》上报告了这一研究成果。

主持这项研究的日本福冈九州大学陨石研究人员Takaaki Noguchi表示:“我们的研究结果表明,这种脆弱的微粒不但能够在雪里,也可以在冰里被保存下来。”

之前在南极发现彗星尘埃的法国陨石研究人员Cecile Engrand表示,下一步将对这些微粒的有机构成进行更为细致的分析。她说:“对这些彗星颗粒进行的研究将有助于更清楚地了解行星的形成过程。它们是那段时期最好的见证者。”

(2014年12月09日 《中国科学报》)

13.根据文章内容,以下各项说法中有误的两项是(5分) .....

A.2010年,一个法国研究小组报告说他们在南极冰雪中发现了一些致密且富含碳的彗星微粒,随即被确认为彗星尘埃。

B.在从南极发现彗星尘埃之前,收集彗星尘埃的唯一途径是驾驶研究飞机在平流层高度飞行。

C.科学家曾经认为,高孔隙度的、极其脆弱的粒子无法在地球上存在。南极彗星尘埃的发现否定了这一观点。

D.夏威夷地球物理与行星学研究所星际物质学家John Bradley认为,传统的平流层收集彗星尘埃的方法限制了科学家对彗星尘埃的测试和分析研究。

E.通过将南极冰雪融化并过滤融水的方式,科学家已经采集到至少3000个直径为10微米或更大的微小陨石。

F.科学家对从南极冰雪中发现的微小陨石分析后发现,它们与采集自平流层的彗星尘埃几乎无法区分。

14.以下各项中表述有误的一项是(3分) ..

A.进行南极彗星尘埃研究的不仅有日本科学家,还有法国科学家。

B.在平流层的彗星尘埃采集方式中,科学家常用捕蝇纸捕捉彗星微粒。

C.在南极发现的彗星尘埃不仅存在于南极的雪里,也可能存在于南极的冰中。

D.在南极发现的彗星尘埃可能与太阳系的形成处于同一时期。

15.与在南极收集彗星尘埃相比,在平流层收集彗星尘埃存在什么不足?(4分)

16.请根据文章内容分析,从南极冰雪中发现彗星尘埃有何科学意义?(4分)

四、阅读下文,完成17-19题(15分)

纪连海:那些不为人知的故事

纪连海是第一位走进央视《百家讲坛》的中学历史老师。他的讲述诙谐幽默、鞭辟入里。

第18篇:同心县—学年第一学期九年级教学质量监测

同心县2015—2016学年第一学期九年级教学质量监测

语 文 试 卷

1.全卷总分120分,其中卷面分4分。答题时间150分钟。 2.答题前将密封线内的项目填写清楚。

3.请各位考生将所有答案全部答在答题卡相应的位置上。

一、积累。(36分)

1、古诗文默写。(16分)

(1)浊酒一杯家万里, 。(范仲淹《渔家傲.秋思》) (2) ,西北望,射天狼。(苏轼《江城子.密州出猎》) (3)过尽千帆皆不是, 。(温庭筠《望江南》)

(4) ,坐断东南战未休。(辛弃疾《南乡子.登京口北固亭有怀》)(5) ,载不动许多愁。(李清照《武陵春》) (6)东篱把酒黄昏后, 。(李清照《醉花阴》)

(7)了却君王天下事, 。(辛弃疾《为陈同甫赋壮词以寄之》) (8)力尽不知热, 。(白居易《观刈麦》) (9)更深夜色半人家, 。(刘方平《月夜》) (10) ,人迹板桥霜。(温庭筠《商山早行》) (11)酒困路长惟欲睡, 。(苏轼《浣溪沙》) (12) ,原是今朝斗草赢。(晏殊《破阵子》) (13) ,一任群芳妒。(陆游《卜算子.咏梅》) (14) ,衡阳雁去无留意。(范仲淹《渔家傲.秋思》) (15)陈涉太息曰:“嗟乎! !”(司马迁《陈涉世家》) (16) ,奉命于危难之间。(诸葛亮《出师表》) 2.在下面括号里填上正确的字。(2分)

妙手( )得 锋芒( )露 地大物( ) 置之( )外

3、可爱的汉字。(仿照例子,完成汉字与汉字的对话。)(2分)

“兵”对“丘”说:兄弟,踩上地雷了,两条腿都炸没了? “巾”对“币”说:戴上博士帽就身价百倍了! (1)“也”对“她”说: (2)“由”对“甲”说:

4、结合语境,将下列句子填入横线处,顺序最恰当的一项是( )(2分)

国家发展、民族振兴,不仅需要强大的经济力量,更需要强大的文化力量。 没有先进文化的发展,没有全民族文明素质的提高,就不可能真正实现现代化。 ①是一个民族真正有力量的决定性因素 ②改变一个民族的命运 ③文化是一个民族的精神和灵魂 ④可以深刻影响一个国家发展的进程

A.②④③① B.①③④② C.③①④② D.③①②④

5、一老外来华留学4年,主攻汉语。临毕业,参加中文晋级考试:考题是请写出下面两句话的区别在哪里? (1)、冬天:能穿多少穿多少;夏天:能穿多少穿多少。 (2)、剩女产生的原因有两个,一是谁都看不上;二是谁都看不上。 (3)、女孩给男朋友打电话:如果你到了,我还没到,你就等着吧;如果我到了,你还没到,你就等着吧。 (4)、单身的原因:原来是喜欢一个人,现在是喜欢一个人。 老外泪流满面,交白卷,自叹不如,回国了。

聪明的你请在以上四题中任选两题....,写出所选题目当中两句话的区别在哪里(4分) __________________________________________________________________________ 6.最近热播的《芈月传》,成为街头巷尾人们茶余饭后的谈资。有人主张让青少年好好看看此剧,因为它就是一部很好的语文教材,囊括的语文知识举不胜举;有人认为剧中权利的争夺、宫廷政治争斗不利于青少年的身心健康发展,你认为青少年可否“追剧”,请谈谈你的观点。(2分)

__________________________________________________________________________

7.漫画赏析。(3分)

(1)请给这幅漫画拟个标题(1分) _______________________________

(2)请写出这幅漫画的寓意(2分) _______________________________ _______________________________

8、中新社石家庄2016年1月3日电(记者陈林)元旦假日期间,华北地区再遭雾霾侵袭。河北多地被雾霾持续笼罩。3日上午,石家庄正定国际机场能见度最小不足100米,丽江-石家庄-沈阳等4个航班取消,广州-石家庄CZ3131航班等4个航班备降其他机场。当日13时41分起,石家庄机场天气好转,备降航班陆续返回。记者从河北高速交警总队指挥中心了解到,3日早间北京以南所有高速全部因雾关闭,后多条高速逐渐开启。由于雾情反复,截至记者18时发稿时,大广高速廊坊到邯郸辖区所有站口全线关闭,黄石高速石家庄到沧州辖区沿线站口全线关闭,衡水、邢台辖区所有高速因雾关闭。河北气象部门2日将霾预警由黄色预警信号升级为橙色预警信号。

请用一句话概括这则新闻的主要内容。(不超过20字)(2分)

9、亲近名著。(3分)

作为中国第一部歌颂农民起义的长篇 体小说《 》,生动地描绘了梁山好汉们从起义到兴盛再到失败的全过程。鲜明地表现了“ ”的主题。这恰好印证了“哪里有压迫,哪里就有反抗”的真理。

二、阅读(40分)

(一)(16分)

10、解释下面句子中加点词语。(4分)

(1)未尝不叹息痛恨..于桓、灵也_______________________________ (2)欲信.大义于天下_______________________________ (3)是以先帝简拔以遗.陛下_______________________________ (4)固以怪.之矣_______________________________

11、翻译下列句子。(6分)

(1)由是感激,遂许先帝以驱驰。_______________________________

(2)怀怒未发,休祲降于天,与臣而将四矣。_______________________________ (3)苟全性命于乱世,不求闻达于诸侯。_______________________________

12、《出师表》中,诸葛亮分析天下形势,在文中的作用是什么?(2分) ___________________________________________________________

13、《陈涉世家》中,陈胜是怎样为起义做舆论准备的?(2分) ___________________________________________________________

14、《武陵春》中,“只恐双溪舴艋舟,载不动许多愁”是一个“创意出奇”的句子,请说说此句“奇”在何处。(2分)

___________________________________________________________

(二)阅读下文,完成下列各题(共10分)

智者无言

(1)大海的深处是平静的。花朵的一生是无声的。巍峨的山脉是缄默的。 (2)大自然中许多蔚为壮观的生命往往以沉默示人,而人的生命是否亦当如此? (3)有这么两个人应考厨师。甲一开言便说了个天花乱坠,从传统的中国名菜到洋派的西式糕点,从每道菜的用料着色到火候调味,似乎无一不能无一不精。乙则静候一旁一语不发。待问及他时,他只是说:“给我30分钟的时间,我做一桌菜出来看看便是。” (4)不用说,“讷于言”的厨师被录取了。

(5)机智和美妙的语言有时只不过是一种一瞬的智慧和淋漓的表达,但智慧和表达本身并不证明结果。朴实的行动才是开在成功路上的鲜花。

(6)认识一个人,不要光听他怎么说,而应当看他怎么去做,因为有的语言言不由衷。就

2 像认识一棵树,不需急于去看它春天开的什么花,因为有一种事实叫做华而不实。你可以等到秋天,那时候你去看树上的果实,果实是花的语言,也是树的注释。

(7)芸芸众生,那个与你仅有一面之交便一览无余的人,你会觉得他索然无味,因为他说得太多。而那个一直一声不吭以沉默示人的人,你不仅仅对他印象深刻,而且产生了探询他的愿望─因为那沉默带给女性的是典雅矜持,带给男性的是深邃练达。你要记住,生活中有些东西藏在心里便是一种真实,一种深刻;说出来,反而索然无味,比如一生一世的爱情。

(8)很小时候,妈妈对我说:你看大街上那两个吵嘴的人,声嘶力竭的人往往理屈词穷。因为真理不在他那一边,他想先声夺人来掩饰自己的心虚。而那个沉默者心里一身正气,自然可以不动声色,因为沉默是强有力的蔑视。

(9)一个浅显的道理是:乌鸦和知了无疑是最善叫的,但它们都不是人类的宠物。 (10)火山爆发那一声震天的巨响,那力拔山兮的气概,令人叹为观止。但这一奇观不会随时可见,为那一瞬间的壮观,它在地层深处沉寂了千年万年!

⑾有两种方法可以使世界变大:登上山峰远眺,闭上眼睛深思。它们与语言无关。由此看来,世界不是语言描绘得大的,想必人也盖同此理。 ⑿智者无言。

15、细读全文,说说作者的观点是什么?(2分)

___________________________________________________________

16、文章开头写了大海,花朵山峦,它们的共同点是什么?这样写的作用是什么?(2分) ___________________________________________________________

17、第三段讲述了两个厨师应考的事,其中运用的论证方法主要有哪两种?(2分)

___________________________________________________________

18、“世界不是用语言描绘得大的,想必人也盖同此理。”这句话中的“此理”说的是什么道理?(2分)

___________________________________________________________

19、你是否同意本文作者的观点,请谈谈你的看法。(2分)

(三)阅读《掌声》,完成下列各题。(共14分)

⑴在教室门外,我听到,像往日一样,上课铃一响,教室里一下子静了下来。

⑵走进教室,就感到安静里好像潜伏着一种与平日不同的气氛。但我还是平静地走上讲台,还没开口讲课,就发现全班五十多个同学都直勾勾地盯着我身后的黑板。 ⑶为了把学生的注意力吸引到我的讲课中,我立即以平静的语调导入新课: ⑷\"同学们,今天,我们上--\"

⑸我一边说一边转身准备往黑板上板书。

⑹班上的女学生王娜娜边站边喊道:\"报告老师,你看黑板上--\"

⑺我一下子惊呆了,原来黑板有一行清楚的粉笔字:\"高老师是个--\"后边有一个不太清晰的\"坏\"字。

⑻这是我教学十年来从未出现过的情况,过去每次上语文课时,黑板总是擦得干干净净。这几个字,显然是我有什么得罪学生的地方,他们是在公开向我挑战。 ⑼我仔细一看,这字体像是我昨天批评过的李晓写的。

⑽我心里的火一下子蹿得老高,感到头皮都\"啪啪\"炸响。但是瞬间我就控制住了自己,并决定改变教学内容。

(11)我面带笑容说道:\"同学们,今天,我们上说话课,题目有些同学已经知道,并替我写在黑板上,谢谢这位同学。\"

(12)尽管我的话里透着真诚,很多同学还是一脸的不安。

(13)我用粉笔把不太清晰的\"坏\"字重描了一下,并添上了\"老师吗?\"。 (14)许多同学这才发出了善意的笑声,课堂气氛已转入正常。

(15)我立即一口气说了下去:\"我,就是你们的高老师,是个坏老师吗?今天,我愿意把一个真实的我向同学们介绍一下。\"

(16)接着我详细地介绍我的生活和工作的情况,也坦诚地承认了自己的一些弱点和缺点。(17)由于是说自己,我说得非常流畅,一句多余的话也没有,口才比平常显得更好。我说完了,教室里一片沉静。我感到,同学们被我的真诚感动了。果然,一阵热烈的掌声响了起来,全班同学都在热烈地鼓掌。连李晓眼中也有点晶莹,他的手也拍得非常起劲。3

(18)--主课是从来不兴鼓掌的,这在我的教学生涯中是第一次。 (19)我心中为自己即兴设计的教学方案陶醉了。

(20)我潇洒地将\"高老师\"三个字底下画上了一道横线,擦掉了\"高老师\"三个字,将题目改成\"是个吗?\"要求道:\"请各位同学考虑一下,完善题目,并以这个题目说一段话。\" (21)李晓第一个举起了手,我让他站了起来。

(22)他说:\"我的题目是--李晓是那个写\'高老师是坏\'这几个字的学生吗?\" (23)我心中一惊,学生的眼光是多么犀利啊!尽管我自认为表现得很潇洒,但还是被学生一眼看穿了。

(24)李晓讲得也非常流畅,否定了黑板上的字是他写的。尽管我心中不相信他的话,但对他的说话艺术还是赞许的。学生们又鼓起掌来,这掌声似乎比给我的更热烈。 (25)掌声一落,女学生王娜娜举起了手,她说:\"我说话的题目是\'王娜娜是个坏学生吗\'。\"王娜娜承认那行字是她写的,主要是想看一下老师是否有雅量,到底有多大的雅量。她说她不是个坏学生,高老师也不是个坏老师。今天,她感到老师的形象更加高大了。 (26)我非常惊喜,为她的大胆活泼而高兴,更为她的说话水平而高兴。全班同学的掌声又一次热烈地响了起来。

(27)此事过去又快十年了。如今,李晓已成为一个著名作家;王娜娜在法国留学,已获得博士学位,正在读博士后。这个班的学生见到我或者来信时,说最佩服的是我处理这节课的方式,这节课是他们印象最深刻的一堂语文课。

(28)其实,近十年来,这三次掌声也仍然时时回响在我的耳边。 20、请用简要的语言概括这篇小说的主要内容。(3分) ___________________________________________________________

21、体会文中画线句“我一下子惊呆了”和“我心中一惊”,说说两个“惊”字表现的人物心理有何不同。(2分) ___________________________________________________________

22、下列有关本文的分析不恰当的一项是(2分)(

A.第⑵段中“安静里好像潜伏着一种与平日不同的气氛”一句的作用是为接下来矛盾的 突然出现蓄势。

B.从高老师对突发事件的处理上,我们可以看出他是一个不仅有理性,而且有智慧,善于克制自己又能随机应变的老师。

C.第(27)段写李晓成为作家和王娜娜在法国留学且博士后在读这一结果,印证了这两个学生都不是“坏学生”。

D.从这篇小说的主旨来看,是要告诫同学们要尊重老师,不要随意损害老师的人格尊严,只有这样,才能使师生关系更加和谐。

23、第(17)自然段中“连李晓眼中也有点晶莹,他的手也拍得非常起劲。”这句话在本文中有什么用?(2分)

___________________________________________________________

24、仔细揣摩本文第(25)段写王娜娜发言的文字,体会它对推动小说的情节发展起到什么作用。(2分)

___________________________________________________________

25、意本文的标题若改为“一堂难忘的语文课”好不好?请简述理由。(3分) ___________________________________________________________

三、写作(40分)

一个人在成长的历程中,需要的有时很多,有时很少。有的人需要无限的鼓励、关怀、理解和空间,有的人则只需要一张书桌、一个上学的机会,有的人渴望成长,无惧挫折;有的人则害怕成长所付出的代价……

亲爱的同学,你认为成长最需要什么?请结合自己的经历、认识和感受,在下面题目的横线上,填入一个你自己认为合适的词语,(如宽容、空间、挫折、快乐、付出代价等。)然后作文。 题目:成长,需要

要求:(1)除诗歌外,文体不限。

(2)字数不少于600字。(3)文中不要出现真实的人名、校名。

第19篇:九年级第一学期语文教案

1、沁园春

知识目标:

1、写景、议论、抒情的表达方式

2、背诵课文、默写课文 能力目标:理解背景、把握大意

教育目标:热爱祖国大好河山,树立伟大抱负和坚定信念 重点、难点:理解“数风流人物还看今朝” 教时:两课时 教学过程

一、检查预习

1、指名朗读课文

2、让学生谈谈对这首词的初步理解。

二、题解与背景

本文体裁词。词又叫诗的别体,长短句。沁园春,词牌名。

三、读课文,整体感知课文内容

这首词分上下两阕,各写什么? 上片:描写北国雪景; 下片:纵论历代英雄

四、研究下列问题

1、总写北国雪景的句子是?

2、作者以“望”统领下文,都望到了什么?(长城、黄河、山脉、高原

这些景观大势磅礴,显示了诗人博大的胸怀、雄伟的气魄,是实写。

3、哪几句是虚景?

4、你认为以动写静的句子是哪些?以静写动的句子是哪些?(山舞银蛇,原驰蜡象„„)

5、在下片中,能起到领起下文的字是?它领起哪些句子?(惜)

6、如何理解本文的言主旨? 诗人描写北国雪景,有雄壮之气势,豪放的风格,抒发了诗人对祖国山河的热爱,表现了诗人豪迈的胸怀。

五、重点词句子的理解。

1、“山舞银蛇,原驰蜡象”的含义。

2、“江山如此多娇,引无数英雄竞折腰”在结构上的作用。承上启下,过渡作用

3、结句“俱往矣,数风流人物,还看今朝”有何寓意?

只有今天的一代英雄人物,才能使多娇江山完全称意。这是对一代新人的呼唤和企盼。

六、作业。背诵这首词。 [补充资料]

〔顿失滔滔〕这里指黄河因冰封而立刻消失滚滚的波浪。

〔看红装素裹,分外妖娆(rào饶)〕红日和白雪互相映照,看去好像装饰艳丽的美女裹着白色外衣,格外娇媚。

〔竞折腰〕折腰,倾倒,躬着腰侍候。这里是说争着为江山奔走操劳。

〔秦皇汉武〕秦始皇嬴(yíng盈)政(前二五九??前二一○),秦朝的创业皇帝;汉武帝刘彻(前一五六??前八七),汉朝功业最盛的皇帝。

〔略输文采〕文采本指辞藻、才华。“略输文采”,是说秦皇汉武,武功甚盛,对比之下,文治方面的成就略有逊色。

〔唐宗宋祖〕唐太宗李世民(五九九??九),唐朝的建立统一大业的皇帝;宋太祖赵匡胤(yìn印)(九二七??九七六),宋朝的创业皇帝。

〔稍逊风骚〕意近“略输文采”。风骚,本指《诗经》里的《国风》和《楚辞》里的《离骚》,后来泛指文章辞藻。

〔天骄〕汉时匈奴自称为“天之骄子”(见《汉书?匈奴传》),后以“天骄”泛称强盛的边地民族。

〔成吉思汗(hán寒)〕元太祖铁木真(一一六二??一二二七)在一二○六年统一蒙古后的尊称,意思是“强者之汗”(汗是可汗的省称,即王)。后来蒙古在一二七一年改国号为元,成吉思汗被推尊为建立元朝的始祖。成吉思汗除占领中国黄河以北地区外,还曾向西远征,占领中亚和南俄,建立了庞大的帝国。

〔只识弯弓射大雕〕雕,一种属于鹰类的大型猛禽,善飞难射,古代因用“射雕手”比喻高强的射手。“只识弯弓射大雕”,是说只以武功见长。

2、雨说

为生活在中国大地上的儿童而歌

教学目标

(一)知识与能力

1、了解作者及写作背景。

2、诵读课文,进一步培养学生阅读欣赏现代诗歌的兴趣。

(二)过程与方法

1、引导鉴赏诗歌思路:欣赏诗歌意象意境,理解诗歌内涵主旨、把握诗歌情感,提高审美情趣。

2、学习拟人、联想、想象等手法,提高鉴赏能力。

(三)情感、态度、价值观

体味作者情感,引导学生发现美、热爱美、追求美,培养学生热爱自然、热爱祖国的高尚情操。

、教学重点:

诵读课文,赏析诗歌意象意境,把握诗歌内涵,体会作者感情,提高审美情趣。

教学难点 :学习和运用生动形象、凝练精美的诗歌语言。

教法设计: 诵读感悟、比较鉴赏、拓展迁移

教时安排: 二教时 (第一课时)教学步骤

一、导入新课

“好雨知时节,当春乃发生。随风潜入夜,润物细无声。”这是唐代诗人杜甫的名句。而诗人郑愁予的寸却获得了一种生动活泼的灵气。 1作家简介

郑愁予是台湾诗人,本名郑文韬(tao),祖籍河北,1933年生于山东济南。被称为“中国的中国诗人”。

童年时他跟随当军人的父亲走遍大江南北;抗战期间,他随母亲转徙内地各处。在逃难途中由母亲教读古诗词;1舛9年随家人去台湾,一面学习,一面写作,并受到纪弦的赏识,1955年他毕业于中兴大学法商学院,1963年成为现代诗社中的主要成员。

2、有感情地朗读诗歌。

3.提问学生“雨说”它在诗中出现了几次。以此引导学生理出诗作结构,并和学生一起为诗作的四个部分命名。 a.第一部分:(一至三节) 雨的到来 b.第二部分:(四至五节) 雨的邀请 c.第三部分:(六至八节) 雨的来意 d.第四部分:(九节)

雨的祝福

4.提问:诗作是第几人称写作?是什么修辞手法? 是第一人称写作,是拟人的修辞手法。题目是“雨说”,因此全诗都是用雨的口吻直接叙述,直接抒情。

5.提问:在第一节到第五节中出现过哪些景物? 合作讨论,找出第一节到第五节出现的景物:田圃、牧场、鱼塘、小溪。

6.说明诗作开篇点题,雨在大地的期待中到来,第一节描绘了大地枯旱的景象:田圃荒凉,种子禁锢;牧场枯黄,牛羊绝迹;鱼塘低浅,游鱼缺水;小溪干涸,水落石出。

在教师的讲解中,想象缺少雨水滋润的大地的景象,体会春雨来临所带来的生命气息。

7.概括出诗中雨到来之后大地的景象。

合作讨论,概括雨来临后大地复苏的景象:田圃泥土润如油膏,牧场新苗添绿,池塘鱼儿欢腾,溪水叮咚流淌。

8、齐读第1-5节

提问:雨让大地生气勃勃,那么在诗人笔下,雨是什么样子的?让学生从视觉、听觉、触觉、嗅觉等方面来描述。

可以结合“随风潜入夜,润物细无声”和朱自清的《春》中描写春雨的内容。 回答提问:视觉上,“像丝缕那样把天地织在一起”,表现雨的细密;听觉上,“走得很轻,而且温声细语的”,表现雨的淅沥;触觉上,“亲一亲”,表现雨的轻柔„„

第二课时

教学步骤

1.请学生有感情地朗读诗作,可选择集体、个别、自由朗读的方式。要求:字音要读准;停顿要恰当,句子的层次要分清,做到不添字、不漏字、不倒字。

2.在大地的期待中,雨悄然来到,为万物带来了勃勃生机。提问:雨的家乡在哪里?它是怎么长大的? 雨的家乡在遥远的地方,它是在“自云的襁褓中笑着长大的”。认真听教师的讲解,领会“白云”和“笑着”的深刻含义。(白云是自由的象征,雨在白云中孕育,自由、幸福地成长。)

3.提问:雨做了这一切,它的来意是什么? 它的来意是“教你们勇敢地笑”。引导并点明寓意:雨希望孩子们能够冲破束缚,自由自在、快乐地生活。

4.讲解并提问:第三部分描写了柳条儿、石狮子、小燕子、旗子见到春雨后“笑”的场面,运用什么修辞手法? 运用拟人的修辞手法。柳条儿“笑弯了腰”,石狮子“笑出了泪”,小燕子“笑斜了翅膀”,旗子“哗啦啦地响”。

5.组织学生讨论:如何理解“只要旗子笑,春天的声音就有了/只要你们笑,大地的希望就有了”这句话? 雨的到来给万物带来欢笑,雨是自由和快乐的代言人,因此,只要孩子们能够勇敢地用笑声表达出追求幸福、快乐的愿望,大地就有希望了。

6.提问:最后一节诗中哪些词句暗示了雨的命运? “不再回去”、“快乐地安息”都表明了雨的命运。 7.提问:雨的祝福是什么?如何理解? 雨的祝福是孩子们“吃着苹果擦着嘴”,意指享受到自由和幸福的生活。 8.讲解诗歌的寓意:

诗人借雨融入大地的现实,赋予雨以象征意义??牺牲自己的生命,为孩子们带来追求自由、幸福的信念和理想。 本课总结

本诗于1979年创作的自由体新诗。全诗共九节,以题目“雨说”开篇,并通过“雨说”的反复,贯穿诗篇,形成前后呼应的结构。诗人以拟人化的手法,赋予春雨以说话能力,采用第一人称与儿童对话,塑造了~个温柔亲切的“爱的使者”的形象。全诗洋溢着轻松愉快、清新活泼的气息。通过这首诗作的学习,大家应该初步了解自由体新诗的特点,感受到诗人深厚的古文功底和对现代性创作技巧的把握。 作业:

1、

熟读并朗诵诗歌。

2、

完成课后练习。 板书设计

3、星星变奏曲

教学目标:

(一)知识目标

1、了解朦胧诗派的代表人物、作品,及其主要艺术风格和成就。

2、理解诗歌中星星、静夜、大地等物象的象征意义,学习比拟、对比等修辞手法。

(二)能力目标

1、学习朗读朦胧诗歌的技巧,学会品味诗歌语言,体会作者字里行间流露出的感情。

2、学会比较阅读的方法,通过诗内物象的对比,理解诗歌主旨。

3、仿照诗歌结构,开展诗歌写作的活动。

(三)情感目标

1、理解诗人对现实的否定和对光明的向往,以及即使面对重重磨难也不放弃希望的精神。

2、体会诗人经历人生苦难之后对诗意人生和自由精神的向往。教学重难点

(一)教学重点:

1、了解朦胧诗的审美特征。

2、理解诗歌所寄托的诗人的理想和信念。

(二)教学难点:

理解“星星”的象征意义。 课时安排

一课时 教学过程

一、题解与导入

大家都学过著名诗人郭沫若写的《天上的街市》。这首诗以丰富的想象,写了热闹自由的天上街市,通过写牛郎织女的自由来往来表达作者对美好生活的向往。今天我们来学习《星星变奏曲》这一首诗,体会作者所要表达的思想感情。

二、朦胧诗

是新时期的一个非常重要的文学流派,是*后期一群自我意识开始觉醒青年,利用诗歌的形式对现实进行反思和追求诗歌独立的审美价值的产物。作者江河,是变革时代的中国新文学史上的“朦胧诗”运动的主将之一。 (放录音带)

三、读诗歌,整体感知、讨论

1、诗中的星星象征什么?诗人反复咏叹星星表达了怎样的渴望? (象征着光明,象征着美好的事物,表达了他对光明的渴望)

2、两小节诗在结构上及语言上有什么异同点?

(都用“如果-----谁还需要---还会---”开头;从第五行看,有变化,用“谁不愿意”和“谁愿意”)

四、问题研究

1、这首诗为什么题为《星星变奏曲》?

这是借用音乐术语来说的,它提示这首诗是围绕一个主题与主要意象变换情境,委婉抒情。

诗的上下两节结构相称,基本手法和用语相同或相近,有同样的韵律。但上下两节的情境,意味发生了变化,如用“谁不愿意”和“谁愿意”就不同了。(句式、词语)这种情形如同音乐,两段录音曲在保持基本轮廓的基础上出现一点变化。

2、诗中多用比喻,理解一些句的寓意:如:“每一首诗都是一群颤抖的星星” “风吹落一颗又一颗瘦小的星星”

五、练习:根据练习三要求写几句诗。练习二:

1、“柔软得像一片湖”,把夜晚的安宁、平静、柔和、温馨和自由在用形象的语言表达出来,给人以温柔如梦的美好感觉。

2、这是将“鸟落满枝头”与“星星落满天空”形成类比联想,以想像中春意盎然,充满生机的景象与缀满繁星的景象类比,使人意会到令人向往的诗一般的生活境界。

3、这两句诗,分别从听觉上和视觉上描写声音的隐约、白丁香的朦胧,将人们带入一种令人陶醉的朦胧迷离的美好意境,表现了“春天”之美,“光明”之美。

4、外国诗两首

蝈蝈与蛐蛐

教学目标: 1.知识目标

了解济慈和叶赛宁及其田园诗的特点 2.能力目标

理解诗歌内容,把握匠心构思揣摩语句,领悟诗歌的意境美和声韵美,提高学生的审美意识。 3.情感目标

通过朗读,感受诗歌形象,体会其感情意蕴

确定以上的三个目标是要体现“新课标”提倡的“知识与能力、过程与方法、情感与态度”并重的教学理念。

四、教学重点与难点

(一)教学重点

理解诗歌内容,把握诗歌的构思。

(二)教学难点

感受诗歌形象,体会其情感意蕴。

五、教法与学法

(一)教法

1,诵读法引导学生感悟诗歌,把握诗歌主旨及诗人的情感意蕴。 2.感悟法引导学生感悟理解诗歌的具体内容、语言特点等。

3.比较法运用同题材诗的扩展阅读,培养学生对诗歌的鉴赏能力。 课时安排

1课时。 教学内容和步骤

一、导入新课。

同学们,这节课再向大家介绍一篇浪漫主义诗人??济慈(板书)创作的十四行诗《蛔蝈与蛐蛐》(板书)。

二、教读新课。

1.指名朗读这首诗。正音。

2.简介作者和时代背景。引导学生阅读注释①,教师补充。

济慈(1795?1821),英国诗人。1795年10月29日生于伦敦。9岁时父亲去世,母亲改嫁,济慈和两个弟弟由外祖母收养。15岁时母亲又病故,外祖母委托两名保护人经营他们兄弟的财产。起初济慈是学医的,他喜好文学,但对医学并不厌弃,通过考试获得

内科医生执照,还继续学习外科。直到1816年11月间,济慈才决心放弃医学,从事文学创作。他的诗诗中有画,色彩感和立体感都很强。他认为,一个大诗人对美的感受能压倒或抵消一切其他的考虑,这就是他的“天然接受力”的思想。他曾说,他可以深入到一只麻雀的性格中去,同样可以“在瓦砾中啄食”。济慈是英国浪漫主义诗人中最有才气的诗人之一,他的诗对后世的影响很大,维多利亚时代诗人、唯美派诗人、“意象派”诗人都受到他的影响。可惜正当他初展宏才的时候,就不幸与世长辞,时年才25岁。

3.学生朗读,整体把握诗歌的基调,注意处理朗读语调,注意音步和轻重音的正确掌握。

4、这首诗在构思上有什么特点? (提示:开头一句与“大地的诗歌呀,从来没有停息“相对应分为两部分;动静相衬,意境优美。)

盛夏//蝈蝈的乐音

大地的诗歌//充满生机

严冬//蛐蛐的歌儿

5.理解虫鸣的象征意义。

诗人创造蝈蝈和蛐蛐的意象,表露了济慈的创作思想,即诗应该给人以安慰,并提高人们的思想。蝈蝈和蛐蛐充满活力,它们的鸣声周而复始,永不停息,那就是大地的诗歌从来不会死亡,鼓舞人们乐观地对待生活。

6、尾声:“从蛐蛐的歌中仿佛听到了蝈蝈的呜叫”。理解盛夏的蛔蝈和冬天的蛐蛐是怎样联系起来的。这是诗人的联想,将冬天的蛐蛐的歌儿呼应盛夏的蝈蝈的乐音。点出起伏不停,点明中心:从来不会死亡,从来没有停息的大地的诗歌。

三、布置作业。熟读成诵。

在理解诗的思路结构的基础上背诵这首诗。

一、作者简介:

叶塞宁,俄罗斯诗人,他的诗感情真挚,格调清新,并擅长描绘大自然景色。

二、赏析,归纳这首诗的内容,并体会诗歌所描写的意境美 第一节:写夜的静谧

第二节:写溪水的歌唱 (以动写静)

静谧、美丽 第三节:写月光下大自然的美丽 第四节:再次写到美丽的月色

三、结合课后练习二训练:

1、用自己的话,描绘所写的夜景

2、这首诗传出诗人怎样的心境

传达出诗人对大自然的热爱和他安适、宁静的心境。

四、反复朗诵这首诗,体会诗中所描绘的美景。五作业 课后练习一

5、敬业与乐业

教学目标

知识和能力:

学习和积累文中运用的大量经典语句、格言;初步了解、能够判断“举例子”和“讲道理”两种论证方法;体会演讲语言口语化的特点。 过程和方法:

初步学会阅读逻辑性比较强的议论性文章。在整体感知过程中逐步理清行文思路;联系语境、借助注释理解一些古代名言的含义。 情感态度和价值观: 树立对事业的责任心和热爱事业的态度。能联系自身生活经验对“敬业与乐业”精神有所品味与领悟、反思与批判。

教学重点、难点

1.理解这篇讲演词的结构安排,体会层次分明、条理清晰的特点。

2.体会语言表达通俗浅显、准确周密,生动有力的特点。 教学时数

两课时 教学过程

第一课时

一、导入新课

《敬业与乐业》是梁启超七十多年前,对上海中华职业学校学生的一次讲演,虽然时间已经过去七十多年了,但梁启超先生所讲的内容对我们仍然很有教育意义。

二、简介作者

梁启超(1873-1929),中国近代维新派领袖,学者。字卓如,号任公,又号饮冰室主人。广东新会人。清光绪举人。和其师康有为一起,倡导变法维新,并称"康梁"。著述涉及政治、经济、哲学、历史、语言、宗教及文化艺术、文字音韵等。其著作编为《饮冰室全集》。

三、指导学生阅读课文

明确:课文共七段,总体结构是"总一分一总"式:开头总起全篇;然后分别论述敬业和乐业,两个分论点平行并列;最后总结全篇。条理清晰,纲举目张。全文可分为三个部分。

第一部分(第1段):揭示全篇论述中心。

第二部分(第2~6段):论述敬业和乐业的重要。

第三部分(第7段):总结全篇,勉励人敬业乐业。

四、学习课文内容

(一)学习第一部分。

思考:讲演词开头交代题目的来源其用意是什么?哪句话揭示了全篇论述的中心?

学生讨论、交流。

明确:文章开头先交代题目的来源,用意在于说明题目虽然来源于古籍《礼记》和《老子》里的两句话,但跟原文语句的本意不同,是"断章取义"造出来的。这样交代既可避免听者把题目和《礼记》、《老子》里面语句的原意混同了,又可显示出论述的科学性,讲演的趣味性;同时,提出题目来由,也便于揭示全篇的论述要点,使听者对讲演的纲目有一个大致的认识。

(二)学习第二部分。

1.朗读第

三、四段。课文第

三、四段是论述"敬业"的重要,可作为第二部分的第一层。

思考:(1)作者怎样阐述什么叫"敬"?(2)如何论述怎样才能做到"敬业"?

学生讨论、交流。

明确:第三段作者阐述什么叫"敬",引用了朱熹的名言"主一无适便是敬"后,紧配上自己的通俗化的解释:"凡做一件事,便忠于一件事,将全副精力集中到这事上头,一点不旁骛,便是敬。"然后顺理成章地论述凡人类的正当的劳动、正经的事,"其性质都是可敬"的,并举"当大总统"与"拉黄包车"这两件事加以证明,侧重点在"凡职业没有不是可敬的","因自己的才能、境地,做一种劳作做到圆满,便是天地间第一等人。"

第四段论述怎样才能做到"敬业",先引用《庄子》中的语句并作阐述,再举木匠做成一张好桌子和当政治家的建设成一个共和国家,挑粪的把马桶收拾得干净,和当军人的打胜一支压境的敌军这两组例子,说明无论做什么都要"丝毫不肯分心到事外"(即"敬业")。接着从反面论述"一个人对于自己的职业不敬"的害处,指出"敬业主义,于人生最为必要,又于人生最为有利",并引用庄子、孔子的名言,进一步强调敬业的重要。举例和引用的自然结合使文章的说理立于不败之地。

2.朗读第

五、六段。这两段论述"乐业"的重要,作为第二部分的第二层。

思考:作者举了怎样的例子来论述"乐业"的重要,又是如何论述"乐业"的道理的?

学生讨论、交流。

明确:作者在第五段中剖析了生活中常会遇到的感叹"做工苦"这个实例,说明要"从劳苦中找出快乐来",很自然地点到了"乐业"的"乐"。第六段,更深入一步,论述"乐业"的道理,分四点说明"凡职业都是有趣味的",这是作者的经验之谈,令人信服。紧接着又引用孔子的两句名言进一步证明"人生能从自己职业中领略出趣味"(即乐业),"生活才有价值","这种生活"才算得"人类理想的生活"。

五、小结

讲演词的第二部分内容理解的重点是阐述什么叫做敬业,为什么要敬业,怎样做才算是敬业。

六、布置作业

1.探究练习二。

2.选用随堂练习。

第二课时

一、复习旧课,检查背诵

学生背诵"我老实告诉你一句话„„真算得人类理想的生活了。"

二、继续学习课文第三部分

学生朗读第七段。

明确:作者根据自己的亲身经验,指出"责任心"和"趣味"跟"敬业"与"乐业"的关系最为密切:"责任心"就是"敬业"广趣味"就是"乐业"。他认为做事必须具备责任心和善于"从职业中领略出趣味。"一个人对待自己的职业,能够有责任心、有趣味发扬敬业与乐业的精神,专心致志做圆满,就是"人类合理的生活"。结束句"我望诸君和我一同受用!"用短句,用感叹的语气,充满了对听讲的青年学生的殷切期望,感情强烈而真挚。

三、探究活动

(一)思考:作者在文章中提出了许多有关敬业和乐业的观点,你最信服的是哪一点?你有没有不同意见?说说你持这种意见的理由。

学生小组合作,讨论、交流。

明确:文章中的观点如下:

1."敬业乐业"四个字,是人类生活的不二法门。

2.人类一面为生活而劳动,一面也是为劳动而生活。

3.凡职业没有不是神圣的。

4.凡职业没有不是可敬的。

5.人生在世,是要天天劳作的。

6.劳作便是功德,不劳作便是罪恶。

7.因自己的才能、境地,做一种劳作做到圆满,便是天地间第一等人。

8.凡做一件事,便把这件事看作我的生命,无论别的什么好处,到底不肯牺牲我现在做的事来和他交换。

9.敬业主义,于人生最为必要,又于人生最为有利。

10.从劳苦中找出快乐来。

10.凡职业都是有趣味的,只要你肯继续做下去,趣味自然会发生。

12.每一职业之成就,离不了奋斗。

13.因为每一职业之成就,离不了奋斗;一步一步的奋斗前去,从刻苦中将快乐的分量加增。

14.人生能从自己职业中领略出趣味,生活才有价值。

15.敬业即是责任心,乐业即是趣味。 „„

学生的回答无论是赞同还是不赞同,都可以;关键在于言之有物,言之有理,言之有据。目的在于通过讨论加深对文章的理解。

(二)做资料卡片。

让学生找出文中自己喜欢的句子或含义深刻的句子,制成资料卡片,要注明出处以及卡片的类型等;书写要清楚规范。

四、小结

1.中心意思:这篇讲演针对听讲者的实际情况,提出了"敬业与乐业"的论题,深入地论述了敬业与乐业的重要性,殷切地希望大家发扬敬业、乐业的精神,去过人类合理的生活。

2.写作特点。

(1)论证条理清晰,论据生动有力。这篇讲演词开头提出了论题,中间主体部分分两层论述敬业和乐业的重要,末尾总结全篇。条理很清晰。证明论点的过程,举了多种论据:有生活中的实例,有古代、外国著作中的事例,有作者亲身经历中卓有成效的经验,还有古人的流传至今的名言警句等。这些论据的精选运用,使讲演词具体、生动,富有说服力。

(2)语言通俗浅显,准确周密,概括有力。全篇讲演用简明的短句,很少用长句;多用口语,明白如话,一听就懂;引用古代文句时,注重化深为浅。同时文中多次用了设问句和反问句,引用不少古代名言警句,又自己提炼了许多精辟的警句,都使语言显得概括有力。

五、布置作业

1.

背诵课文第六段。 2.

选用随堂练

6.纪念伏尔泰逝世一百周年的演说

教学目标

1、领略两位杰出人物的思想光辉。

2、品味雨果包含激情、诗化的语言。

3、了解演说的特点。教学过程

一、导入

从题目上我们可以知道这是维克多,雨果为纪念伏尔泰百年(1778-1878)冥诞的演说词。为什么要纪念伏尔泰?(他有哪些贡献)下面我们一起来听听,雨果的演说。

雨果,法国作家,父亲是拿破仑军官。

伏尔泰,法国作家、哲学家、启蒙思想家。出生于资产阶级家庭。从小喜爱文艺。中学毕业后,因作诗讽刺宫廷而遭囚禁。在狱中完成第一个悲剧《奥狄浦斯王》,出狱后因宣传启蒙思想,多次受迫害并被放逐国外。为避开当局追捕,晚年避居法瑞边境,积极参加政治斗争。

二、整体感知

1、从雨果的演讲中,可以了解到伏尔泰为人类文明做出了怎样的贡献?

伏尔泰以猛如雷电的笔写下不朽的文章,提示封建君主的残暴,法官、神甫欺压人民的本质,教导人民从混沌愚昧中起出来,推翻君主统治,开创一个民主文明的法兰西。

2、雨果为什么把伏尔泰生活的时代看作是“一个新的纪元”的开始?

在伏尔泰生活的时代前期,统治阶级压迫人民,而人民却甘心受迫,不敢、也不懂起来反抗,社会愈加昏暗腐败。这时许多像伏尔泰这样的启蒙主义思想家用自己的写下了不朽的篇章,唤起了人们反抗的意识,投入到为民主而斗争的自由中去,所以,雨果把伏尔泰生活的时代看作是“一个新的纪元”的开始。

3、作者认为“伏尔泰不仅是一个人,他是一个世纪”的原因是什么?

伏尔泰的一生经历了达到极点的专制时期和刚刚露出一线晨曦的革命时代,伏尔泰比国家领袖更重要,他是思想的领袖。

4、“对于我们每个人来说,进步的两个方面很清楚地显示出来。“其中”两个方面“是指什么?

第一:做一个人,我们要行使自己的权力,第二:做一个公民,我们要恪尽职守。

三、品味语言。结合课后习题三

四、小结:

这篇演说,是对人类先贤的崇高礼赞,对思想与人格力量的热情颂扬,也是对人类社会永远需要正义的呐喊、对良知的呼唤。

五、作业

选择你最喜欢的部分背诵。

7 傅雷家书两则

教学目标

一、教学目标

知识目标

理解傅雷所说的“坚强”的含义,认识到敢于面对现实、勇于回顾历史、理智地对待成功并继续奋斗,才是真正的“坚强”。

过程和方法

体会信中讲述道理和抒发感情的两种方式。 情感态度和价值观

能够领悟突破挫折和成功、以平和心态面对现实和过去、以积极心态迎接未来的人生境地。

二、教学重难点

1、读懂两封家书,体会到其中不同的期望和情感。

2、能够准确而深刻地理解“坚强”的双层内涵。

3、研读词语,体会含义和感情。

三、教学方法

自由、合作和探究 课时安排 2 教学步骤

第一课时

1、复习书信的格式,导入新课

2.这两封信是傅雷接到傅聪的信后的回信。请浏览这两封信,说说它们的主题各是什么? 第一封信,如何面对挫折,第二 封信,如何对待成功。 3.第一封家书的开头称呼是什么?为什么要这么称呼? 称呼是“聪,亲爱的孩子”,因为傅聪 当时正是精神消沉的时候,这样温情的称呼,会给傅聪带去父母的爱。

4.在儿子面对挫折和心灵的苦闷时,作为父亲,傅雷是如何劝解的? 提示:注意课文中运用的比喻和引用的故事。

首先是宽慰,不必为父母担心。 其次肯定两点:一是父母永远都是孩子倾诉苦闷的对象;二是人的一生就是在情绪的起伏中渡过。最后以过来人的身份,给予傅聪如何面对情绪跌宕的建议。

5.傅雷运用太阳、雨水、五谷、庄稼的比喻和克利斯朵夫的故事,他的用意是什么? 总结:要学会保有平和的心态,控制情绪的起伏。然后,要保持冷静的头脑,理智地分析,吸取教训,引以为戒。(太阳、雨水、五谷和庄稼的比喻,是说明任何事情都要掌握分寸,)

第二课时

1.第二封信,写在傅聪获奖之后。在信的开头,傅雷把儿子的信比喻成什么?表现了什么样的感情? 比喻成新年中的“礼物”。表现了傅雷夫妇对儿子的信的盼望和接到信后的喜悦。 2.第二封信中,傅雷是如何表达对儿子成功的激动的? 结合奥运冠军的感受,体会傅雷作为父亲,毫不矜持地直抒胸臆,表达对儿子成功的喜悦之情。傅雷把自己的激动和喜悦,分为三个层面:父母对孩子的成功感到幸福;欣赏艺术的喜悦;为了祖国的荣誉而 激动。

3.傅聪的成功,使傅雷异常激动,但是他在信中是否只是停留在这种喜悦之情中?他还写了什么? 他并没有只停留在这种喜悦之情中。他还称赞了傅聪面对掌声、赞美的冷静。 4.傅雷用“我更高兴的更安慰的是”起到什么作用? 这句话承上启下,从对儿子艺术成功的喜悦之情延伸到对儿子思想成熟的欣慰之情。使傅雷更高兴、更安慰的是傅聪没有被成功冲昏 头脑,没有因为暂时的成功减少对艺 术的探索和追求。

5.在两封信中,傅雷都提到了“坚强”。他对于“坚强”的理解是什么?你能够找到两封信中,对“坚强”内涵的直接解说吗? 关于“坚强”的解说:第一封信中“一个人惟有敢于正视现实,正视错误,„„越来越坚强”,第二封信中“你能坚强(不为胜利冲昏了头脑 是坚强的最好的证据),只要你能坚强,我就一辈子放了心!”

总结:在傅雷看来,坚强,既是能够面对人生挫折、情绪跌宕,更是能够正确对待掌声、赞美,用一种平和盼心态,宠辱不惊,胜不骄,败不馁。 6.在傅雷看来,坚强的最高境界是什么? 是拥有二颗赤子之心。

7.怎么理解“赤子之 心”? 提示:抓住赤子之心的纯洁。

赤子之心,是艺术创造的最高峰。而这种心灵的纯洁是人类共同的美好愿望。

8、傅雷结合音乐院长对傅聪的评价,以及舅舅关于傅聪小时候的记忆,提出自己的希望和建议。你能够找到吗? 希望傅聪做新中国的钟声,把中国的古老文明传递到世界艺术殿堂的各个角落,并且指出:中华民族是一个具有诗人灵魂的民族,因此音乐的表现力应该增加“气吞斗牛”的阳刚之气。

9.组织讨论:在信的末尾,傅雷又回到了情绪的“矛盾与快乐”问题,比较第

一、二封信中傅雷关于如何面对人生情绪起伏的解说。

第一封信里,他说人生必定充满了情绪上的起伏跌宕,惟有庸碌的和超然的人才不会浮沉;第二封信中,他强调“有矛盾正是生机蓬勃的明证”,以此鼓励儿子,要勇敢面对各种各样的矛盾,在不断解决矛盾的过程中不断提高自己,趋向“完美”。

评价:作为一位伟大的学者,也是一位伟大的父亲,傅雷的伟大之处就在于,他借助儿子成功的机会,给予了他更多的艺术砥砺,向他昭示出更高的人生境界。 总结

这是两封普通而又不寻常的家书:普通,在于它们是一位父亲写给身在异国他乡的儿子的内心告白;不寻常,在于这两封家书,从儿子的日常生活出发,既给予儿子艺术的教诲,更昭示出崇高的人生境界。傅雷和傅聪,已经超出父子、朋友的界限,成为艺术上、思想上的知音。 作业

讨论完成课后练习

1、傅雷运用古战场的比喻,目的是什么? 提示:对于第一封信的学习,要抓住傅雷运用的比喻和引用的克利斯朵夫的故事。

凭吊古战场时,烈火硝烟散尽,只余断壁残垣,金戈铁马、血肉厮杀都已被岁月的黄沙掩埋。这时候,即使亲历过战斗的人,纵有万千感慨,也不会再像当初那样身不由己、欲死欲活。这个比喻,十分真切地表达出所有“过来人”回首“过来事”的心情 ??苍凉而平静,沉郁而超然。

8、致女儿的一封信

教学目标: 知识与能力:

1、自学理清书信的基本格式;

2、合作体会不同对象、不同目的,所选择的不同表达方式;

3、品味交流思想的不同语言风格 过程与方法:

自读理解积累,合作导引释疑,学习创作交流。 情感态度与价值观:

了解“爱情的真谛,感受伟大爱情的美好;

进行爱情教育,帮助学生走好青春第一步,正确把握萌动的情感。 教学设想:

这篇课文是苏霍姆林斯基写给女儿的信,是作者对女儿进行的一次生动的爱情教育。文字朴实晓畅,语言生动形象,昭示了爱情的真谛。爱情这个火辣辣的词,撞击着每个青年的心扉,人在花季,多彩的心事就像爬山虎攀上了年轻人的心房,稍不留神就容易踏上早恋的航船,教书育人,是语文教育应尽的责任。鉴于这种考虑,拟对教材作如下 教学重点和难点:

利用课文内容对学生进行一次爱情教育;让学生明白只有懂得爱的真谛,才会拥有幸福快乐的人生。 教学时数:一课时。

教学方法:讨论研讨相结合,以学生自主学习为主。 教学内容与步骤:

一、课前活动

1.查找关于作者苏霍姆林斯基的介绍,了解有关书信的写法; 2.搜集有关爱情的名言警句; 3.收看古今中外的经典爱情故事:《孔雀东南飞》《梁山伯与祝英台》《魂断蓝桥》 《罗密欧与朱丽叶》《泰坦尼克号》《廊桥追梦》《乱世佳人》等等。

二、导入新课

自从盘古开天地,三皇五帝到如今,爱情 一直是人类传唱不衰、古老而又新鲜的话题,地老天荒,海枯石烂,由爱情而演绎的故事, 更是那样的惊心动魄。“在天愿做比翼鸟,在 地愿为连理枝”,这是唐代诗人白居易的诗句。

无论哪个时代,爱情都是一生一世的许诺。醉 过方知酒浓,爱过方知情重。同学们,人在花 季,多彩的心事也许正困扰着你,是呀,人间 有情,红尘有爱,这令人魂牵梦绕的情丝正等着你成长!但是,你真正懂得爱情的真谛吗?

到底什么是爱情?今天大家也许能从苏霍姆林 斯基的《致女儿的信》一文中得到启示。 新课内容:

1.学生自读课文,完成基础知识的积累(媒体辅助教学): 苏霍姆林斯基是(国籍),出身在一个农民家庭,一生著作颇丰,主要有《学生的精神世界》、《全国发展的人的培养问题》。苏霍姆林斯基的辉煌成就给他带来了很高的声誉。1957年他39岁时被选为俄罗斯教育科学院通讯院士,1959年荣获功勋教师称号。他还荣获两枚列宁勋章,多枚乌申斯基奖章和马卡连柯奖章。不少国家的教育领导机构和有影响的学术团体,纷纷邀请他出国讲学。1968年他再次当选为苏联教育科学院通讯院士,p年9月被选为全苏教师代表大会代表并荣获社会主义劳动英雄称号。他作为一位有独创精神的教育家被载人苏联史册。成为世界著名教育家。 2.课文内容研讨:

门面对女儿的问题,作者没有轻下结论,他是怎样解释“什么是爱情”的?作者到底认为什么是爱情?请用自己的话概括一下。

(2)作者为什么不直接告诉女儿“什么是爱情”?这样写有什么好处?

(3)怎样理解“但做一个幸福的人,只能是在你成为有智慧的人的时候” 这句话的含义?

(4)有一天,柏拉图问老师苏格拉底什么是爱情,苏格拉底叫他到麦田里走一遭,在走的过程中不能再回头重走,在途中可以摘一棵最大最好的麦穗,但只能摘一棵。柏拉图觉得此事很容易办到,便充满信心地往前走,谁知过了半天没有回来。最后,他垂头丧气出现在老师跟前,诉说空手而回的原因:难得看见一株看似不错的,又不知是不是最好的;因为只可以摘一株,便挑挑捡捡,总希望前面有更好的;等到走到尽头时,才发现手上一棵麦穗也没有。这时,苏格拉底告诉他:这就是爱情!

课文也是用故事的形式诠释什么是爱情这个大问题。爱情是美好的,是人类历久常新的话题,对于爱情,你是怎样理解的?(学生也可以用故事的形式解释。) 3.积累:爱情是这样美好,是人类最伟大的情感,看看名人是怎样看待爱情的:要求学生展示课前准备好的资料(教师也准备一些)。 青少年时代是人一生中生命的曙光,这一时期需要积蓄力量,去创造美好的东西,其中包括为人类明智而勇敢的爱所创造的精神力量。是的,孩子,你要深思,需要有自己的力量,包括爱情的力量,这种力量需要坚持终生。珍惜它、爱护和发扬它直到生命结束,使之成为专一不二忠贞不渝的精神力量,避免犯错误和遭到失望。我把爱情称之为明智的、勇敢的力量,只有这样,才能算是人类真正的爱。 爱情只有是理智的,聪明的,审慎的,它才是高尚的。 爱情,是一种炽烈的感情,一定要让理智做心灵的主宰。 ----苏霍姆林斯基《爱情的教育》

那种用美好的感情和思想使我们升华并赋予我们力量的爱情,才能算是一种高尚的热情;而使我们自私自利,胆小怯弱,使我们流于盲目本能的下流行为的爱情,应该算是一种邪恶的热情。法国作家乔治?桑 真正的爱情是专一的,爱情的领域是非常的狭小,它狭窄到只能容下两个人生存;如果同时爱上几个人,那便不能称做爱情,它只是感情上的游戏。 ----德国诗人、剧作家席勒

爱情不会因为理智而变得淡漠,也不会因为雄心壮志而丧失殆尽。它是第二生命;它渗人灵魂,温暖着每一条血管,跳动在每一次脉搏之中。 ----英国散文家、文学评论家艾迪生

爱情是一片炽热狂迷的痴心,一团无法扑灭的烈火,一种永不满足的欲望,一分如糖似蜜的喜悦,一阵如痴如醉的疯狂,一种没有安宁的劳苦和没有劳苦的安宁。 ----英国学者理查德.弗尼维尔。

三、拓展

爱情,不同时期被赋予不同的内容,为了更深入地理解爱情的真谛,我们再读下面一篇文章。 三个故事

很复杂,其实也很简单。

爱情是什么?朋友给我讲了这样三个故事。

第一个故事很凄美:在医院病房的一隅,躺着一位奄奄一息的病人,在世间弥留的最后一刻,他把苍老的手伸给浊泪涟涟的老妻,感谢他半个世纪以来对自己无微不至的照顾。他深知自己不久就要踏上黄泉之路了,为了报答他的真情,也为了安抚自己的心灵,他决定告诉她一个深埋在心底的秘密。但是,没等他把这个秘密说出口,他的老妻就把手轻轻按到了他的嘴上、她说:

“我不需要听什么爱的秘密,在我看来,最大的爱的秘密就是我们在茫茫人海中相识、相爱,手牵着手一道走过了50年的风雨历程„„ ” 丈夫感动得涌出了热泪,最后带着那永远的秘密平静地离开了人间。

第二个故事发生在很久以前。一座城市居民经常和另一个城市的居民打仗,很多年都不分胜负。这两座城市,一座坐落在高地,另一座坐落在洼地。

处在高地上的城市头领想出了一个令人心酸的绝人之道:把高地的水库打开。结果可想而知,洼地的城市被大水围困,注定要灭亡。水库打开,哭叫之声不绝于耳。看到如此的惨状,围困者做出了一个人道的姿态,派船去救落水人,但是,派遣的船只不多,只能容纳极少一部分人,他们的头领喜欢女人,所以只能让妇女上船,并要求只能带自己最喜欢的一样东西。有的带上自己的玉镯;有的带上自己的金银首饰;有的带上自己的铜镜„„她们想这样既保住了自己的生命,又不失自己的财产。惟有一位妇女肩扛着自己的丈夫,奋力上船。一个士兵阻拦道:“船上只允许上妇女,不许运男人。”那位妇女说:“这就是我最喜欢的东西。”士兵无言以对,只好乖乖地让她上船。在那次水患中惟一幸存的男人就是那位妇女的丈夫。

朋友讲的第三个故事是一家电视台的综艺游戏节目:摸手认妻。这个游戏由6对夫妻参加,要求把6位男士的眼睛蒙上,让他们依次去摸6位女士的手,从而摸出自己的妻子。

前3位都模错了,惹得现场观众哈哈大笑。第四位上场了,主持人依然是一丝不苟地将那位男士双眼蒙上,接着原地转了三圈,同时又示意六位女士互相调了一下位置,过后搀着那位男士来到一字排开的六位女士面前说:开始。那位男士伸出手抓住第一位女士的手,也就三四秒的时间,对身边的主持人说这不是我的妻子,接着摸第二位女士,当他抓住第二位女士的手后,比第一次时间稍长一点,大概有十来

秒时间,他对主持人说:“她是我妻子。” 说后,主持人将那位男士眼睛上的纱布去掉,这期间,现场的观众已经向这位男士报以热烈的掌声了。无疑这位男士猜对了。主持人不失时机地问那位男士:“你摸得这么准,其中有什么秘密?”那位男士对着话筒,郑重其事地说:“没有什么秘密,我凭的是手感,是结婚十多年来培养出来的手感。”这时画面上出现了那位男士妻子的脸,此时那位幸福的妻子已是热泪滚滚。

“爱你一生一世足矣,我想,这就是爱情。” 朋友最后说道。

四、讨论

同学们至此应该明白了,爱情是人类最炽热的情感,最美好的情愫,每个人都渴望幸福,向往爱情。然而过早摘取爱情的果实,我们能收获甜美吗?可以就早恋现象谈谈自己的认识。

五、总结

“爱情是什么”是个既简单又深奥的问题,作者用充满诗意的故事告诉我们:万物生存、繁殖、传宗接代,彼此忠诚,永远思念,这就是爱情,是人类才有的爱情。爱情只有是理智的,聪明的,审慎的,它才是高尚的,才会使人感受到幸福。青春不是爱情的季节,这时的我们还没有足够的能力去承担。

六、拓展活动

同学们,爱是美好的情感,人的全部道德都可以在爱情上反映出来。爱情是人类文明的产物。根据自己的理解,你以自己喜欢的形式,说说什么是爱情。可以是一句箴言,也可以是一段随笔,更可以是一篇文章。

9、

故乡

教学目标:

知识目标:了解小说的特点

能力目标:学会分析人物形象 情感目标:理解文章主题

教学重点:

分析人物,评价人物,探讨人物与主题的关系。

教学难点:

1、人物的肖像描写及其作用

2、故乡的"变"说明了什么 课时安排

3 教学过程:

一、导入

我们在前几册课本中已经学过了不少小说,如《最后一课》、《社戏》等,小说以刻画人物形象为中心、通过完整的故事情节和典型的环境描写来反映社会生活。

1、一篇小说必须具备哪三个要素?

人物、情节、环境是小说的三要素,塑造人物形象是反映社会生活的主要手段,因此,人物是三要素中的主要要素。

2、常见的塑造人物形象的方法有?

肖像、心理、行动、语言描写,以及正面、侧面描写。

3、小说情节起着展示人物性格、表现作品主题的作用,它一般包括:

开端、发展、高潮、结局,有的还有序幕、尾声。

4、环境:A社会环境:主要交代作品的时代背景。B自然环境;用以烘托气氛、表现人物感情等。

5、时代背景

辛亥革命后,封建王朝的专制政权被推翻了,但代之而起的是地主阶级的军阀官僚的统治。帝国主义不但操纵了中国的财政和经济命脉,而且操纵了中国的政治和军事力量。由于这双重的压迫,中国的广大人民,日益贫困化。这篇小说是鲁迅在1919年12月初,回故乡接母亲时亲眼看到故乡的破旧不堪和农民生活的贫困,百感交集,思绪万千,一年后就以这次经历为素材,创作了小说:《故乡》。

二、整体感知

1、听课文录音,熟悉课文,思考小说的线索,情节结构。

小说以时间为序,以我回故乡的所见所闻为线索展开故事情节,全文可按“渐近故乡??在故乡??离开故乡三个方面分为三个部分:

一、交代了时间、地点、回家原因及心情。

第二课时

二、“我”在故乡期间的见闻和感受,

1、老屋的寂寥,决定搬家。

2、回忆与少年闰土的友情。

3、“杨二嫂”的变化。

4、闰土的变化。

三、“我”怀着深深的失望与痛苦的心情离开故乡,把希望寄托于未来、寄托于下一代。

三、人物形象

①文中写了哪些人物?在这些人物中,其中哪些人物刻画得较详细? 文中塑造的人物有:闰土、杨二嫂、“我”,“我”母亲、水生、宏儿;其中闰土、杨二嫂、“我”,刻画得比较详细。

②分析人物的性格特点,然后用一两句生动的话对人物作一评价。

少年闰土中年闰土

外貌淳朴天真、可亲可爱受尽生活折磨、命运悲惨 动作语态聪明勇敢、活泼开朗迟疑麻木、痛苦难言

对“我”的态度情真意切亲密无间被封建礼教牢牢束缚 对生活的态度天真活泼、对生活充满热情和 希望将希望寄托神灵

闰土(1):多子,饥荒,苛税,兵,匪,官,绅,如同一把把挫刀,在时间这块励石上,活生生地把一个少年小英雄磨成了一个木偶人。

闰土(2):一声“老爷!”就把自己与儿时的好友隔在了无形的厚障壁的两边;带回家去的香炉、烛台何尝不是在增加这厚障壁的高度呢! 杨二嫂:一个被社会扭曲了的变形人,不仅表现在从当年的“豆腐西施”变成了今天的“圆规”,而且还表现在自己被侮辱、被损害的同时,又在侮辱和损害着别人。

“我”:曾使自己魂牵梦萦的故乡,一下子就显露出了它的丑陋;为使它美丽,只有在本来没有路的地方走出一条新路来。

“我”母亲:慈祥、宽容、厚道地对待所有的人,对待这个世界;世界却没有给她以公平:变卖家产,告别家园,跟儿子去外地谋生,她的前面是一片未知。

水生:一个20年前的闰土,20年后,他或许不会再成为今天的闰土;对他来说,与生俱来的苦难其实也是一种改变命运的财富!

宏儿:从“飞”出家门的那一刻起,他就开始“飞”向这个既陌生又新鲜的世界了。

第三课时

四、深入探究

1、在课文中找出描写环境的语句,并说说它的作用。

12段:明朗、美丽、令人神往,2段:晦暗、萧条、令人悲凉。

2、作者离开故乡时有没有依依不舍的情感?他的心情是怎样的?

没有,人与人之间的隔膜使作者气闷,闰土的变化使作者悲哀。

3、为什么“我想到希望,忽然害怕起来”?

因为我的希望是让后辈过幸福的生活,但社会的黑暗,困难重重,害怕希望不能实现。

4、最后一个自然段,作者把“希望”比作“路”。这段话有什么含义?表达了作者怎样的思想感情?

这段话告诉我们,希望的有无,取决于实践,只有希望,不去努力奋斗,等于没有希望;希望虽然遥远,而且实现起来困难重重,但只要努力去奋斗,去实践,希望就能实现。表达了“我”对“新生活”一定会来临的坚定信心。

五、本文主题的探讨。

①茅盾先生认为这篇小说的主题是“悲哀那人与人之间的不了解,隔膜”。 ②反映辛亥革命前后农村破产,农民痛苦生活的现实,揭示产生这种现实的根源。 ③表达对改造旧社会创造新生活的愿望与信心。

六、作业:

课文中的水生和宏儿以后会有怎样的命运?发挥想象,为他们写一个故事。

10、孤独之旅

教学目标: 知识与技能:

1、把握小说主要情节,了解小说主要内容;

2、理解小说中细节描写和环境描写的作用;过程与方法: 运用自主、合作、探究的学习方式进行学习。 情感态度与价值观:

培养学生积极的人生态度,正确对待成长中的困难和挫折。 教学重难点:

理解小说中细节描写、环境描写的作用。 学情分析

学生在上一篇课文的学习中已初步了解了小说的三要素,并对人物形象有了较为具体的认识,对小说情节及环境有了一定的认识。可以说对小说的三要素,学生仅限于简单认识和理解。本节课将通过对课文的教学,让学生对小说的三要素有进一步的认识和把握,而又侧重于对环境描写的认识、理解和把握。 教学建议:

1.在自读中学会圈点勾画,快速捕捉主要信息; 2.在朗读中逐步进入情境,体会文中美好的情韵; 3.在研读中联系实际,体验成长的感受; 4.教学方法可以小组讨论、个人感悟为主。 教学课时:1课时 教学过程:

一、导入新课

请同学谈谈对“孤独”二字的理解,我们总会感到凄凉,酸苦,每天都是阳光灿烂的日子该多好。可是小小少年总要长高,烦恼和孤独总会尾随我们而来。可以说孤独随时间而来,孤独使生命更加灿烂更加丰富多彩。同学们,让我们今天一起来学习曹文轩的小说《孤独之旅》。

二、初读课文,整体感知课文内容

1.学生自读课文,朗读时注意在文中圈点勾画出主要信息。 2.检查预习字词情况。

注意下列字词的读音:嬉闹、掺杂、给予、撩逗、凹地、胆怯 积累下列词语:厚实、嬉闹、一落千丈、置之不理、歇斯底里

3.重点朗读以下段落,学生朗读时,注意体会人物心中的孤独感,读出语气,读出感情。

(1)从“小木船赶着鸭子„„杜雍和这才将船停下来”。 刚开始,杜小康想回家,父亲则不肯,怕自己也会像儿子一样突然对前方感到茫然和恐惧。 (2)从“这才是真正的芦荡„„并且迟迟不能人睡”。 到达芦苇荡后,父子感受不同,儿子“害怕”,父亲也有些“慌张”,却安慰自己的儿子。 (3)从“日子一天一天地过去了„„就不再忽然地恐慌起来”。 随着时间的流逝,父子俩感到孤独。

(4)。从“那天,是他们离家以来所遇到的最恶劣的天气„„也滴在跟在他们身后那群鸭的羽毛上„„”最恶劣的天气中,杜小康经受了考验 4.整体感知课文内容。

(1)要求学生用一句话概括文章的内容和主旨。 这篇文章讲述了一个什么故事,表达了什么的主题?学生只要说出自己的想法即可

(2)本文虽是长篇小说的节选,但也有完整的故事情节。要求学生阅读课文,理清小说的故事情节。

开端;写杜小康因家庭经济困难而不得不辍学跟着父亲去放鸭。 发展:写杜小康与父亲撑船赶鸭去芦苇荡的经过和感受。 高潮:写杜小康与父亲在芦苇荡中遇到暴风雨的情景。 结局:写鸭子长大了,杜小康也“长大”了。

三、精读课文,揣摩课文的语言

1、首先我们怎样理解课文标题的含义。(概括杜小康的孤独从哪些方面体现出来?)

(1)失去交流环境。对于一个离开学校和同学的少年来说,是无比痛苦,非常寂寞的,失去了同伴的帮助,没有同学间的交流,:等于失去了正常的生活。 (2)失去了人文环境。杜小康随父亲去放鸭,离开了村庄,离开了人群,父子缺少交流,“越来越单调,越来越干巴巴的了”。这种空虚、贫乏的生活,在精神上是一种折磨。

(3)恐惧自然环境。大自然的空阔与未可知,对一个幼小的心灵,具有强大的压迫力,由此带来的心理上的恐慌让他“迟迟不能人睡”。

(4)恐惧未来环境。在放鸭的路上,杜小康想的是:还要走多远?前方是什么样子?这不仅是对放鸭而言,更代表了他对前途的迷茫与无奈。

2、课文有关自然环境的描写很出色,请找出几处,并说说这样的环境描写对表现杜小康的成长起什么作用。(1)P70.写杜小康对远逝的家的眺望和鸭群扇形水流的描写,表现杜小康的幼稚、软弱和恋家。鸭群流水虽然美,但毕竟单调、永恒,并不好玩。这些更增添了对在家乡生活的留恋。

(2)P71.写黄昏时炊烟和河面的热气,是那么的单调、虚无,更增添了杜小康孤寂的感受。

(3)71有关鸭群的描写:鸭群向芦苇荡行进,偶尔“朝着这片天空叫上几声”,烘托出杜小康无比寂寞的心情;晚上鸭群“将主人的小船团团围住”,时常看看主人是否在船上才安心睡觉,鸭子的恐惧感衬托出杜小康的恐慌。 (4)P72。有关芦苇荡的描写:芦苇荡“如绿色的浪潮直涌到天边”,无边无际,给人以极大的心理压力,让杜小康“害怕”和“胆怯”。 当杜小康习惯了孤独的生活,“再面对浩浩荡荡的芦苇”时,就不再“恐慌”了,恐惧只是心理的折射。

(5)P75。有关风雨的描写:芦苇荡的暴风雨是极其可怕的,“雷声”“如万辆战车从天边滚动过来”,“暴风雨”“歇斯底里”“天昏地暗”“仿佛世纪已到了末日”。暴风雨冲垮了鸭栏,惊散了鸭群。杜小康表现了大人似的勇气与坚强,在暴风雨中搏斗,暴风雨给了他一个成长的机会和舞台。

3、文中随着故事情节的发展对杜小康成长过程作了细致的描写,杜小康的心理是如何变化的? 提示:

出发时:P69

茫然,恐惧 到达芦苇荡时:P72

害怕,胆怯 安顿之后: P73

感到孤独

时间久了:

P74

不再忽然地恐慌 那天雨后天晴:

P76

坚强。

4、如何理解杜小康这一人物形象呢?

杜小康从幼稚、软弱、恋家的孩子成长为一个心灵成熟的男子汉的过程。

四、研读课文,结合自己的体验谈感受。(各抒己见,畅谈自己的感受)

1.困难和挫折,往往是人成长的催化剂,结合自己的实际,谈谈学习本文的收获。

2.你有过孤独的时候吗?请讲述你孤独的情景和心理感受。 3.请你就杜小康父亲的教子方式,谈谈你的理解和看法。

五、课外作业

学习本文后,让学生明白,人物和环境总是互相依存的。离开了环境,人物便没有活动的天地,人物性格的发展便失去了依据。请结合自己的成长历程,写一篇文章,注意要有一定的环境描写。

11、我的叔叔于勒

一、教学目标 知识目标:

①了解作家。②掌握常用字词。③分析人物形象及其作用。④理解小说主题。 能力目标:

①提高学生分析人物性格的能力。

②把握小说曲折的情节,培养学生的思维力、想像力和口头表达能力。 德育目标:

树立正确的金钱观。

二、教学重难点

重点:理解作者从多方面刻画人物形象,以展现人物内心世界的方法;运用对比手法刻画人物性格。

难点:悬念的铺排,构思的巧妙。

三、教学过程 1.导入新课

在世界文坛上,有三在短篇小说之王,他们是法国的莫泊桑、俄国的契诃夫、美国的欧?亨利。今天我们就来学习莫泊桑的名作《我的叔叔于勒》。

2、听课文诵读录音,思考下列几个问题: (1)于勒是怎样一个人?

(2)菲利普夫妇、一家为什么盼望于勒回来?

(3)小说的结局是菲利普一家人躲开了日夜盼望的于勒,这是什么原因? (4)这篇小说家提示了怎样的主题思想?

3、学生讨论,在全班交流明确: (1)穷??富??穷

(2)据说做生意赚了钱,成为全家的“希望”、“福音”。 (3)直接原因:是菲利普只认识钱和于勒又沦落为穷人;

根本原因:是那个金钱至上的社会制度。

(4)小说写了一个普通小市民家庭的日常生活,作者运用了对比的手法,充分描述了菲利普夫妇对待亲兄弟于勒的前后截然不同的态度,画出一幅资本主义社会里贫穷则兄不认弟的触目惊心的惨象,揭示了资本主义社会人与人之间的关系是“纯粹的金钱关系”的主题思想。

4、精读船上巧遇的片断,了解小说在刻画人物时运用的主要方法。

主要通过:表情、动作、言语的描写来揭示人物的内心世界,刻画人物性格。

四、小结。

这篇小说运用对比手法成功刻画菲利普夫妇只认钱不认人、爱面子的形象。提示了资本主义人与人之间的金钱关系。

五、作业。

1、找出小说中的环境描写,分析它们的不同点,说说它们各有什么作用?

2、课后练习四。附板书:

第一部分:1-4

故事的开端,盼望

于勒。 第二部分:5-19 故事的发展,夸赞

于勒。 第三部分:20-47 故事的高潮,巧遇

于勒。 第四部分:48-49 故事的结局,躲避

于勒。

12 心

一、教学目标

知识与技能:

⑴简要复述小说内容,培养学生的概括能力;

⑵分析和评价小说人物形象,理解“心声”的含义。

过程与方法:

在老师的引导下,学生通过课前预习课文,整体感知:课上速读课文,掌握故事情节;课中精读

文,理解人物形象的刻画;独立思考,讨论交流,理解文章主题和细腻的心理描写;联系自己的生活经历,

明白小说所反映的问题。

情感态度和价值观:

⑴、展示学生个性,引导学生从文中摄取有益的东西,提高学生的人文素养。

⑵、让学生从京京的身上体悟亲情,懂得在今后的生活中应怎样去珍惜亲情、友情。

【教学重点】:

本文重点是弄清“心声”含义,在熟识小说情节的基础上,分析小说人物形象进行归纳。

【教学难点】:

小说在塑造人物时主要运用心理描写,这是本课的难点;初次接触小说,侧重引导他们体会心理 描写的特点和作用一)( (

二、

课时安排:二课时

三、

教学过程 第一课时

一、导入。听取学生在学习和生活中的感觉。

二、正音听读 捻 niǎn

噗pū

蜷 quán

簪zān 琅琅láng

发窘jiǒng 抽噎yē

恍惚 huǎnghū

抽泣qì棱角分明léng xīsū

2、听读课文,要求同学听准读音,整理故事情节。

三、整体感知课文。

学生朗读课文,思考下列问题。

1、京京为什么渴望朗读课文?又是在什么情况下实现了自己的愿望的?他能不能读好课文呢?

当李京京看到讲义上印的《凡卡》时“他喜欢极了”,因为凡卡使他想起乡下跟爷爷在一起的幸福生活,想起了和妮儿一块儿摘桑果、吃桑果的趣事。课后听了赵小桢“软绵绵、奶声奶气”地练习,更增强了他公开课上事先布置好的赵小桢不敢举手,别人更不敢举手,只有李京京举起了手。“程老师有点慌乱了”,在实在没有人举手的情况下,处于无奈,程老师只好叫李京京读。凡卡的遭遇引起了有着相似经历和同样不幸命运的李京京深深的共鸣。他读得声情并茂,“教室里一片寂静”,大家都被感动了。

2、嗓子沙哑的李京京为什么想读而且一有机会就能读得很感人,而声音又脆又甜的赵

小桢事先答应了读,却最终没有勇气在公开课上举手,担心自己读不好。 李京京和赵小桢的区别在于,朗读过程中是否有自己的情感参与。(当然情感参与的前提并不限于相似的经历,而应当是朗读者在理解的基础上进行的体验性阅读。凡卡是个穷孩子,朗读时要准确地把握住他的情感,而不能读成爱撒娇的小姑娘的情感。)

3、李京京在实现愿望的过程中,吐露了怎样的“心声”?

心声:凡卡的故事唤起了他深埋在心底的亲情和友情,他渴望将自己内心的情感表达出来。

4、李京京为什么关注赵小桢等人练习? 明确:“他喜欢极了。那个穷苦的、可怜的小人儿凡卡,不知怎么,总是缠在他心上,弄得他一整天都有点儿神情恍惚”。这样的心绪,为渴望表达又被粗鲁地阻止李京京关注赵小桢等人练习提供了合理的心理基础。同时作者再次突出表现了他读书愿望的迫切以及对课文情感的准确把握,为下文李京京读书的成功埋下伏笔,也为戏剧性高潮的到来提供了合理性。这一笔不仅丰满了人物形象,还显出情节安排上不枝不蔓的特点。

5、怎样理解“他真想念。不是要出风头,而是心里有种憋了很久的感情,想痛痛快快地念出来,吐出来”? 明确:“憋了很久的”,有渴望表达深埋与心底的亲情、友情的心声,也有压抑已久渴望张扬的个性。

四、小结:

这些文字之所以感人得益与小说中细腻的心理描写。小说刻画人物有多种方法。本文主要是描写李京京的心理活动。课文中有直接的心理描写,如课前非常想读课文的愿望,树林里对爷爷、对妮儿的思念,课上想读又不敢举手的矛盾,真情投人而读过了头的懊悔等等;有间接描写,如课前渴望老师让他读书的眼巴巴地望着老师的神态,结结巴巴的话语,课上“他咬紧了嘴唇,郑重地举起右手”,希望得到朗读机会的动作等。这些描写,细腻而真切地写出了李京京的心路历程,也突出了小说的主题。

第二课时

一、李京京是一个怎样的人物形象?请找出描写李京京行为和心理的句子。形象:朴实真诚、善良而宽容、自信而勇敢,对生活有着美好的愿望的少年。 (1)公开课前,程老师布置朗读任务时时:

“不安地扭动身子”

“眼巴巴地望着老师” 结结巴巴地说:“老师,我能„„念一段吗?”

(2)回家路上,偷偷地在树林里读课文,并深情回忆:

忍不住„„他选了一段,轻轻地念出声来

他咽了一口唾沫,把讲义举在面前,终于大声地从头念起来

公开课难道是上台表演吗?嗓子不好的人,就只能躲在树林子里读他喜欢的课文吗?京京心里难受极了。 (3)第二天放学后,听赵小桢练习朗读时:

①当赵小桢将穷孩子凡卡读成娇滴滴的小姑娘时:

不,不是这样的。京京听着,在心里说,不是这样的。 “不是这样的!”京京终于叫出来。

②当赵小桢惊讶地质问并和其他同学一起奚落他时: 京京有点儿发窘。„„他嘟嘟囔囔地说„„

京京红着脸,固执地嘟囔着„„

他委委屈屈地离开教室„„ (4)公开课上,被指定的同学不举手时:

京京在坐位上不安地扭动着身子。他真想站起来。可是如果举了手,程老师会喊他吗?课后赵小桢会不会嘲笑他?他真想念。不是要出风头,而是心里有种憋了很久的感情,想痛痛快快地念出来,吐出来。

他咬紧了嘴唇,郑重地举起右手,眼睛一眨不眨地望着程老师。

二、请一位同学有感情地朗读描写李京京读书情景的文字。

三、朗读第14小节,想象这段景物描写的画面,体悟历经磨难而今“柳暗花明”的胜利者的心情。

四、

怎样分析程老师的人物形象?

明确:程老师“认真,好强,又有点自信”,区教育局公开课交给她完成,也说明她水平不低,特别是为上好这节课,她绞尽了脑汁。她不让李京京朗读课文,是因为李京京嗓子沙哑得厉害,怕他念不好,影响了学校荣誉。为什么这样说呢?小说结尾写得很清楚:当程老师被李京京出乎意外的精彩朗读感动时,竟“从讲台上走下来,走到他面前,声音发颤地说:

‘李京京,请你„„把课文全部读完吧。”’这种对工作负责的精神是应当也必须肯定的。程老师是现实中的人,为求得领导好评,采取急功近利的办法,把公开课上成表演课,这一点也是可以理解的。但是小说客观上反映出的公开课只追求形式,甚至弄虚作假的风气是应当摒弃的。

五、深思探究

课文中的“心声”除了李京京渴望表达深埋在心底的亲情和友情外,还含有何种深意?

明确:作者想借主人公的心声,呼吁我们的教育工作者要尊重、关爱每一个学生的愿

望和感情的心声。

六、布置作业 :完成课后练习题

(三),

13、事物的正确答案不止一个

教学目标:

1.知识目标:理解观点和支撑观点的材料之间、中心论点和分论点之间的关系;

2.能力目标:培养独立阅读、认真探究、积极讨论的能力; 3.情感态度价值观:激活思维,激发想像力,崇尚科学。1. 重点难点

1.重点:理解本文的中心论点和分论点。

2、难点:理解并运用事实论据。教学时间:一课时 教学过程

一、导入

前两年高考作文题,就是以本文的开头的图形为题,它告诉我们什么道理呢?

二、阅读课文思考问题:

1.事物的正确答案为什么不止一个?

因为事物是丰富复杂的,生活中解决问题的方法并非只有一个,而是多种多样。

2、为什么要确立“事物的正确答案不止一个”的思维方式?是用了怎样的论证方法阐明这一事理的?运用这一论证方法的好处是什么?

作者运用了正反对比的论证方法阐述这一道理。先从反面说,“如果你认为正确答案只有一个的话,当你找到某个答案以后,就会止步不前。”;再从正面说。“不满足于一个答案,不放弃探求这一点非常重要。”正反对比,使说理全面,而双透彻,增强了说服力。

3、产生创造性思维必须具备哪些条件?又用了怎样的论证方法论证的? (1)富有创造性的人总是孜孜不倦地汲取知识,使自己学识渊博。 (2)必须有探求新事物,并为此而活用知识的态度和意识。 (3)持之以恒地进行各种尝试。

作者采用了举例论证的方法,具体而又确凿地阐明了事理。

4、区分一个人是否拥有创造力,关键看什么?

拥有创造力的人留意自己细小的想法。即使他们不知道将来会产生怎样的结果,但他们很清楚,小的创意会打开大的突破口,并坚信自己一定能使之变为现实。

5、如何才能成为一个富有创造性的人?

1关键是要经常保持好奇心,不断积累知识;2不满足于一个答案,而去探求新思路,去运用所得的知识;3一旦产生小的灵感,相信它的价值,并楔而不舍地把它发展下去。

三、阅读3-6段,回答问题。

1、第五段中的“这种情况”、“当事人”“它”分别指什么?

这种情况:知识随时都可能进行组合,形成新的创意。当事人:富有创造性的人。它:新的创意。

2、认为正确答案只有一个有什么危害性? 当你找到某个答案以后,就会止步不前。

3、选文中,有一个非常重要的过渡段,请指出是哪一段?并说说其作用? 第四段,承上启下,使文章层次分明,又结构严谨。

4、根据选文内容,概括“创造性的思维”所必需的“要素”。

(1)富有创造性的人总是孜孜不倦地汲取知识,使自己学识渊博。 (2)必须有探求新事物,并为此而活用知识的态度和意识。 (3)持之以恒地进行各种尝试。

5、为什么说“不满足于一个答案,不放弃探求这一点非常重要”?

因为生活中解决问题的方法并非只有一个,而是多种多样。如果你认为正确答案只有一个的话,当你找到某个答案以后,就会止步不前。

四、小结

本文按照逐层递进的逻辑顺序论证了怎样做一个富有创造性的人这个中心论点。

五、作业:完成课后练习

14 应有格物致知精神

教学目标: 知识目标:

1、积累词语:“格物致知、彷徨、袖手旁观、修身、齐家、诚意、正心”等词语。

2、了解丁肇中,理解“格物致知”的真正含义。能力目标:

1、自读课文,抓住过渡段、关键句、关键词等,理解作者的观点,理清文章的论证思路。

2、学习“摆事实、讲道理”的论证方法。情感态度与价值观:

引导学生感受科学实验精神的重要性,树立格物致知精神及乐于动手实验的科学品质。

重点难点:

(l)理解真正的格物致知的精神。 (2)学习编写阅读提纲。

(3)理解文章的论证方法。 教学时间:一课时 教学过程

一、导入

1.作者简介。 2导语。

有人说中国的孩子“高分低能”“会读书不会研究”缺乏创新精神和实践动手能力。问题出在哪里呢?丁肇中教授的文章会给我们极大的启发。

三、整体感知

1.默读课文,讨论编写阅读提纲。

讨论并归纳:可从提出问题、分析问题、解决问题这三个部分来列提纲。 第一部分(l~2段):提出问题。中国学生应该怎样了解自然科学?要格物致知。 第二部分(3~12段):分析问题。 第一层(3~5段):分析中国教育不重视格物致知的社会根源。举例王阳明的格物是格已,这种观点不能适用于现在的世界。 第二层(6~10段):分析实验精神在科学上的重要性。 第三层(11~12段):分析在这种文化背景下中国学生的现状c以“我”举例。 第三部分:解决问题。强调我们需要培养实验的精神的意义并对我们这一代提出希望。

2问:丁肇中教授在文章中指出中国学生应该怎样学习自然科学?

要有格物致知精神。 3.问:“格物致知”出之哪里,它的意思是什么?

“格物致知”出至《四书?大学》丁教授引用“格物致知”的意思是从探察物体而得到知识。也就是通过实验得到知识。

4.问:传统的中国教育并不重视真正的格物和致知的原因是什么?

可能是因为传统教育的目的并不是寻求新知识,而是适应一个固定的社会制度。 5.问:《大学》里讲的格物致知的目的是什么?

目的是诚意、正心、修身、齐家、治国、平天下。 6问:这个目的与丁教授讲的格物致知的目的有没有一样?

不一样。丁教授讲的目的是通过实验得到新知识。这才是格物致知的真正意义。

7.问:第4段举王阳明“格物”失败的例子证明了什么?

王阳明的“格物”实是“格已”,而不是研究万物的道理,这是无用的,不能适用于现

在的世界的。证明传统的中国教育的目的是不正确。 8.问:实验的过程应该是怎样的?

实验的过程应该是积极的、有计划的探测。 9.问:第8段举研究竹子的例子证明了什么?

举研究竹子证明实验的过程不是消极的观察而是积极的动手栽种、研究生长过程、科学的观察。

10.问:第9段讲的是什么道理?

实验的过程不是毫无选择的测量,而是需要小心具体的计划,要有一个适当的目标。

11.问:在这样的文化背景下,中国学生的现状是什么样的?

中国学生大都偏向于理论而轻视实验,偏向于抽象的思维而不愿动手,功课成绩很好不会做研究。

12.问:第12段举我个人的经验证明了什么?这是什么论证?

举我个人的经验证明光“用功”埋头读书,对于实际的需要毫无帮助。属于举例论证。

13.问:为什么说当今社会需要真正的格物致知精神?

真正的格物致知精神不但在研究学术中不可缺少,而且在应付今天的世界环境中也是不可少的。 14.问:丁教授认为《四书》中格物致知真正的意义是什么?

第一,寻求真理的堆一途径是对事物客观的探索;第 二,探索的过程不是消极的袖手旁观,而是有想像力的有计划的探索。

四、小结:

本文就中国学生的现状,指出格物致知的重要性和真正意义,希望我们这一代要培养实验的精神。文章用举事例讲道理论证了中心论点。

五、作业:完成课后作业。

15、短文两篇

教学目的:

1、知识与能力:学会阅读简单的议论文,区分观点与材料(道理、事实、数据、图表等),正确把握观点与材料之间的关系,掌握论述语言的特点。

2、过程与方法:激发学生阅读议论文的兴趣,掌握阅读议论文的方法,养成阅读议论文的习惯。

3、情感态度价值观:让学生明白,只有具创新意识,创新精神,才能成为有用人才,才能为祖国贡献力量。教学重、难点: 重点:

(1)课文中出现的名言警句的体会学习。

(2)举例论证、道理论证、比喻论证等方法的运用。 难点:本文论述的语言十分简练,在学习时体会其作用。 课时安排 2 教学过程:

第一课时

一、检查预习情况。

分小组比赛,检查积累材料的情况。要求每小组派代表把整理好的关于读书方面的名言警句、名人读书的趣事讲出,以多者为胜方。

二、简介作者导入新课

培根(1561?1626)英国哲学家、作家,出生贵族家庭。剑桥大学毕业,后又学习法律,1618年任大理院院长,封为勋爵。1621年因受贿为国会弹劾去职,嗣后居家著述。1626年冬由于在野外试验雪的防腐作用而受寒致死。《随笔》是其文学方面主要著作,开创英国随笔这特有体裁。

三、指导学生通读全文,把握内容:

1、指导学生朗读课文。

明确:这篇随笔写法不同一般,它论述的范围相当广泛,语言十分简练,几乎一句话就是一个观点。

2、理解文章内容:作者从哪几个方面谈读书?

提示:要理清作者的思路,运用学过的知识归纳、总结。 明确:文章可以分为三个层次去理解: 第一层:(从开头到“全凭观察得过且过知”)阐述读书的正确目的。 (1)先从正面说,读书有三种不同目的:怡情、傅彩和长才。重点阐述了读书的好处。

(2)后从反面指出读书的三种偏向: 并论述读书和经验的关系。

(3)最后指出,只有明察事理的人才能够读书、用书,而用书的智慧是在观察生活中得来的。

(4)提问体会句子含义及作用。 文章开头的第一句话,运用的是排比说理。 (5)提问体会论证方法: 对比论证、比喻论证。

“盖天生才干犹如自然花草,读书然后知如何修剪移接。” 体会:

培根在其作品中往往采用这种手法,显出其文笔老练,说理透彻。又如:“德行犹如宝石,朴素最美。”“声名犹如大河,空虚无物者浮,实学有才者沉。”„„ 这种比喻的运用,使其文章更显生动形象,通俗易懂。 第二层:(读书时不可存心诘难作者?始能无知而显其有知)阐述读书的方法。 (1)本层第一句,作者这样写的用意是什么?

明确:这一句指出读书要仔细推敲细思,反对故意挑刺,迷信书本和公限于文字推求。

(2)对不同的书,作者又主张怎样?

明确:学生找出语句,明确作者主张采用的不同的读书方法。

(3)找出文中作者提倡把读书和讨论、作文、做笔记结合起来的那些句子。 第三层:(读史使人明智?结尾)

阐述读书能塑造人的性格和弥补精神上各种缺陷。 分两个小层理解:

(1)作者在这一层举了一系列的例子,目的是论述读书能塑造人的性格,体会其说理方法。

明确:排比说理、归纳说理。(正面指出)

(2)作者进一步指出,人的精神上的缺陷,可以用读书来弥补,就像身体百病,可以用运动驱除一样。 (反面指出) 论证方法:对比论证、举例论证。

四、讨论与交流:

(1)学习本课后,对议论文的写作的认识体会。

(2)本课中出现的名言警句很多,以小组为讨论单位,谈谈对这些内容的看法。

五、总结、归纳:

六、布置练习:摘抄有关“读书”方面的名句扩大写作素材库,积累写作经验。

(二)不求甚解

第二课时

教学目标:

1.以自主探究的学习方法为主,充分激 发学生的主动意识和探究精神; 2.认识读书的益处;养成良好的读书习惯; 3.培养学生搜集资料、处理信息的能力。 教学过程: 1.导人新课

2.让一个学生朗读全文,其余的同学一边听一边思考: ①本文的主要内容是什么?

②这位同学读得好在哪里?有哪些需要改进的地方? 3.快速阅读《不求甚解》,边读边思考: 找出“不求甚解”在本文中的含义 。

4.学生自主提出有关的问题,分小组研讨。 ①“好读书” 和“不求甚解” 有什么关系?

②肯定“不求甚解” 的方法为什么要反对马马虎虎的态度? ③本文主要讨论了什么问题,用了哪些论证方法? 5.拓展训练

学习《不求甚解》的写法,对下列成语进行辩证分析,运用求异思维,口述新的立意。

①“见风使舵”新解 ②“班门弄斧”辩 ③“滥竿充数”辩

(明确:成语新解,重在从不同的角度去诠释,划清必要的界限,从正名人手,这往往是行文的前提,另外要紧密联系实际,借题发挥,做到有感而发,有的放矢。“见风使舵”一词,多用于那些不讲原则的世故之徒,但如果我们赋予“风” 以新的含义,喻指实际情况的变化或者规律,我们可以翻出新意;根据实际情况的变化,掌握方向,采取相应的措施,推动社会主义的航船向前发展。这样联系现实不是大有文章可做吗?“班门弄斧”这个成语,可以从故事人手,翻出“不迷信权威,敢于向权威挑战” 的新意。“滥竿充数” 这个成语,也可以从故事人手,翻出“滥竿之所以能充数,在于欣赏者的水平低下” 的新意。) 6.作业 : 背诵课文

16、中国人失掉自信力了吗

【教学目的】

知识与能力目标,

1、积累词语,掌握文中的一些词的读音,词义,并学会运用。

2、理解重要语句的深层含义,体会鲁迅的忧患意识和爱爱精神。

3、掌握反驳论证的方法。过程和方法目标 通过质疑、讨论等方法学习本文结构严谨、波澜起伏的艺术特色,学习反驳论证。 情感态度和价值观目标

弘扬爱国主义精神,增强民族民主意识,增强民族自信心、自豪感,为祖国美好的明天而奋斗。 【教学重点难点】

1.体会鲁迅的忧患意识和爱国精神。2.理解重要语句的深层含义。 3.学习文章的写作技巧。 【教学时数】:1课时 【教学步骤】

一、解题导入新课

本文是一篇驳论文。这种文体一般是先指出对方错误的实质,或直接批驳(驳论点),或间接批驳(驳论据、驳论证);继而,针锋相对地提出自己的观点并加以论证。从广义上来看,这又是一篇杂文,杂文短小精悍、写法灵活。鲁迅的杂文有“投枪”“匕首”之喻,由此也可窥见鲁迅杂文风格之一斑。 本文收入《且介亭杂文》,属鲁迅后期杂文。

二、研习课文

(一)整体把握,理清思路 1.速读课文,扫清字词障碍。

2.对方的错误观点是什么?作者为什么认为它是错误的?作者正面提出的观点是什么?作

者提出观点的依据是什么? 明确:对方的错误观点是“中国人失掉自信力了”,因为信“地”信“物”信“国联”,从来就没有相信过自己;而现在的求神拜佛,则是在自欺了。作者正面提出的观点是“中国有并不失掉自信力的中国人在”,因为有过去和现在的事实为证。

(二)具体研习,突出重点、难点 1.诵读第1~5段,讨论探究: (1)作者既然认为对方的观点是错误的,为什么还要一再承认对方说的都是“事实”?

明确:首先承认对方说的是事实,但通过分析事实后,发现对方要说的其实不是自信力的问题,这样,对方的观点就不存在了。

这是驳论的一种方式,由真实存在的依据推导出错误的结论,从而证明对方的观点是错误的。指出对方论据证明论点的过程不成立,这是驳论证。它欲擒故纵,先“抬”后“贬”,好像抖包袱,吊起读者的胃口,达到讽刺的效果。

要驳论证,关键要能透过现象看本质,因为对方或因为己方利益或因为认识上的局限,往往用事实掩盖了真相,这就需要作者能拨乱反正。因此驳论证比驳论点、驳论据更能体现智慧的力量

(2)第5段有一段加点的文字,国民党检察官曾删掉这段文字,这说明什么? 明确:“求神拜佛”恐怕是国民党在山穷水尽时自欺和欺人的最后一招。鲁迅是个现实感很强的人,他一针见血地指出“求神拜佛”的危害和严重后果,这自然触到了国民党的痛处,并且,让他们陷入捉襟见肘的境地。所以,检察官惟恐去之而不及。

从加点文字也可以看到鲁迅的境况,但鲁迅却不惮于在枪林弹雨中为民众奔走呼号,其无私无畏的精神令人敬佩。 2.品读第6~9段,讨论探究: (1)“中国的脊梁”指什么人?为什么他们的牺牲不能为“大家”所知道? 明确:“中国的脊梁”是指脚踏实地地为民族的进步而奋斗的人们,他们是使中国挺立起来的“脊梁”,他们往往来自下层或代表着广大民众的利益。但由于中国长期的专制奴役统治,“中国人向来就没有争到过‘人’的价值,至多不过是奴隶”(《灯下漫笔》),许多事情自然无权知道真相。另外,近代以来的许多社会变革多缺乏对广大民众的宣传发动,这样一来,对于为他们的自由尊严而牺牲的先驱者,他们多半表现出“看客”的心态,不能知道在某种程度上演化成不愿知道了。对于这一点,文中虽然没有提到,但结合当时的社会现状,不能说没有这个因素。 (2)“状元宰相”、“地底下”的含义分别是什么? 明确:“状元宰相”是指统治阶级的御用文人,鲁迅曾评述御用文人“他的帮法,是在血案中而没有血迹,也没有血腥气”,他们的功能就是为世道搽上“自欺欺人的脂粉”。“地底下”指变革社会的积极力量,因为它来自民众,故称“地底下”,在当时,这股力量指中国共产党。他说:“那切切实实,足踏在地上,为这现在中国人的生存而流血奋斗者,我得引以为同志,是自引以为荣的。”(《答托洛斯基派的信》)把“状元宰相”、“地底下”相对比,可看出鲁迅的“立人”精神和民本思想。

(三)欣赏品味

1.揣摩第1段话,三个副词“总”、“只”、“一味”能否互换位置?为什么? 明确:不能。“总”让人看到国民党政府夸耀“地大物博”时的洋洋自得,同时“总”又写出夸耀者的底气不足,因为,夸来夸去只能夸这一样,且惟恐别人不知,这也暗接“只求国联”的必然性:“只”是抓救命稻草时的“执著”,仰人鼻息之状跃然纸上;“一味”则是深陷而不能自拔,沉迷于其中的滋味因而也不愿自拔。这三个副词准确地写出了国民党政府在自欺的道路上越走越远的“事实”,极富讽刺意味。

2.揣摩下列加点字词的表达效果:

„„不过一面总在被摧残,被抹杀,„„那简直就是诬蔑。 明确:“总”写出当时社会的黑暗程度,并间接讴歌“中国的脊梁”屡挫屡战的精神。“简直”表达了作者的激愤之情。

(四)课堂训练

1.作者用什么方法反驳对方的错误论调?

明确:用了驳论证的方法,即指出对方的论据不能证明观点,然后正面立论:“中国有并不失掉自信力的中国人在。” 2.“中国的脊梁”在当时有什么意义?

明确:当时国民党到处散布悲观论调,广大民众也因“中国的脊梁”的总被摧残、被抹杀而看不到光明。鲁迅反复赞扬“中国的脊梁”,可以鼓舞斗志,增强中国人的自信力。

(五)课堂小结

通过这节课的学习,我们学习了驳论文的一种写作方法,理解了鲁迅关于“中国人自信力”的阐述,“中国的脊梁”在当时的社会曾大大鼓舞国人的士气,在今天仍有很强的现实意义。同时,鲁迅杂文的语言技巧和讽刺手法的运用也极大地增强了文章的表达效果。

四、布置作业

17、《智取生辰纲》教案

教学目的: 知识与技能:

1.借助课文注解或工具书理解文中较生僻词语的意思,如:纲、权且、趱行、端的、舒头探脑、尴尬、去处、兀自、口浅、却才、计较等。2.学习明暗两条线索叙述故事的方法。

3.学习环境烘托表现人物,设置矛盾冲突刻画人物的方法。 过程与方法:

1.教学思路和教学环节安排:情境导入(介绍背景)——初读课文(把握情节)——再读课文(“智”在何处)——研读课文(概括形象)——拓展延伸(布置作业)

2.了解分析作品的叙事线索,理解本文精巧的构思。

3.训练学生的阅读和写作能力,提高学生欣赏优秀文学作品的能力。情感态度价值观:

1.鉴赏本文情节跌宕起伏、人物刻画栩栩如生、摇曳多姿的艺术特色,感受悠久丰富的中国传统文化的魅力,培养学生健康的审美情趣。 2.引导学生走进古典文学天地,主动阅读古代优秀作品,在历史文化土壤中吸取营养,提高文学修养。。 教学重难点:

一、了解认识杨志的思想性格。

二、学习作品明暗两条线索的特殊叙事方法。课时安排

第一课时

教学要点 :

了解〈水浒传〉的主要内容及节选课文中的故事情节。环境描写对情节的作用。

教学过程

一、导入新课

〈水浒传〉是一部歌颂农民起义的作品。主要思想倾向是官逼民反。历代正统文人都把农民起义看作是“犯上作乱”,而 罗贯中却深刻揭露出农民起义是由于官府的沉重压迫。小说的局限在于只反贪官,不反皇帝。小说歌颂了一系列英雄。本文节选的,〈智取生辰纲〉描述的主要人物杨志也是“一百单八将 ”之一,节选课文中他还没有“造反”。

〈水浒传〉作者施耐庵的生平不详,以下仅供参考:苏州人,声于元末,名初为避祸迁于江苏兴化县。

二、学生读书:要求能复述基本故事情节。思考问题:吴用等人是如何智取的?

三、学生读完书,复述故事情节。之后,思考、讨论、回答。教师点拨

智取生辰纲是吴用的计策。他就像玩魔术一样,眼见得一桶酒,人家吃没事,杨志一行人吃,却倒下了。作者把谜底放在最后揭晓,最后才说出人名。说明计策,上文则隐去人名,就显得扑朔迷离,神奇莫测。晁盖等七人主动亮出客商的伪装,解除了杨志的戒备心理。在歌声中,白日鼠挑酒上场,引众军汉买酒。其实两桶都是好酒,料杨志必然阻止。晁盖一伙过来先喝一桶。借舀酒做手脚,前一瓢掩饰,后一瓢下药,白日鼠追这个,夺那个,写不露丝毫破绽。这一伙痛饮解渴,安然无恙,引得众军汉心痒难耐。晁盖一伙的做法,完全麻痹了杨志一伙的警惕心理,终于诱其入了圈套。有趣的是白日鼠故意来个正题反作,一个劲不卖,说:“这酒里有蒙汗药在里头了。”论事实,这倒是真话,真可谓“假作真时真亦假”。

四、小说中环境描写是怎样的,有什么特点?

从杨志一伙上路起,到杨志自己也喝酒解渴止,反复写气温、写太阳、写树林;直接写“热”的近二十处之多,以“凉”衬“热”也不下十余处。写法也多变:有通过作者的介绍来写的,有通过八条好汉之口来写的,有通过杨志或虞侯、老都管之口来写的,更多的是通过军汉的语言和行动来写的。

描写特点:①渲染气氛,增强实感。②文不在多,妙在传神③烘托心理,形神毕现。④转换场景,发展情节。

五、布置作业:

划出小说中能够表现杨志思想性格的语句。归纳其性格特点。 板书设计:

内容:农民起义 《水浒传》

局限:只反贪官,不反皇帝 自然环境:天气炎热

作用:铺垫和推动情节发展

第二课时

教学要点

分析杨志的性格特点及课文的两条线索安排之巧妙。 教学过程

一、导入新课

一个不体察下情的领导,是不受欢迎的领导,杨志正是这样一个人,同学们,根据自己所划的语句,讨论、归纳杨志的思想性格。

二、学生讨论、交流杨志的思想性格特点

杨志相当精明,相当警惕。杨志致命的弱点,则是不善带兵,不善处理关系。

三、本文有两条线索,一是押送生辰纲,一是智取生辰纲,这样写有怎样的表达效果

故事情节更加曲折,引人入胜,增强艺术效果。先写杨志押送生辰纲,归结到一点,是要写明白杨志非常小心谨慎为什么最后准许众军汉喝了那桶酒,也就是说杨志怎么会掉进吴用设置的圈套。作者一路写来,一面写杨志的精明,一面写他的蛮横,终于左右不了局面,最终进入了“智多星”吴用设计的圈套。

四、小结:

《智取生辰纲》故事情节波澜迭起扣人心弦,人物刻画细腻入微形象生动,环境描写逼真简练恰到好处。它是以晁盖、吴用为核心的好汉群体第一次向贪官“出手”,与“官家”冲突,是大规模农民起义的前奏。

五、布置作业

18、杨修之死

教学目标:

1、知识与能力目标:

﹝1﹞理清文章脉络,简要概括事件。

﹝2﹞探究杨修死因

﹝3﹞多角度分析曹操、杨修的性格

2、过程与方法目标:

培养学生收集、归纳、整理信息的能力,训练学生拓展创新思维能力和口头表达自己见解的能力。

3、情感、态度与价值观:

﹝1﹞充分感受中国传统文化的丰厚底蕴,激发学生阅读古典小说的浓厚兴趣。

﹝2﹞培养学生正确的人生观、价值观。

4、重点、难点的确立

《语文新课标》明确指出:“阅读是学生的个性化行为,要珍视学生独特的体验。因此,我把如何发散学生的思维,培养学生的创新意识作为本节课的重点即探究杨修的死因。杨修之死的原因究竟是什么?争论由来已久,学生往往会有一些富有个性化的独特见解。面对这一系列的问题将怎样攻破呢?只有把原因分析透彻,杨修和曹操的性格才能跃然纸上。因此探究杨修的死因又成为本文的难点,

教学课时:2课时。 教学过程:

第二课时

一、导入新课:

(首先放《三国演义》的主题曲)师由音乐导入:一曲《滚滚长江东逝水》以其特有的雄浑和古朴为我们翻开了三国的历史画卷,作为欣赏者的我们,仿佛又被带到了金戈铁马的古战场。经受了一番刀光剑影的洗礼。是呀,在三国的历史舞台上,曾经有过许多鲜活的面容,涌现出许多的人物。我们曾经接触过其中的两位,一位是足智多谋的?(生:诸葛亮)一位是知人善任的?(生:刘备),今天,让我们共同学习罗贯中的《杨修之死》,看看作者笔下另两位人物是什么形象的。(师板书课题)

师:首先了解故事发生的背景,《杨修之死》选自《三国演义》第72回,故事发生在曹操、刘备两个军事集团争夺汉中之地最为激烈的时候。当时诸葛亮已智取汉中,曹操屡屡受挫,又不甘心失败,只好退兵斜谷口,以待时机。但终因蜀兵积极防守,难以进取。“杨修之死“的故事就是在这样的背景下发生的。

二、初读课文,整体感知,疏理小说情节。

1.检查字、词(给加点的字注音,用“恃才放旷”造句)

庖官

主簿

绰刀

恃才放旷

2.朗读要求:语速要适中,句子停顿要准确,尤其注意语言要符合人物身份、场合, 要读出感情来,完成下面的问题: (1)概括杨修性格特点的词语是什么? 答案:恃才放旷

(2)曹操为什么要杀杨修?

明确答案:因为杨修为人“恃才放旷,数犯曹操之忌”

(3)阅读前3自然段“适庖官进鸡汤。操见碗中有鸡肋,因而有感于怀”。联系上下文,你认为他“有感于怀”的“感”是什么?

明确:屯兵日久,欲要进兵,又被马超拒守;欲收兵回,又恐被蜀兵耻笑,心中犹豫不决。

(4)曹操看到碗中鸡肋,便把它当作口令,是某种情绪或心境的流露。请说一说是什么情绪或心境?

抢答后明确:犹豫不决、烦躁不安。

(5)杨修根据什么得出曹操“必班师”的结论?

明确:根据鸡肋“食之无肉,弃之有味”得出

(6)“行军主簿杨修,见传“鸡肋”二字,便教随行军士,各收拾行装,准备归程。有人报知夏侯惇。惇大惊”。夏侯惇为什么会“大惊”?

明确:因为擅自行动,客观上确实起了“惑乱军心”的作用,是违反军纪的。(7)操大怒曰:“汝怎敢造言乱我军心!”喝刀斧手推出斩之„„“造言”是什么意思?这个词用得准确吗?

明确:编造的话;准确,因为曹操毕竟没有说过退兵的话,只是杨修的分析猜测。

(8)对曹操杀杨修,你怎样评价?

明确:以“惑乱军心”处死杨修,似乎杀得有根有据,合情合理。这正是曹操为人奸诈险恶之处。

(9)读中6自然段:这六段的总叙(中心句)是什么?

明确:原来杨修为人恃才放旷,数犯曹操之忌。

(10)用简洁的语言概括六个小故事的主要内容。

明确:道破题字玄机;分吃一盒酥;揭穿“梦中杀人”;轻率揭发吴质;教曹植出门方法;教曹植作答。

(11)在所有“恃才放旷”、“犯曹操之忌”的事中,哪一件最令曹操忌恨?

明确:是为曹植作答教,实际上是干预了曹家兄弟的继位之争。这致使曹操极为反感,遂生“杀修之心”。

(12)杨修“恃才放旷,数犯曹操之忌”的六件事,按什么顺序排列?属叙事方式中的哪一种?

明确:按“犯忌”的轻重。根据是:操虽称美,心甚忌之;操虽喜笑,而心恶之;操闻而愈恶之;操因疑修谮害曹丕,愈恶之;操大怒,因此亦不喜植;操见了大怒曰:“匹夫安敢欺我耶!”此时已有杀修之心。

用了叙事方式中的补叙

(13)阅读后3自然段:曹操既杀杨修„„下令来日进兵。次日,兵出斜谷界口。开头写曹操“欲要进兵,又被马超拒守;欲收兵回,又恐被蜀兵耻笑,心中犹豫不决”,这里却下令进兵,是为什么?

明确:这是曹操掩盖想退兵的心事,进而掩盖杀杨修的用心。表现曹操的虚伪奸诈。

(14)下面写曹操强行进兵,结果惨败而回,还险些送了性命,这是不是与“杨修之死”无关?

明确:不是。这些与下文“方忆杨修之言„„”联系,证明先前杨修的分析和预言是非常正确的;再次揭露曹操以“惑乱军心”杀杨修是借口,真正原因是杨修“恃才放旷,数犯曹操之忌”。

(15)“方忆杨修之言,随将修尸收回厚葬,就令班师。”先“厚葬”近侍,又“厚葬”杨修,两次“厚葬”,刻画曹操什么性格?

明确:刻画曹操的虚伪奸诈

你一言我一语

3、在抢答的基础上,归纳杨修和曹操的性格特点 杨修:恃才放旷。具体包括:不拘礼俗、轻率大意,不管在什么场合、什么情况,都无所顾忌。

评:杨修这样的人,只可作学问,不可搞政治。

曹操:残酷、多疑、奸诈、虚伪。

评:“千古第一奸雄”

4、总结(教师(Teacher)完成)

(1)分出胜负方。

(2)明确杨修之死的原因

四、板书设计:

表面原因:鸡肋事件

惑乱军心

深层原因:恃才放旷

数犯曹操之忌

3.朗读任务:课文写了杨修犯曹操之忌的哪几件事?请用概括性的语言说出来。对杨修每次犯忌,曹操各有什么心理反应?朗读后做答。边朗读边指导朗读。

学生(Student)活动:找出小说的三要素。

人 物:曹操、杨修。

故事情节:曹操杀杨修。

环 境:战场。

复述文中的六个小故事。

明确:这六个故事分别是:破字,分酥,破梦,告密,出门,答教。

归纳:事件 心理

私解“鸡肋” 怒杀杨修

破译曹操写“活”字的用意 虽称美,心甚忌之

分食一合酥 虽喜笑,心甚恶之

揭穿曹操梦中杀人用意 愈恶之

密告曹丕 愈恶之

教曹植出城 大怒

替曹植作答教 有杀修之心

三、熟读课文,将文章分为三大部分,并归纳段意。

1.学生(Student)讨论后明确:

第一部分(第1——3自然段)写杨修被曹操所杀。

第二部分(第4——9自然段)记叙杨修和曹操之间矛盾的发展过程。

第三部分(第10——12自然段)写曹操兵败班师。

2.插叙的顺序。插入的六件事是杨修之死的根本原因。以杨修“身死因才误”做为行文线索。

3.初步思考:杨修和曹操给人留下什么印象?

明确:杨修:恃才放旷。

曹操:阴险虚伪、狡诈奸猾。

(关于这个问题的回答,学生(Student)只要能说出俩人性格的主要特点即可。)

第二课时

一、再读课文,研讨探究,概括杨修的死因。辩论活动,了解人物性格(学生(Student)各抒己见)

1 辩论活动,了解人物性格。

师:本文的故事情节是曹操杀杨修。任何事情的发展都是有其渊源的,杨修之死也是这样。这是情节发展的必然,也是他性格发展的必然。下面请同学们以“杨修之死是否完全归因于曹操”展开辩论。

男生为正方,观点为:杨修之死归因于曹操。

女生为反方,观点为:杨修之死不归因于曹操。

辩论要求:要有理有椐(你的观点要有事实依据)。

在辩论中理解人物性格特征

明确:曹操——阴险狡诈;杨修——恃才放旷。

总结归纳:一是由于杨修能够摸透曹操的心思,曹操既嫉妒他的才能,又考虑到留他在身边终不免造成祸患,总想找一个堂堂正正的罪名把他杀掉。这大概是许多人的共同看法。

二是杨修恃才放旷,为显示自己的聪明才智,置军纪于不顾,一闻“鸡肋”就自行收拾行装,并煽动其他人也做归计,因此,他的被杀是咎由自取。

三是由于杨修已深深卷入曹丕和曹植争夺接班的斗争之中,在曹丕已经得势的情况下,他必将成为这场斗争的牺牲品。曹操为身后接班人的安危考虑必定会杀掉他的。

二、精读课文,把握杨修、曹操的形象。(要求:

1、跳读课文,抓关键词;

2、品析关键词;

3、用短语概括总结二者的形象)

杨修

曹操

恃才放旷

阴险狡诈

才思敏捷

城府很深

卖弄聪明

表里不一

伺察敏锐

虚伪多疑

轻率大意

刚愎自用

果敢大胆

残忍歹毒

谋略不凡

暗藏祸心

料事如神„„

固执虚伪

老谋深算„„

两个人物都独具个性,请同学们看一看他们的照片(打出二人图象)但是同学们看这都是《三国》中他们的形象,那么在历史上他们就是这样的人吗?请大家回家搜集材料,经过研讨写个小报告。

三、课外拓展,为曹操、杨修各进一言,辨证看待历史人物

1 今天,我们通过学习《杨修之死》,了解到曹操性格阴险狡诈的一面,但是我们看问题要用一分为二的观点来分析,同学们心中的曹操是个什麽样的呢?

学生(Student)讲有关肯定曹操的故事:如他知人善用,他爱惜老百姓的麦田,他胸有大志等。(可背诵已经学过的《龟虽寿》《观沧海》等。)

教师(Teacher)总结:曹操阴险狡诈,奸诈狡猾,被称为“天下第一奸雄”,但这只是他性格中的一个方面,他还是个任人唯贤,善于招纳贤才,又有雄才大略的人。所以,他的性格是多面的,复杂的,我们要辨证的一分为二的看待历史问题。

2 那么,假如让你给曹操和杨修各进一言,你会和他们说些什么?(学生(Student)各抒己见,畅所欲言)

教师(Teacher)总结:现代青少年大多数都是独生子女,有一部分人在家都是小皇帝小公主,养成了一些自私,惟我独尊,狂妄自大的坏习惯。今天,有心理学家总结,杨修的死因与他的心理有关,只是当时的人没有从心理学角度去考虑罢了。一个有心理障碍的人往往表现为:无法正确调整情绪,控制个人言行,狂妄自大、放荡不羁、孤傲自负„..在这里老师希望大家在今后的学习生活中能互相理解、尊重,在当今这个提倡合作、团队精神的社会里找到自己的位置。

四、小结:通过今天的学习,我们了解了《三国》中杨修和曹操的性格特点,知道了杨修的死因,获得了一些启示,望大家能在杨修身上学到“有则改之,无则加勉”,祝愿每个人都能拥有美好和谐的人际关系。

五、结束语

最后,请大家欣赏毛阿敏演唱的电视剧《三国演义》的片尾曲,让咱们在悠扬的歌声中回忆那虽已逝去,但永留在我们心中的一个个鲜活的面容吧。

六、作业:试为曹操写“人物评论”。

19范进中举

教学目标

1 知识目标:理解文章内容,欣赏讽刺艺术。

2、能力目标:培养学生的阅读分析能力, 总结归纳能力,口头表达等能力。

3、德育目标:

① 认识封建科举制度对人民的毒害,珍惜今天的幸福生活。 ②培养团队精神。

教学重点:

1、学生通过讨论理解文章内容,欣赏讽刺艺术。

2、以学生自主学习为主,培养学生阅读分析、总结归纳、口头表达等能力。

教学难点:范进发疯的原因及文章主题的理解。 教学时间:二课时 教学过程

第一课时

一、导入

1 作者简介。

本文节选自《儒林外史》第三回,作者吴敬样(1701-1754),字敏。轩,安徽省全椒县人,清代小说家。

2.关于作品《儒林外史》是一部章回体长篇讽刺小说,是中国古典文学中讽刺艺术的最高峰,给后世的谴责小说以直接影响。它以整个封建社会为批判对象,以不同类型的封建知识分子为中心,通过对他们生活和心灵的描绘与剖析,勾画了一群“儒林”的丑恶形象。

二、整体感知

1.速读课文,给课文分段并归纳段意。 讨论并归纳:全文分成两个部分。 第一部分(l~2段):写范进进学回家到中举前的情况。 第二部分(3~结束):写范进中举后情景。

2.提问:课文是怎样描写范进的喜极发疯的过程? 讨论并归纳:范进喜极发疯的过程可分作四层。 第一层写昏厥。 第二层写疯跑。

第三层写跌倒。

第四层写疯走集上。。 3.提问:课文写范进的喜极而疯说明了什么?

讨论并归纳:在封建社会,读书人一旦中举,那就平步青云。范进末中举前穷愁潦倒,受尽屈辱。中举后,即能吐尽几十年的苦水闷气,从此将身价百倍,富贵荣华。因此,这是范进梦寐以求的,一旦到手,竟喜极而疯。文章着重刻画范进颠狂的丑态,揭露了封建科举制度对读书人的毒害。

4提问。范进是本文着力刻画的中心人物,试分析范进的人物形象。

讨论并归纳:范进是一个热衷功名的下层知识分子,大半生穷困潦倒,到54岁才考进秀才。他中举之前,穷得揭不开锅,邻里没有一个借米周济他。他地位卑微,受人歧视岳父可以任意辱骂他。他中了秀才.社会地位才有所改变,可是胡屠户对他的态度仍然十分粗野傲慢,臭骂他,训斥他,他总是习惯的“唯唯连声”,逆来顺受,甘受屈辱。范进中了举,情形就完全不同了。几十年来的贫困、屈辱一旦过去,梦寐以求的功名富贵一旦出现,政治、经济、社会地位一旦政变就使他惊喜得发了疯。那可憎可笑的疯癫形象被描绘得淋漓尽致、范进的喜疯病被胡屠户一骂一打治好后,他回忆疯额中的情景,别的都“昏昏沉沉”记不得唯独“记得是中的第七名”。课文刻画了范进这个为功名利禄而神魂颠倒的典型形象。

三、小结

本文着力刻画了范进这个热衷科举的下层知识分子的典型形象。描述了穷秀才范进中举前后的生活经历,深刻揭露并辛辣地讽束了封建科举制度,揭露了封建科举制度的腐朽及其对读书人的腐蚀和毒害。

四、作业

完成课后练习三第1题。

第二课时

一、导入

上节课,我们分析了范进的形象以及这个人物所体现的主题。这节课,我们再来探讨一下课本描写的另一个人物形象胡屠户。

二、正课

1.提问:范进中举前后,胡屠户对他的态度发生了哪些变化?课文从哪几个方面写出这些变化?表现胡屠户是一个什么样的人? 讨论并归纳:

(l)语言描写,

(2)动作描写,

小说运用对比手法,通过写胡屠户在范进中举前后截然不同的两种态度,形成鲜明的对比,使他的前倔后恭、欺贫爱富、趋炎附势、嗜钱如命、庸俗自私的典型市侩的性格,跃然纸上。

2.提问:张乡绅张静斋是作者刻画的一个次要人物,范进中举后,张乡绅为什么来拜会,又送银又送房?

讨论并归纳:他结交新贵,为的是攀附范进宗师周学道和杨知县,来巩固和扩大自己的权势。

3.提问:范进未中举前,家里断粮有没有乡邻来接济?中举后乡邻有拿鸡奎来的,有拿白酒来的,也有背了斗米来的,也有捉两只鸡来的,范进疯了都是众邻居来帮忙这表现了什么?

讨论并归纳:众乡邻对权势者或敬畏或滔媚,对不幸者冷酷无情。表现了封建社会的世态炎凉。 4.提问:讽刺是本文突出的艺术特征,想一想在笑的背后隐含着什么?

讨论并归纳:在笑的背后隐含着作者对科举制度怀着深恶痛疾的态度,作者痛感科举制度已把读书人腐蚀到了不可救药的地步。作者借小说人物的塑造,深刻揭露封建科举制度的腐朽以及对读书人的腐蚀和毒害。

三、小结

课文对比描写范进中举前后,特别是出榜那一天截然不同的境遇,着重刻画范进颠狂的丑态与他周围各色人等趋炎附势的嘴睑,批判的锋芒不仅对着科举制度而且指向整个封建社会。

四、作业

完成课后练习

一、

二、三题。

20、香菱学诗

教学目标:

1、知识与技能:整体把握文章的故事情节,学会作者以多种手法刻画人物形象及香菱成功的原因。

2、过程与方法:通过自主、合作、探究的学习方式,使学生掌握学习小说的要点与方法。

3、情感态度与价值观:通过学习本文,加深对《红楼梦》艺术成就的了解,不断提高学生的文学修养。

课程标准提出:“对文学作品,学生要能理解人物形象,初步鉴赏文章语言艺术”。“让学生具有创造性阅读的能力,注重情感体验,有较丰富的积累,形成良好的语感。”根据本文内容特点和学生年龄特点,本课的重难点设置为:

重点:分析香菱人物形象,学习香菱苦志学诗的精神。

难点: 香菱学诗取得成功的原因?1.通过学习本文,加深对《红楼梦》艺术成就的了解,不断提高学生的文学修养。 课时安排 1 教学过程:

一、导入新课。

二、整体感知

??

1、学生自读全文,识记生字生词和相关的文学常识。

??

2、本文的主要人物是谁?重要的人物还有几个?注意在文中画出能表现人物性格的语句。

三、研读赏析

??

1、学生讨论:香菱有着怎样的身世和处境?

??明确:香菱是一个从官宦小姐沦为奴婢的悲剧人物。她有着不幸的命运:从小遭受灾难,被拐卖,又被呆霸王薛蟠生拖死拽弄到家里,终于沦为侍妾,受尽凌辱和折磨。薛蟠遭打外出后,香菱住进了大观园,有机会接触大观园内这许多富于才情的少女们,特别是像林黛玉这样的才女,萌发了强烈的精神追求,本文就是从香菱刻苦学诗写起的。 ??

2、结合作品内容,讨论:香菱是一个怎样的人?从哪里可以看出来?明确:精华灵秀,悟性极强。学诗时“挖心搜胆,耳不旁听,目不别视”,已到了“呆”“疯”“魔”的程度。

??原来香菱是一个极富文学气质的人,她早就想学诗了,但苦于没有机会,只好自己弄本旧诗,偷空看两首。进入大观园后,深藏在内心的精神饥渴一下子勃发起来,进园的当晚就来找黛玉,希望向黛玉学诗。黛玉热情指导,列举名作让香菱阅读。香菱拿了诗回来“诸事不顾,只向灯下一首一首地读起来。宝钗连催他数次睡觉,他也不睡。宝钗见他这般苦心,只得随他去了”。

精神食粮的大量摄入,使香菱的内心充实丰富起来,她悟出了一些道理,她说:“诗的好处,有口里说不出来的意思,想去却是逼真的。有似乎无理的,想去竟是有理有情的。”她举了《塞上》一首为例说:“‘大漠孤烟直,长河落日圆。’想来烟如何直?日自然是圆的:这‘直’字似无理,圆字似太俗。合上书一想,倒像见了这景的。”香菱的艺术感受力很高,她读诗眼前就能出现诗歌中那动人的形象。香菱所体会到的,正是今天已众所周知的艺术辩证规律。

3、结合作品内容,讨论:课文中的林黛玉有怎样的性格特征?如何理解? 明确:在《红楼梦》这部作品中,多愁善感、羸弱多病、孤傲尖刻是林黛玉主要的性格,但在本文中我们却看到了一位诗人气质的林姑娘,哪里还有什么小性儿,她热情指导香菱,不厌其烦,不顾劳累,显现出了光风霁月般的襟怀。从她给香菱所开的书目来看,她的学识积累又是何等的深厚呵!

富于才情的林黛玉精神生活的一部分主要内容就是读诗、作诗。文学这片广袤的土地可供她这位寄人篱下、冰清玉洁的孤女自由地翱翔。联想个人的身世,林黛玉对香菱这无父无母的孤儿富有同情,也很有好感。所以当香菱求教于她时,她饶有兴致地承担起老师的责任。她说:“既要作诗,你就拜我作师。我虽不通,大略也还教得起你。”她简单明确地提出了作诗的要领,使香菱打消了不少顾虑,很快缩短了教与学之间的距离。

4、体会作者塑造人物的高超方法。

明确:曹雪芹是横绝千古的天才,他善于运用细腻准确的语言塑造人物。《红楼梦》中四百多个栩栩如生的人物,个个音容笑貌有别、性格鲜明独特。无论是正面的,还是反面的;无论是主子,还是奴仆,都给予了分寸恰当的描写,浓淡相宜,恰到好处,给人一种强烈的艺术美的享受。《红楼梦》里人物的出场入场,一颦一笑,来龙去脉,口角眉梢,心头话尾,舌尖牙缝,歌哭笑骂,正经,胡调„„没有一处不是活灵活现的。

例如,描写香菱刻苦构思诗歌就非常传神。在黛玉的诱导下,香菱的视野不断扩大,当她进入创作冲动状态时,形象光彩夺目:“连房也不入,只在池边树下,或坐在山石上出神,或蹲在地下抠土,来往的人都诧异”;写第三稿时“挖心搜胆,耳不旁听,目不别视”,甚至梦中喊出:“可是有了,难道这一首还不好?”对这个一心扑在诗歌创作中的人物,作者准确地描绘出她的一言一行和神情变化,联系香菱的这份苦心学诗、精血诚聚与她的悲惨身世,我们怎能不感叹造物者的不公!

这种通过人物语言和动作神态表现心理的艺术手法,是中国古典小说的优秀传统,而《红楼梦》则把它发展到极致。它与作家通过自己的叙述,从旁静止地分析人物心理相比,更显得跳脱生姿而又富于内涵,同时也更符合生活实际。

四、拓宽延伸

1.通过塑造香菱这一形象,寄予了作者怎样的思想感情? 明确:在本文中,作者以饱蘸深情的笔,讴歌了香菱这个精华灵秀所钟的薄命女子。香菱这样的人竟然和薛蟠生活在一起!而且受他的凌辱作践。那薛蟠只会胡扯“女儿愁,洞房里撺出了大马猴”之类,是极端粗陋鄙俗之人。香菱的境遇又那么值得同情惋惜!封建社会如此扼杀人才!不公平到这等地步!这正是需要读者明白的道理,也正是作者美学观点之体现。 2.有人说,“《红楼梦》既是女性的悲歌,又是女性的赞歌”,试作分析。 明确:《红楼梦》在描写人物时,女性无疑是最重要的,所以才有人评价说“《红楼梦》既是女性的悲歌,又是女性的赞歌”。曹雪芹以其精细的雕塑力,创造出许多个性鲜明、优美动人的女性形象。他不仅对袅娜风流的林黛玉,对有着炭火一般热情而被生生扑灭了的晴雯,而且对那个不惜用生命来证明自己清白的尤三姐都充满了强烈的同情和赞美。和封建社会蔑视女子、虐待女子的腐朽思想相反,曹雪芹提出了尊重女性,提高女性地位的新思想,并且歌颂了女子的多才多艺,赞扬了女子反迫害反虐待的斗争精神。他曾这样写到:“凡山川日月之精秀,只钟于女子,须眉男子不过是些渣滓浊沫而已。”把女子提高到比男子更高的地位。推崇女子,鄙弃男子,这就把几千年来“男尊女卑”不正当的地位颠倒过来,这样的思想认识对封建统治阶级宣扬的“男尊女卑”“夫为妻纲”的伦理纲常是一个猛烈的冲击,充分体现出曹雪芹反对封建礼教的超人勇气和远见卓识。

3、通过林黛玉指导香菱学诗,体现了曹雪芹怎样的诗歌创作思想?这对我们学习语文有何作用?

提示:林黛玉收香菱作学生,提出做诗要从以下三个方面进行:

首先,要多读。黛玉让香菱诵读王维的五言律诗一百首,杜甫的七言律诗一二百首,李白的七言绝句一二百首。“读诗百遍,其义自见”,诵读这种方法对于我们学写诗也是非常正确而必要的。随着诵读篇目数量的增加,你对诗歌的语言感知能力就会越来越强,诗歌的阅读水平也就会越来越高。这好比盖房子,先要打好地基,地基越牢固,房子也就可以盖得越高大。诵读就是学好诗歌的根基,这是提高鉴赏能力的根本途径。

其次,黛玉认为,要学诗就要学一流的。王维的五言律诗是最好的,除了杜甫,没有人能赶得上他;七言律诗,杜甫的诗要是打一百分的话,恐怕要再找出一个八十分的都不可能,后来一个李商隐也还可以;七言绝句,那是不会有人写得过李白的了。这三个人,李白是“诗仙”,杜甫是“诗圣”,王维人称“诗佛”,唐朝的诗歌,成就最高的就是他们三位了,以前一直是这样评价的。后来白居易取代了王维的位置,因为他的诗里面有一些反映劳动阶级的生活、思想等等,还有他的长篇,如《长恨歌》《琵琶行》都不错,但是以前一直是以李白、杜甫、王维三个人为首的。林黛玉这个看法也是很对的。要学就学一流的,学不到一流还可以成为二流,要是一开始就学二流的,那只能学成三四流了。我们在阅读时,也要挑选文质兼美的作品,这对于陶冶情操,培养纯正的文学趣味是非常有益的。

第三,要大胆创作,要敢于想像。香菱学诗,第一首,通篇就是说了“月亮很亮”这么一个意思。所以黛玉就说,初学者看的诗少,措辞不雅,要放开胆子去做,要有想像力才行。这也是很对的。恩格斯在评论德国诗人普拉顿的时候曾强调说:“写诗必须有大胆的想像。”可以说,没有想像就没有形象思维,也就没有诗。形象思维的过程自始至终都贯穿着丰富的艺术想像,大胆的想像可以使极平常的生活景象焕发出奇特的美感、美质,令人耳目一新。

《红楼梦》中的这些文艺创作理论对我们都有很大的启示作用 五作业

课后练习二

21、陈涉世家

教学目的

知识与能力目标:

1、了解陈涉起义这一伟大的历史事件,了解《史记》与司马迁;

2、理解课文的基本内容;

过程与方法目标:

3、熟读课文,疏通文意;

情感与价值目标:

4、了解陈胜、吴广领导的农民起义的经过及其历史意义。教学重点

了解陈胜、吴广领导的农民起义的经过及其历史意义。 领会课文运用对话表现人物性格的写作特点。

教学难点

初步了解文言文中一词多义、成分省略等语言现象。 课时安排 3课时

第一课时

教学内容:

解题,了解课文的内容结构,分析课文第1段。 教学步骤

1.导入新课:

在中国历史上发生过多次农民起义,这些农民起义为残酷的封建统治所迫,充分说明了它通民反的理。每次农民起义都沉重地打击了黑暗的封建统治,有的甚至推翻了封建王朝,是推动历史前进,促使变革封建生产关系,促进生产力发展的动力。中国历史上第一次农民起义是发生在秦朝末年的陈胜、吴广领导的大泽乡起义,这次农民起义建立了我国历史上第一个农民政权??张楚,给予秦朝的暴政以致命的打击,促使了秦速亡,使由秦始皇开始的而想万世统治中国的秦王朝只存在短暂的15年便被刘邦的西汉政权所取代。陈胜、吴广是什么样的人,他们是在什么情况下发动起义的,起义的经过怎样,我们学习了司马迁写的《陈涉世家》就知道了。

2.解题。学生阅读注释①,教师可作如下补充。

①司马迁的父亲司马谈,学识渊博,在汉武帝时就任太史令(史官),掌管秘籍、史料、职司记载等。司马迁自幼受父亲熏陶,10岁就到长安学习,20岁起曾多次到南北各地游历,观察各地的风俗人情,采集传闻异说,为后来写作《史记》打下基础。38岁时继任太史令,更有机会博览皇家藏书,掌握了丰富的史料。42岁时开始写作《史记》。后由于上书替投降匈奴的汉将李陵辩解而触怒是上,获罪下狱,被处官刑。为完成《史记》,司马迁忍辱负重,发愤著书,历时10多年,终于写成历史巨著《史记》。

(2)〈史记》是我国第一部纪传体通史,也是我国第一部传记文学。这部书记载了上自黄帝不至汉武帝的三千多年历史。全书包括十二“本记”(记历代帝王政迹),十“表”(大事年表)、八“书”(记各种典章制度)、三十“世家”(记诸侯国兴亡)、七十二“列传”(记重要历史人物的言行事迹)。书中史料翔实,作者思想倾向也极为鲜明。例如课文《陈涉世家》,作者把农民起义的领袖陈涉的传记列在“世家”里,足见他对陈涉的历史地位和作用看重。鲁迅先生曾称赞《史记》为“史家之绝唱,无韵之《离骚》”。

③秦始皇统一中决定起义后,陈胜、吴广首先分析天下形势,拟定斗争策略。“天下苦秦久矣”,简明而又准确地概括了当时的形势,揭露了秦王朝的苛政,表达了人民群众积蓄已久的不满与反抗情绪,揭示了暴发农民起义的必然性。在这种形势下举行起义,一定会得到人民群众的支持和响应。陈胜又分析了统治集团内部的矛盾,制定了借深得人心的扶苏、项燕的名义号召群众,发动起义的斗争策略。陈胜坚信,这样做一定会得到“宜多应者”的效果。然后为起义作舆论准备,他们运用问卜、念鬼、鱼腹藏书,篝火狐鸣等方法制造舆论,鼓动人心,树立威信,使人们相信“大楚兴,陈胜王”的必然趋势,结果在士卒中很快得到反应。陈胜的做法虽然是借助鬼神迷信来争取群众,但作为一种策略,充分显示了陈胜、吴广的斗争才智。

3.学生齐读课文,体会课文记叙方法上的特点。

本段仍以人物对话为主,间以叙事。陈胜和吴广商议大计,分析天下形势,制定斗争策略等,都以他们对话的形式向读者表达,让读者如闻其声,如见其情,对他们的性格理解得更透彻。

第二课时

三、分析课文第3段

1.教师领读或由学生自读课文后,参阅注释理解文义,并注意下边加点词的意思:

吴广素爱人:一向。向来。广故数言欲亡:故:故意。亡:逃跑。陈胜佐之:帮助、协助。成死者固十六七:固:本来。十六七:十个里面有

六、七个。从民欲也;顺从。为坛而盟:建筑。祭以尉首:用。将兵徇蕲以东;率领。车六七百乘:古时一车四马为一乘。骑千余:古时一人一马为称骑。将军身被坚执锐:通“披”。陈涉乃立为王:于是被拥立。诸郡县苦秦史者:被„„压迫的。杀之以应陈涉:虚词“来”。

2、这段文字记叙了陈胜、吴广发动起义的经过和起义后的浩大声势,以及建立农民政权的情况。可讨论如下问题:

①陈胜、吴广是通过哪些步骤发动起义的? 陈胜、吴广发动起义通过王个步骤:“并杀两尉”揭开了起义的序幕,接着发动群众参加起义,然后建立领导系统。“并杀两尉”选择了“将尉醉”的适宜时机,又设下激尉要杀吴广的圈套,使众人认为杀之有理。这又表现了起义领袖的机智。杀尉后陈胜当即号召群众起交,他先晓以利害,指出如不起义,只有死路一条。再提出“壮士不死即已,死即举大名耳”,激励众人立下雄心壮志,为国为民干一番事业。尤其提出“王侯将相宁有种乎?”对封建统治阶级提出了有力的挑战,道出了千百万贫苦农民的心声,显示了对起义胜利的坚定信心和要做天下主人的英雄气概。陈胜的这些话,说得慷慨激昂,激动人心,果然得到众人的热烈拥护和响应,起义队伍很快组织起来了。

②起义发动后,起义队伍为什么能形成浩大的声势并建立起农民政权? 起义发动后,起义军斗争矛头直指秦政权,攻城略地,势如破竹,所向披靡。在战斗中起义,队伍迅速扩大,很快建立了农民政权??张楚。究其原因,除起义领袖敢于反抗,胸怀大志,正确谋划,策略得当外,还有九百徒后的热烈响应,勇敢战斗。然而最根本的原因是秦的暴政逼得人民不得不反。“天下苦秦久矣”,深刻揭示了秦王朝对贫苦人民的压迫剥削程度之深,致使怨声载道,人心思变,使农民起义有着坚实的基础。三老、豪杰的话“伐无道,诛暴秦”就点明了起义的性质,也是历史的必然,所以农民起义是不可抗拒的历史潮流。故而陈胜、吴广发动起义后,很快就形成了法大的声势,不久农民政权也建立起来了。 3.学生齐读课文后,讨论这段文字在材料安排和记叙方法上有什么特点?

材料安排上的特点是洋略结合。详写起义的发动经过,对陈胜、吴广“并杀两尉”、发动群众等都详细描写,并把叙事和对话结合起来,让起义领袖的机智勇敢,强烈的反抗精神得以充分的体现。而对起义后的浩大声势及建立农民政权则略写,省去了战斗情况和筹建政权的描写,因为从上文看来这已是情理中的事。记叙方法上的特点是,以侧面描写表现陈胜、吴广的功绩。借三老、豪杰的嘴,赞颂农民起义的功业,以诸郡县杀秦朝酷交响应陈胜,表现农民起义得到全国各地的拥护。

四、概括课文内容和中心思想:

课文记叙了陈胜、吴广领导的我国历史上第一次农民起义的原因,经过和浩大声势,表现了农民起义的伟大力量,赞颂了陈胜、吴广的反抗精神及历史功绩。

五、课后练习与思考:

复习课文,注意保文中一词多义,成份省略等语言现象。

第三课时

教学内容

领会课文以对话表现人物性格的写作特点,了解文言文中一词多义,成份省略等语言现象。 教学步骤

1.导入新课:

前二课时我们分析了课文内容,了解了陈涉起义的原因,过程和起义后的浩大声势。这次农民起义沉重地打击了秦王朝的政,揭示了官逼民反的道理,也让读者了解了起义领袖敢于反抗,善于斗争,要为国家干一番事业的雄心大志。那么课文在记叙农民起义的过程中是怎样刻画人物形象的,作为起义领袖的陈胜的性格是通过什么方法表现出来的呢?本课时我们先来讨论这一问题。 2.学生默读课文后,讨论课文表现人物性格的写作特点。

表现人物性格的方法很多,一般通过人物的育行来表现。课文刻画的主要人物是陈胜。从课文内容看,陈胜表达志向、谋划起义、发动起义等都是透过他的语言来表现的。可见识文主要是通过语言,尤其是通过人物间的对话来表现人物性格的。纵观全文,陈胜参与对话的共有三次,第五段中,陈胜同其他佣耕者的对话,表现了陈胜的远大志向和对封建统治阶级的反抗精神。第2段中,陈胜同吴广的对话,表现了他们组织起义的谋略才智。他们对当前形势的正确分析、起义策略的确定、怎样制造舆论等都是通过对话表达出来的。第3段中,陈胜同成卒的对话,又让我们看到了一个敢于反抗,敢于斗争,充满英雄气概的起义领袖形象。可见保文就是通过人物对话把陈胜等人的性格极为鲜明生动地表现出来了。

3.选读课文中有关陈胜参与对话的内容,找出最能表现他性格的语言,并加以适当的评论。最能表现陈胜性格的语言有;1)苟富贵,无相忘”。②“燕雀安知鸿鹄之志哉!”③“今亡亦死,举大计亦死,等死,死国可乎?”④“壮志不死即已,死即举大名耳,王侯将相宁有种乎!”

①句表达了陈胜极想摆脱贫困,改变现状的强烈愿望,反映了陈胜具有“有相同车,有难同当”的朴素阶级感情,说明陈胜虽然出身低贱,但胸怀大志,不寻常人。②句以“燕雀”比喻见识短浅的人,以“鸿鹄”比喻有远大理想的人。通过比喻,表现陈胜年轻时就是个有理想有抱负的人,也反映了他对同伴目光短浅的惋惜名句说明了陈胜面对秦王朝的暴政,面对死亡,毫不畏惧,决心不受命运的摆布,为建立自己理想中的国家而举行起义,表现了他勇于反抗的大无畏英雄气概,也是他“鸿鹄之志”的具体表现。④句彻底否定了封建统治阶级欺骗劳动人民的所谓天生贵种、等卑有序、贵贱有别的谎言,砸碎了长期禁锢劳动人民头脑的封建精神枷锁,是鼓舞人心的战斗口号,表现了陈胜敢于解放思想,敢于推翻封建统治的斗争精神。由此可以看出,陈胜是一个对封建统治强烈不满,敢于反抗,敢于斗争,善于斗争的有志气,有抱负的农民起义的领袖。

4.文言文在表达上常出现一词多义,成分省略等语言现象,有些句子还形成了特有的固定形式。讨论下边句中加点词的 词义和用法:

①项燕为楚将( 【大将,名词〕上使外将( )兵【率领,动词」公将( )鼓之【将要,副词】

③以数谏故[ 因为,连词」杀之以应陈涉【虚词“来”,连词」以吾众诈自称公子扶苏,项燕【把,介词」祭以尉首〔用,拿,介词〕

以上两组句中加点的词意义不同,词性也不相同,有的读音也不同,应该根据不同的语言环境,联系上下文的意思,确定们不同的意义。再讨论下边句中成分省略情况:①衣食所安,(吾)弗敢专也,必以(之)分人。主语 介宾短语中的宾语

②一鼓作气,再(鼓)而(土气)衰,三(鼓)而(士气)竭。谓语 主语 谓语 主语

③扶苏以数谏(上)故,上使(之)外将兵。宾语宾语

④独守丞与(之)战(于)谯门中。宾语 介词 由以上4个例句可以看出,文言文中成分省略的句子是常见的,以省略主语、谓语、宾语和介词为最多。理解翻译时应根据上下文意思补出。

22.唐雎不辱使命

教学目标

知识与技能目标 学习本文通过对话描写表现人物形象的写作方法 培养复述、翻译、朗读的能力和严密的逻辑思维能力 过程与方法目标 领悟本文言辞的巧妙美

情感态度目标 学习唐雎不畏强暴、敢于斗争、善于斗争的精神

教学重点通过对话描写表现人物形象

教学难点领悟本文言辞的巧妙美 课时安排 2 教学步骤

第一课时

导入新课

同学们也许听说过战国时期荆轲刺秦王的故事吧。荆轲在易水上辞别燕太子丹时,慷慨悲歌,“风萧萧兮易水寒,壮土一去兮不复还”。这就是《战国策》里描绘的燕赵游侠之士的形象,虽然他最终失败被杀,但他的故事却一直为后。人传诵,给我们留下了深刻的印象。今天我要讲的则是另一个故事:作为小国之臣的唐雎,在秦王寻衅威胁的情势下出使秦国,能够毫不畏惧,敢于同秦王展开面对面的斗争,最终理直气壮地折服秦王,不辱使命。这种勇气是难能可贵的。这里的唐雎,是《战国策》里塑造的又一个游侠勇士的形象。 (一)教学过程

1.作者、背景简介。

《战国策》是西汉末年刘向编写的一本以记言为主的史书,是一部国别体史书。它叙事年代上接春秋,下至秦灭六国为止,约二百三四十年时间,是我们研究战国时期历史的一部重要著作。它叙事长于铺陈,多加渲染夸张,写得有声有色;语言生动流畅;善于在矛盾冲突中运用对话刻画人物性格,人物形象比较鲜明丰满;在论辩中往往采用故事或寓言作比喻来说明抽象的道理,富有文学色彩。所以它对后代的散文著作有很大影响。

这篇文章记叙了战国末期的一段故事:公元前230年和公元前225年,秦先后灭亡了韩魏两国;安陵是魏的附庸小国,秦想用诈骗手段并吞安陵。安陵君拒绝秦王的要求,在秦王不悦的情况下派遣唐雎出使秦国,目的就在于保全国土,解除与秦国之间的矛盾。

2.全班齐读,然后正音正字。

弗fú 雎jū 怫fú 跣xi?n 抢qiāng 傀guī ?jìn 缟g?o 挠náo 谕yù 3.同学反复朗读,可按照每行为一组的顺序举行朗读比赛,以此激发学习读书的乐趣。

4.借助工具书和参考资料,自行疏通文意,同桌之间可以讨论。

5、文章可以划分为几个部分?全文共四段,可以分三个部分。第一部分(1)叙述唐雎出使秦国的缘由。 第二部分(2-3)写唐雎和秦王面对面的斗争。 第三部分(4)写秦王被唐雎的浩然正气所折服。

(四)整体感知

请同学们用一句话说出课文的主要内容。

本文通过唐雎面对暴秦展开斗争直至折服秦王的记述,赞扬了唐雎坚持正义、勇敢无畏的爱国精神。全文短小精悍,全部用人物对话来叙写和展开情节。 (五)总结、扩展

本课注重了学生朗读能力的培养,疏通了文意,使大家对《唐雎不辱使命》一课有了整体的感知。在战国时期有很多“士”,这个特殊的阶层流传下来许多故事,如毛遂、蔺相如、晏子等,课下可搜集一些。 (五)布置作业

认真阅读,逐句翻译,以小组为单位编写剧本,突出唐雎和秦王的舌战。 第二课时 导入新课

《战国策》论证说理周密严谨,笔锋犀利。综论形势,无不纵横捭阖;指陈利害,无不锋芒毕露。《唐雎不辱使命》以短小的篇幅,写了一个结构完整的故事,情节曲折,人物有鲜明的个性,使人读来如见其人,如闻其声,如临其境。今天我们就来体会一下。 教学过程

一、分各色朗读课文。

二、根据课文内容的理解,简要回答下列问题。1.为什么说唐雎没有辜负使命? 秦王要求用五百里土地换安陵君的封邑,实际是要并吞安陵。而在当时这是不可抗拒的。唐雎奉命出使,凭着自己的机智勇敢,面对横蛮狡诈、色厉内荏的秦王,不畏强暴,敢于斗争,不被威势压服,最后使秦王放弃了易安陵的打算。唐睢维护了本国的尊严,出色地完成了使命。

2、秦王与安陵君“易地”并非真心,何以见得? 秦王并非真心易地,意在吞并。A.“秦王使人谓安陵君曰”可看出秦王对安陵君的轻视;B.“寡人欲以„„安陵君其许寡人”可看出秦王口气强硬,并无商量余地;C.秦王不说,可看出秦王名易实夺的心理

3、安陵君拒绝了秦王的无理要求,实际上是一眼看穿了秦王的真正用心,那为什么还要说“大王加惠,以大易小,至善”的话呢? 安陵君虽然对秦王的野心洞若观火,无奈在敌强我弱的形势下,为争取周旋的余地,没有当场揭穿秦王的骗局。

4、本中怎样体现秦的强大和安陵的弱小?

5、说说“秦王不悦”四个字在故事情节发展中的作用。

“秦王不悦”引起故事的开端,贯穿整个事件的全过程,对情节发展起着推动作用。

6、秦王说,“且秦灭韩亡魏,而君以五十里之地存者,以君为长者,故不错意也”的盲外之意是什么? 言外之意是我秦王如果不“以君为长者”,“错”起“意”来,就会把你和韩魏一样灭亡。

7、秦王说,“天子之怒„„千里”,唐雎说,“伏尸二人„„今日是也。”他们各表达了什么意思? “天子之怒”是秦王慑服唐雎的话,意为要发动战争,消灭安陵;“布衣之怒”是唐雎回击秦王的话,意思是你要我的地,我就要你的命,而且我不惜自己的生命保卫安陵。

8、文中秦王、唐雎分别是一个怎样的形象?

秦王:骄横、狂暴、狡诈、阴险落架 。唐雎:有胆识、正气凛然、不畏强暴。

三、教师小结。

这篇记叙文,写了唐雎忠于使命,不畏强暴,敢于斗争,敢于胜利的英雄气概,揭露了秦王的骄横欺诈,外强中干,色厉内荏的本质,虽不假修饰,却十分鲜明生动,在刻画人物性格方面,取得了很高的成就。

第20篇:第一学期九年级语文工作总结

2013-2014学年第一学期九年级语文教学工作总结

在上好新课的基础上,稳定和鼓励学生,依据计划有目的的进行艰苦的强化训练,从基础知识、语言运用、古诗文默写、阅读、写作等方面作了系统的训练,复习和巩固了所学识,有目的进行艰苦的强化训练是取得成功的基础。试作小结如下:

首先做好学生思想政治工作,明确本学期的学习目标,激发学生学习的热情。本学期时间紧,任务重,几乎是一个月一次大考,每一次考试都对学生们具有重要意义。基于此,我充分利用课内外时间,形式多样地做好学生心理稳定工作。思想问题解决了,学生们就能够安心学习,成绩也有所上升。

备好课,努力提高驾驭课堂的能力。根据这个班的同学比较活跃,上课气氛积极,但中等生、差生占较大多数,备教法的同时也备学生。备课充分,吃透教材,就能调动学生的积极性,上课效果就好。

掌控课堂节奏,注意学生反映。学生在课堂上的一举一动都会直接影响课堂教学。因此上课一定要设法令学生投入,不让其分心。课堂内容丰富,翔实。教态自然,讲课生动,难易适中,照顾全部,就自然能够吸引住学生。所以,每次上课,不管是多么地困乏,都要在上课之前调整好自己的精神状态,每天都要有充足的精神,饱满的热情,让学生感受到一种积极向上的情绪,以这种情绪来感染他们。这样,学生上课就有了一股“劲儿”,授课就事半功倍。当学生在课堂上无心向学,违反纪律时,我尽量采取合适的方式或用手势、或用眼光、或用简单的语言及时提醒,尽量使自己的情绪不受到影响,不带到教学中,避免让原本正常的讲课受到冲击,影响教学效果。

面对全体学生,尤其注意培优补差,加强对后进生的转化工作。语文是一门工具学科,对学生而言,既熟悉又困难,在这样一种大环境之下,要让众多的基础较差的学生学好语文,就要让他们喜爱语文,对语文产生兴趣。否则学生对这门学科产生畏难情绪,不愿学,也无法学下去。为此,我采取了一些方法,就是尽量多讲一些文化生活故事,多读一些源于生活的优秀文章,让他们更了解语文,学会欣赏语文之美,更喜欢学习语文。因为只有语文水平提高,他们才能提高同学们的语文写作能力,对成绩优秀的同学也很有好处。

研究中考试题,提炼解题规律,加强个人学习,研究中招试题的基本形式、考察方向、中招考点等,规范学生答题,加强与教研组内的老师的交流与合作,做好对学生应对中招考试的指导。同时借鉴一些名教师和专家们的做法、经验和理论,

经过一个学期的努力,教学效果和成绩都有待进一步去检验。但是我相信,无论成绩高低,都体现了我在这学期的教学成果。时光流逝,一切都已经成为过去,过去并不是最重要的,重要的是在以后如何自我提高,不断改进自己在教育教学上的不足,使自己在业务素质上不断进步。因此,无论怎样辛苦,我都会继续努力。

崇明县第一学期教学质量调研测试卷九年级语文
《崇明县第一学期教学质量调研测试卷九年级语文.doc》
将本文的Word文档下载到电脑,方便编辑。
推荐度:
点击下载文档
相关专题
点击下载本文文档